You are on page 1of 105

GENERAL P RINCIPLES : ACTUAL CONTROVERSIES 1.

SPS SARMIENTO v SPS MAGSINO Mootness Facts: SPS Magsino bought land from SPS Calderon however it turned out that it was already sold to 3rd parties. SPS Magsino filed a Complaint for Specific Performance, In the alternative, reimbursement of the amount of P383k plus interest, costs of suit in RTC93. RTC granted the alternative, Sold some properties of SPS Calderon with SPS Magsino as highest bidder. It turned out however that these properties were in the possession of SPS Sarmiento who already applied for a TRO & Preliminary Injunction in another court (RTC31). The sheriff successfully evicted SPS Sarmiento. SPS Sarmiento appealed to SC. Issue: REAL Issue: whether or not RTC Branch 31 interfered with the judgment and order of RTC Branch 93, a co-equal court, when it issued preliminary injunction restraining Magsino from occupying the subject properties and ordering them to vacate the same, which in effect enjoined the enforcement of the writs of execution and possession issued by RTC Branch 93? Held: DOES NOT MATTER, ISSUE IS MOOT. Court refuses to discuss. Ratio: RTC Branch 31 has already decided SPS Magsinos Complaint in their favor and ordered the RD of to cancel TCT Nos. Sps Calderon. It also made permanent the injunction and pay attorneys fees and the cost of suitrAlso,spsmagsino declared that they remained in possession of the subject properties. This Petition has become moot and academic as the issue or issues to be resolved herein are merely in relation to the incidents of the main case It is a rule of universal application that courts of justice constituted to pass upon substantial rights will not consider questions where no actual interests are involved; they decline jurisdiction of moot cases. And where the issue has become moot and academic, there is no justiciable controversy, so that a declaration thereon would be of no practical use or value. There is no actual substantial relief to which the petitioner would be entitled and which would be negated by the dismissal of the petition.

Thus, the Court will refrain from expressing its opinion in a case where no practical relief may he granted in view of a supervening event. In sum, the resolution of the issue or issues in this case would be of no practical use or value as the merits of the case has already been decided upon by RTC Branch 31 and the same has been decided in favor of petitioners. Ratio by CA/RTC: 1. While the general rule is that no court has the authority to interfere with the judgment or decrees of another court of equal or concurrent or coordinate jurisdiction, it is not so when a third party claimant is involved. The general rule is confined to cases were the property belongs to the defendant or one in which he has proprietary interest. But when the sheriff, acting beyond the bounds of his office seizes a strangers property, the rule does not apply and interference with his custody is not interference with another courts order. 2. Restoration of the SPS Magsino to the possession of the properties is not tantamount to the disposition of the main case, but is a mere provisional remedy and is not determinative of the question of validity of the titles.

GENERAL P RINCIPLES : R ULE - MAKING

POWER OF THE

S UPREME COURT

2. PHILSA INTERNATIONAL EMPLOYMENT Rule-making power of SC

VS.

SEC.

OF

LABOR

AND

Facts: Petitioner Philsa International Placement and Services Corporation is a recruitment agency for overseas workers. Private respondents (Mikin, de Mesa, and Leyson), who were recruited by petitioner for employment in Saudi Arabia, were required to pay placement fees. While in Saudi Arabia, private respondents were allegedly made to sign a second contract which reduced some of their benefits and privileges. Their foreign employer allegedly forced them to sign a third contract which increased their work hours from 48 hours to 60 hours a week without any corresponding increase in their basic monthly salary. When they refused to sign this third contract, the services of respondents were terminated. Upon their arrival in the Philippines, private respondents demanded from Philsa the return of their placement fees and for the payment of

their salaries for the unexpired portion of their contract. Philsa refused so they filed a case before the POEA against Philsa for illegal dismissal, payment of salary differentials, illegal deduction/withholding of salaries, refund of placement fees; and contract substitution. Philsa was ordered to pay respondents their separation pay, salary deduction and differential pay. Petitioner and private respondents filed separate appeals from the POEA Decision to the NLRC. NLRC modified the appealed decision of the POEA Adjudication Office by deleting the award of salary deductions and differentials. These awards to private respondents were deleted by the NLRC considering that these were not raised in the complaint filed by private respondents. Private respondents elevated the decision of the NLRC to the Supreme Court in a petition for review for certiorari and the court dismissed the case for insufficiency in form and substance, having failed to comply with the Rules of Court and Circular No. 1-88 requiring submission of a certified true copy of the questioned resolution dated August 23, 1989. POEA also found petitioner guilty of illegal exaction, contract substitution, and unlawful deduction. Petitioner insists that it cannot be held liable for illegal exaction as POEA Memorandum Circular No. II, Series of 1983, which enumerated the allowable fees which may be collected from applicants, is void for lack of publication.

to enforce or implement existing law pursuant to a valid delegation. The only exceptions are interpretative regulations, those merely internal in nature, or those so-called letters of instructions issued by administrative superiors concerning the rules and guidelines to be followed by their subordinates in the performance of their duties. Administrative Circular No. 2, Series of 1983 has not been shown to fall under any of these exceptions. 3. KLAVENESS MARITIME AGENCY V. PALMOS (PENDING) Rule-making power of SC 4. EDUVIGES BELTRAN ESPIRITU And FLORINDA B. ESPIRITU, VS.THE COURT OF APPEALS, PEDRO DEL ROSARIO. Rule-making power of SC FACTS: This is an appeal by the petitioners from the unfavorable judgment by default rendered by the Court of First Instance of Rizal in its Civil Case No. Q-7537, an action of respondent Pedro del Rosario to annul a contract of sale of certain lands entered into between petitioners and a certain corporation denominated Golden Earth, Inc. which was never joined as a party in the lower courts, and whose motion for intervention filed with this Court had to be denied for being obviously filed out of time, declaring the impugned sale to be null and void. Petitioners assail the trial court's order declaring them in default. In the face of the finding of the Court of Appeals that in fact, service of summons for both petitioners who are mother and daughter, respectively, was made upon petitioner Florinda B. Espiritu, the daughter, at their house at Tenejeros, Malabon, Rizal and that said petitioner paid no heed thereto because she thought the papers received by her from the Sheriff, for herself and for her mother, were not important. Since they seemed to refer to another case between her and her mother, on the one hand, and respondent Del Rosario, on the other, which had already been settled with the approval of the Court. There is reason to believe, in absence of any showing to the contrary, that Florinda B. Espiritu was a person of suitable age and discretion on the occasion the summons were served on her at their residence. Under these circumstances, it is baseless for petitioners to pretend there has been a denial of due process to them. ISSUE/S: 1. Whether or not the appellate court committed an error when it held that there was a valid substituted service upon petitioners under Section 8 of Rule 14

Issue: W/N the POEA Memorandum is void for lack of publication. Held: Yes. Ratio: The administrative circular under consideration is one of those issuances which should be published for its effectivity, since its purpose is to enforce and implement an existing law pursuant to a valid delegation. Considering that POEA Administrative Circular No. 2, Series of 1983 has not as yet been published or filed with the National Administrative Register, the same is ineffective and may not be enforced. The fact that the said circular is addressed only to a specified group, namely private employment agencies or authority holders, does not take it away from the ambit of our ruling in Taada vs. Tuvera. In the case of Phil. Association of Service Exporters vs. Torres, the administrative circulars questioned therein were addressed to an even smaller group, namely Philippine and Hong Kong agencies engaged in the recruitment of workers for Hong Kong, and still the Court ruled therein that, for lack of proper publication, the said circulars may not be enforced or implemented. Our pronouncement in Taada vs. Tuvera is clear and categorical. Administrative rules and regulations must be published if their purpose is

HELD: 1. No. There was no error committed by the appellate court RATIO: 1. The court held that there is reason to believe, in absence of any showing to the contrary, that Florinda B. Espiritu was a person of suitable age and discretion on the occasion the summons were served on her at their residence. Under these circumstances, it is baseless for petitioners to pretend there has been a denial of due process to them. What is decisive here is that the Court of Appeals annulled, as did the trial court, a contract, not between the plaintiff and the defendants before them, but between said defendants and a third person, not a party to the action, the Golden Earth, Inc., vendee of the lands in question from petitioners. It is beyond question that Golden Earth, Inc. is an indispensable party in this case and its absence renders ineffective all the proceedings subsequent to the filing of complaint, including the judgment. What the trial court should have done upon perceiving that the vendee in the sale sought to be annulled was absent was to order that it be impleaded. DISPOSITIVE PORTION (important for this case): WHEREFORE, the decision of the Court of Appeals is reversed and it is ordered that this case be remanded to the trial court so that it may proceed in accordance with the above opinion, more particularly, to have Golden Earth, Inc. impleaded, and try the case all over again after having summons served anew on petitioners, thereby considering as of no effect its questioned default order. 5. ECHEGARAY v. SECRETARY OF JUSTICE Rule-making power of SC FACTS For resolution are public respondents' Urgent Motion for Reconsideration of the Resolution of this Court dated January 4, 1999 temporarily restraining the execution of petitioner and Supplemental Motion to Urgent Motion for Reconsideration. Petitioner filed his Very Urgent Motion for Issuance of TRO on December 28, 1998 at about 11:30 p.m. He invoked several grounds, viz: (1) that his execution has been set on January 4, the first working day of 1999; and (b) that members of Congress had either sought for his

executive clemency and/or review or repeal of the law authorizing capital punishment. When the Very Urgent Motion was filed, the Court was already in its traditional recess and would only resume session on January 18, 1999. Even then, Chief Justice Hilario Davide, Jr. called the Court to a Special Session on January 4, 1999 at 10. a.m. to deliberate on petitioner's Very Urgent Motion. The Court hardly had five (5) hours to resolve petitioner's motion as he was due to be executed at 3 p.m. Thus, the Court had the difficult problem of resolving whether petitioner's allegations about the moves in Congress to repeal or amend the Death Penalty Law are mere speculations or not. They noted that petitioner's allegations were made in a pleading under oath and were widely publicized in the print and broadcast media. The present Congress is different from the Congress that enacted the Death Penalty Law (R.A. No. 7659) and the Lethal Injection Law (R.A. No. 8177). However, the minority noted that there was hardly a time to verify petitioner's allegations as his execution was set at 3 p.m. Given these constraints, the Court's majority did not rush to judgment but took an extremely cautious stance by temporarily restraining the execution of petitioner. The suspension was temporary - - - "until June 15, 1999." The extreme caution taken by the Court was compelled, among others, by the fear that any error of the Court in not stopping the execution of the petitioner will preclude any further relief for all rights stop at the graveyard. ISSUE W/N the Court has lost its jurisdiction over the case at bar when the same has become final and executory, hence the Court can no longer restrain execution of Echegaray HELD NO RATIO Contrary to the submission of the Solicitor General, the rule on finality of judgment cannot divest this Court of its jurisdiction to execute and enforce the same judgment. Ret. Justice Quiason: By the finality of the judgment, what the court loses is its jurisdiction to amend, modify or alter the same. Even after the judgment has become final the court retains its jurisdiction to execute and enforce it. There is a difference between the jurisdiction of the court to execute its judgment and its jurisdiction to amend, modify or alter the same. The former continues even after the judgment has become final for the purpose of enforcement of judgment; the latter terminates when the judgment becomes final.

It should be stressed that the power to promulgate rules of pleading, practice and procedure was granted by our Constitutions to this Court to enhance its independence. The 1987 Constitution molded an even stronger and more independent judiciary. Among others, it enhanced the rule making power of this Court. Refer to Section 5(5), Article VIII. The rule making power of this Court was expanded. This Court for the first time was given the power to promulgate rules concerning the protection and enforcement of constitutional rights. The Court was also granted for the first time the power to disapprove rules of procedure of special courts and quasi-judicial bodies. But most importantly, the 1987 Constitution took away the power of Congress to repeal, alter, or supplement rules concerning pleading, practice and procedure. In fine, the power to promulgate rules of pleading, practice and procedure is no longer shared by this Court with Congress, more so with the Executive. If the manifest intent of the 1987 Constitution is to strengthen the independence of the judiciary, it is inutile to urge, as public respondents do, that this Court has no jurisdiction to control the process of execution of its decisions, a power conceded to it and which it has exercised since time immemorial. 6. REPUBLIC v. GUINGONA Rule-making power of SC FACTS: The petitioner seek the declaration of the unconstitutionality of P.D. No. 81, Sections 31 of P.D. 1177, and P.D. No. 1967. The petition also seeks to restrain the disbursement for debt service under the 1990 budget pursuant to said decrees which costs around P86 BILLION IN THE P233 BILLION 1990 BUDGET. P.D. No. 81, entitled "Amending Certain Provisions of Republic Act Numbered Four Thousand Eight Hundred Sixty, as Amended (Re: Foreign Borrowing Act)," P.D. No. 1177, entitled "Revising the Budget Process in Order to Institutionalize the Budgetary Innovations of the New Society," P.D. No. 1967, entitled "An Act Strenghthening the Guarantee and Payment Positions of the Republic of the Philippines on Its Contingent Liabilities Arising out of Relent and Guaranteed Loan by Appropriating Funds For The Purpose.

Respondents contend that the petition involves a pure political question which is the repeal or amendment of said laws addressed to the judgment, wisdom and patriotism of the legislative body and not this Court. Similar case was Gonzales wherein the main issue was the constitutionality of the presidential veto in Sec16 of the General Appropriations Act of 1990. The court held that the political question doctrine neither interposes an obstacle to judicial determination of the rival claims. The jurisdiction to delimit constitutional boundaries has been given to this Court. It cannot abdicate that obligation mandated by the 1987 Constitution, although said provision by no means does away with the applicability of the principle in appropriate cases. Thus, Judicial power includes the duty of the courts of justice to settle actual controversies involving rights which are legally demandable and enforceable, and to determine whether or not there has been a grave abuse of discretion amounting to lack or excess of jurisdiction on the part of any branch or instrumentality of the Government. With the Senate maintaining that the President's veto is unconstitutional and that charge being controverted, there is an actual case or justiciable controversy between the Upper House of Congress and the executive department that may be taken cognizance of by this Court. ISSUE: I.. IS THE APPROPRIATION OF P86 BILLION IN THE P233 BILLION 1990 BUDGET VIOLATIVE OF SECTION 5, ARTICLE XIV OF THE CONSTITUTION? II. ARE PD No. 81, PD No. 1177 AND PD No. 1967 STILL OPERATIVE UNDER THE CONSTITUTION? III. ARE THEY VIOLATIVE OF SECTION 29(l), ARTICLE VI OF THE CONSTITUTION? 6

HELD: I. NO. It is true that Section 5(5), Article XIV of the Constitution mandated Congress to "assign the highest budgetary priority to education" in order to "insure that teaching will attract and retain its rightful share of the best available talents through adequate remuneration and other means of job satisfaction and fulfillment,". However, it does not deprive the Congress to respond to what is imperative that is, the national interest and attainment of state policies or objectives.

The 29 billion appropriation for Department of Education is the highest budgetary allocation among ALL department budget and is a clear compliance with the aforesaid constitutional mandate . Since 1985, the budget of education has tripled as well as the compensation of the teachers has been doubled. Now, Regarding the 89 Million Budget, Congress is certainly not without any power, guided only by its good judgment, to provide an appropriation, that can reasonably service our enormous debt, the greater portion of which was inherited from the previous administration. It is not only a matter of honor and to protect the credit standing of the country. More especially, the very survival of our economy is at stake. Thus, if in the process Congress appropriated an amount for debt service bigger than the share allocated to education, the Court finds and so holds that said appropriation cannot be thereby assailed as unconstitutional. II. YES. The long history would provide that the automatic budget appropriation given to the president has its roots with Former President Marcos. Until now, it is operative because of Section 3, Article XVIII of the Constitution recognizes that "All existing laws, decrees, executive orders, proclamations, letters of instructions and other executive issuances not inconsistent with the Constitution shall remain operative until amended, repealed or revoked.". An examination of the aforecited presidential decrees show the clear intent that the amounts needed to cover the payment of the principal and interest on all foreign loans, including those guaranteed by the national government, should be made available when they shall become due precisely without the necessity of periodic enactments of separate laws appropriating funds therefor, since both the periods and necessities are incapable of determination in advance. The automatic appropriation provides the flexibility for the effective execution of debt management policies. Clearly, the claim that payment of the loans and indebtedness is conditioned upon the continuance of the person of President Marcos and his legislative power goes against the intent and purpose of the law. The purpose is foreseen to subsist with or without the person of Marcos. III. NO. On the third issue that there is undue delegation of legislative power, the law must be complete in all its essential terms and conditions when it leaves the legislature so that there will be nothing left for the delegate to do when it reaches him except enforce it. If there are gaps in the law that will prevent its enforcement unless they are first filled, the delegate will then have been given the opportunity to step in the shoes of the legislature and exercise a discretion essentially legislative in order to repair the omissions. This is invalid delegation. 16 The Court finds that in this case the questioned laws are complete in all

their essential terms and conditions and sufficient standards are indicated therein. The legislative intention in R.A. No. 4860, as amended, Section 31 of P.D. No. 1177 and P.D. No. 1967 is that the amount needed should be automatically set aside in order to enable the Republic of the Philippines to pay the principal, interest, taxes and other normal banking charges on the loans, credits or indebtedness incurred as guaranteed by it when they shall become due without the need to enact a separate law appropriating funds therefor as the need arises. The purpose of these laws is to enable the government to make prompt payment and/or advances for all loans to protect and maintain the credit standing of the country. Although the subject presidential decrees do not state specific amounts to be paid, necessitated by the very nature of the problem being addressed, the amounts nevertheless are made certain by the legislative parameters provided in the decrees. The Executive is not of unlimited discretion as to the amounts to be disbursed for debt servicing. The mandate is to pay only the principal, interest, taxes and other normal banking charges on the loans, credits or indebtedness, or on the bonds, debentures or security or other evidences of indebtedness sold in international markets incurred by virtue of the law, as and when they shall become due. No uncertainty arises in executive implementation as the limit will be the exact amounts as shown by the books of the Treasury. Separate Opinions CRUZ, J., dissenting: I regret I must dissent. One of the essential requirements of a valid appropriation is that the amount appropriated must be certain, which means that the sum authorized to be released should either be determinate or at least determinable. As has been uniformly held: It is essential to the validity of an appropriation law that it should state the exact amount appropriated or the maximum sum from which the authorized expenses shall be paid, otherwise it would be void for uncertainty, since the legislative power over appropriation in effect could have been delegated in such case to the recipient of the funds appropriated or to the official authorized to spend them. (State v. Eggers, 16 L.R.A., N.S. 630; State v. La Grave, 41 Pac. 1075). The presidential decrees on which the respondents rely do not satisfy this

requirement. Even President Marcos as a legislator, did not know how much he was appropriating. The ponencia assures us that "no uncertainty arises in executive implementation as the limit will be the exact amounts as shown by the books of the Treasury." That is cold comfort, indeed, if we consider that it is the Treasury itself that is sought to be limited by the requirement for certainty. The intention precisely is to prevent the disbursement of public funds by the Treasury itself from "running riot." We surely cannot defend an appropriation, say, of "such amounts as may be necessary for the construction of a bridge across the Pasig River" even if the exact cost may be shown later by the books of the Treasury. This would be no different from the uncertain appropriations the Court is here sustaining. I think it is a mistake for this government to justify its acts on the basis of the decrees of President Marcos. These are on the whole tainted with authoritarianism and enfeebled by lack of proper study and draftmanship, let alone suspect motives. I suggest that these decrees must be reviewed carefully and whenever proper, set aright by necessary modification or outright revocation. Instead, the respondents are invoking them blindly. Sarmiento, J., concurs. GENERAL P RINCIPLES : NATURE
OF

Petitioner subsequently filed a motion with the RTC, manifesting the existence of an LRA Report which states that the subject lots were previously applied for registration and were both decided under Cadastral Case and, in this regard, moved that the aforesaid decision be set aside. RTC granted said motion andset aside the decision in the cadastral proceedings and reiterated that LRA issuethe corresponding decree of registration and certificate of title. The Republic filed a petition for certiorari (certiorari petition) before the CA. CA granted respondents certiorari petition and it ruled that petitioner should have raised in its application for registration the existence of a decision in Cad. Case No. 37 as it is required to prove its absolute ownership over the same and that no controversy regarding the matter of its ownership exists.Moreover, the CA pronounced that the RTCs Amended Order which set aside the decision in Cad. Case No. 37 was in utter disregard of the policy of judicial stability, stating further that only the CA can annul judgments of the RTC. Petitioner moved for reconsideration which was however denied. Hence, this petition. ISSUE: W/N the CA erred in annulling and setting aside the RTC Decision and Amended Order as well as the final decree of registration issued in favor of petitioner over the subject lots. HELD: No.

PHILIPPINE

COURTS

7. FIRST GAS POWER CORP v. RP REP. BY SOLGEN DOCTRINE OF JUDICIAL STABILITY

RATIO: In order that the petitioner for the registration of his land shall be permitted to have the same registered, and to have the benefit resulting from the certificate of title, finally, issued, the burden is upon him to show that he is the real and absolute owner, in fee simple. In this case, records disclose that petitioner itself manifested during the proceedings before the RTC that there subsists a decision in a previous cadastral casewhich covers the same lots it applied for registration. To note, the fact that the RTC did not order petitioner to address the matter or that it did not properly determine the effects of the existing decision to petitioners application does not justify the latters entitlement to have the subject lots registered in its name. Neither can the recommendation of the LRA to have the case set aside be perceived as an ample justification for the RTCs dispositions since this action is precluded by the doctrine of judicial stability. This doctrine states that the judgment of a court of competent jurisdiction may not be interfered

FIRST GAS POWER CORPORATION v. REPUBLIC FACTS: Petitioner sought for the original registration of two parcels of land situated in Batangas City. The RTC granted petitioners application for the registration of the subject lots. It found that petitioner was able to substantiate its bona fide claim of ownership over the subject lots thus it directed the registration of the subject lots in favor of petitioner and the issuance of the corresponding decree by the Land Registration Authority (LRA).

with by any court of concurrent jurisdiction.The rationale for the same is founded on the concept of jurisdiction verily, a court that acquires jurisdiction over the case and renders judgment therein has jurisdiction over its judgment, to the exclusion of all other coordinate courts, for its execution and over all its incidents, and to control, in furtherance of justice, the conduct of ministerial officers acting in connection with this judgment.Therefore, as the RTCs Amended Order was issued in stark contravention of this rule, the CA correctly ordered its nullification. 8. ALFEO D. VIVAS v. THE MONETARY BOARD HIERARCHY OF COURTS AND CERTIORARI AND PETITION FOR REVIEW FACTS: The Rural Bank of Faire, Incorporated (RBFI) was a duly registered rural bank in Cagayan. Its corporate life expired on May 31, 2005 however despite this Alfeo D. Vivas (Vivas) and his principals acquired the controlling interest of RBFI in January 2006. Vivas took certain measures to revitalize the bank. On December 8, 2006 BankoSentralngPilipinasextended the corporate life of RBFI for another 50 years. It also approved of the change of its corporate name to Euro Credit Community Bank (ECBI) as well as the increase in the number of its BoD from 5 to 11.

Pursuant to the New Central Bank Act, the integrated Supervision Department of the BSP conducted a general examination of ECBI. The examiners from the Department of Loans and Credit of the BSP arrived at the ECBI and cancelled the rediscounting line of the bank. Vivas appealed its cancellation. The Monetary arrived at the following findings during the general examination: 1. Negative capital of P14.674 Million and capital adequacy ratio of negative 18.42% 2. Capital Asset Management Earnings Liquidity composite of rating 2 with a Management component rating of 1 3. Serious supervisory concerns on activities deemed unsafe and unsound. The BSP, through a letter, directed the banks BOD to infuse fresh capital, to book the amount of unbooked valuation reserves and to take the appropriate action necessary to address the

violations. Vivas moved for reconsideration of the resolution on the grounds of non-observanve of due process and arbitrariness. There were several instances there the examiners had invited the BOD of ECIB to discuss the matters pertaining to the placement of the bank under the PCA. These meetings never materialized due to the postponements sought by Vivas. In two successive letters the BSP asked the ECBI to explain why it transferred the majority shares of RFBI without securing the prior approval of the MB and to explain why it did not obtain the prior approval of the BSP when it established the banks sub offices. There was a scheduled general examination in 2009, however this did not push through because ECIB asked for the deferment while its appeal from the resolution was still pending. The MB posited that ECBI unjustly and constantly refused to allow the BSP examiners from examining and inspecting its books in violation of R.A. No. 7653 and therefore the MB issued another resolution (Resolution No. 726) which imposed a monetary fine on ECBI. It also referred the matter to the Office of the Special Investigation. (OSI) for the filing of appropriate legal action. A general examination took place and thereafter, the MB issued the resolution approving the issuance of a cease and desist order against ECBI which enjoined it from pursuing certain acts and transactions that were considered unsafe and unsound banking practices. The OSI filed with the DOJ a complaint for Estafa Though Falsification of Commercial Documents against certain officials and employees of the ECBI and the MB denied the appeal of the ECIB from the resolution which placed it under the PCA framework. In 2010, the MB issued a resolution placing the ECBI under receivership. The resolution claimed that the basis of such were the findings that showed that ECBI was unable to pay its liabilities as they become due, that it had insufficient assets to meet those liabilities, that it cannot continue in business without involving losses and theat ECBI has willfully violated a cease and desist order of the MB for acts or transactions which are considered unsafe and unsound banking practices. The MB designated Philippine Deposit Insurance Corp as the receiver of the bank. ECBI then filed this petition for prohibition with a prayer for the issuance oa a status quo ante order or writ of preliminary injunction ordering the respondents to desist from closing ECBI and from pursuing the receivership thereof. ECBI argues that the

MBs acts amounted to grave abuse of discretion amounting to loss of jurisdiction by applying the general law embodied in the New Central Bank Act rather than applying the specific law embodied in Sec 11 and 14 of Rural Banks Act of 1992. And even assuming that the New Central bank act is applicable it is still gave abuse to execute the law with manifest arbitrariness, abuse of discretion, bad faith and violation of constitutional rights. And Lastly, it argues that the power delegated in favor of the BangkoSentralngPilipinas to place rural banks under receivership is unconstitutional for being a diminution or invasion of the powers of the SC, in violation of Sec 2, Article VIII of the Phil Constitution. ISSUE: Procedural Issues (Relevant issues): 1. W/N the filing of the Writ of Prohibition was proper 2. W/N the venue of the action was proper. HELD: 1. NO. Vivas should have filed a petition for certiorari 2. NO. The petition should have been filed with the CA THE PETITION MUST FAIL. Petition for prohibition is DENIED. RATIO: Firstly, Vivas availed of the wrong remedy. The MB issued a resolution which states that any act of the MB placing a bank under conservatorship, receivership or liquidation may not be restrained or set aside except for petition for CERITORARI. Therefore instead of filing for prohibition he should have filed a petition for certiorari. Granting however, that a petition for prohibition is allowed, it is already an ineffective remedy under the circumstances. Prohibition is that process by which a superior court prevents inferior courts, tribunals, officers, or persons from usurping or exercising a jurisdiction with which they have not been vested by law and confines them to the exercise of those powers legally conferred. The rule on prohibition is enshrined in Sec 2, Rule 65 of the Rules on Civil Procedure: Sec. 2. Petition for prohibition - When the proceedings of any tribunal, corporation, board, officer or person, whether exercising judicial, quasijudicial or ministerial functions, are without or in excess of its or his jurisdiction, or with grave abuse of discretion amounting to lack or excess of jurisdiction, and there is no appealor any other plain, speedy, and adequate remedy in the ordinary course of law, a person aggrieved thereby may file a verified petition in the proper court, alleging the facts with certainty and praying that the judgment be rendered commanding the respondent to desist from further proceedings in the action or matter specified therein, or otherwise granting such incidental reliefs as the law and justice require. x xxx.

As a rule the proper function of prohibition is to prevent the doing of an act which is about to be done. It is not intended to provide a remedy for acts already accomplished. In the case at bar, this petition for prohibition apparently seeks to prevent the acts of closing of ECBI and placing it under receivership. The resolution of the MB, however, had already been issued and therefore the bank has already been places under receivership. Therefore the remedy of prohibition is no longer appropriate. Secondly, The petition should have been filed in the CA. Section 4 of Rule 65 reads: Section 4. When and where petition filed. The petition shall be filed not later than sixty (60) days from notice of the judgment, order or resolution. In case a motion for reconsideration or new trial is timely filed, whether such motion is required or not, the sixty (60) day period shall be counted from notice of the denial of said motion. The petition shall be filed in the Supreme Court or, if it relates to the acts or omissions of a lower court or of a corporation, board, officer or person, in the Regional Trial Court exercising jurisdiction over the territorial area as defined by the Supreme Court. It may also be filed in the Court of Appeals whether or not the same is in aid of its appellate jurisdiction, or in the Sandiganbayan if it is inaid of its appellate jurisdiction. If it involves the acts or omissions of a quasi-judicial agency, unless otherwise provided by law or these Rules, the petition shall be filed in and cognizable only by the Court of Appeals. [Emphases supplied] The fact that the MB is a quasi-judicial agency is already a settled matter as seen in the case of Bank of Commerce v. Planters Development Bank and BankoSentral Ng Pilipinas. Thirdly, petitioner should have observed the hierarchy of courts. Even in the absence of such provision, the petition is also dismissible because it simply ignored the doctrine of hierarchy of courts. True, the Court, the CA and the RTC have original concurrent jurisdiction to issue writs of certiorari, prohibition and mandamus. The concurrence of jurisdiction, however, does not grant the party seeking any of the extraordinary writs the absolute freedom to file a petition in any court of his choice. The petitioner has not advanced any special or important reason which would allow a direct resort to this Court. Under the Rules of Court, a party may directly appeal to this Court only on pure questions of law. In the case before us, there are certainly factual issues as Vivas is questioning the findings of the investigating team. Strict observance of the policy of judicial hierarchy demands that where the issuance of the extraordinary writs is also within the competence of the CA or the RTC, the special action for the obtainment of such writ must be presented to either court. As a rule, the Court will not entertain direct resort to it

GENERAL P RINCIPLES : J URISDICTION 9. San Miguel Properties v. Sec. Hernando B. Perez DOCTRINE OF PRIMARY JURISDICTION FACTS: San Miguel Properties purchased 130 residential lots from BF Homes (represented by Atty. Orendain who was appointed rehabilitation receiver by the SEC.) for Php106,248,000.00. The TCTs covering the lots bought under the first 2 deeds were delivered to San Miguel but 20 TCTs covering 20 of the 41 parcels of land purchased under the third deed of sale were not delivered. BF Homes claimed that it withheld the delivery of the 20 TCTs because Atty. Orendain had ceased to be its rehabilitation receiver at the time of the transactions after being replaced as receiver by FBO Network Management. BF Homes refused to deliver despite demands. San Miguel filed a complaint-affidavit in the Office of the City Prosecutor of Las Pias City (OCP Las Pias) charging respondent directors and officers of BF Homes with non-delivery of titles in violation of PD 957. San Miguel sued BF Homes for specific performance in the HLURB praying to compel BF Homes to release the 20 TCTs in its favor. San Miguel filed a motion to suspend proceedings in the OCP Las Pias, citing the pendency of BF Homes receivership case in the SEC. OCP Las Pias dismissed San Miguels criminal complaint for violation of PD 957 on the ground that the implementation of PD 957 exclusively pertained under the jurisdiction of the HLURB; that there existed a prejudicial question necessitating the suspension of the criminal action until after the issue on the liability of the BF Homes was first determined by the SEC en banc or by the HLURB; and that no prior resort to administrative jurisdiction had been made. There is no dispute that aside from the instant complaint for violation of PD 957, there is still pending with the HLURB a complaint for specific performance where the HLURB is called upon to inquire into, and rule on, the validity of the sales

transactions involving the lots in question and entered into by Atty. Orendain for and in behalf of BF Homes. The HLURB Arbiter ruled that the HLURB was inclined to suspend the proceedings until the SEC resolved the issue of Atty. Orendains authority to enter into the transactions in BF Homes behalf, because the final resolution by the SEC was a logical antecedent to the determination of the issue involved in the complaint before the HLURB.

ISSUE: Whether or not the HLURB had the authority in suspending the proceedings. HELD: Yes. The said ruling simply means that unless and until the HLURB rules on the validity of the transactions involving the lands in question with specific reference to the capacity of Atty. Orendain to bind BF Homes in the said transactions, there is as yet no basis to charge criminally respondents for non-delivery of the subject land titles. In other words, complainant cannot invoke the penal provision of PD 957 until such time that the HLURB shall have ruled and decided on the validity of the transactions involving the lots in question. RATIO: The action for specific performance, although civil in nature, could be brought only in the HLURB. This situation conforms to the doctrine of primary jurisdiction. It is in favor of these agencies that the doctrine of primary jurisdiction is frequently invoked, not to defeat the resort to the judicial adjudication of controversies but to rely on the expertise, specialized skills, and knowledge of such agencies in their resolution. A case that requires for its determination the expertise, specialized skills, and knowledge of some administrative board or commission because it involves technical matters or intricate questions of fact, relief must first be obtained in an appropriate administrative proceeding before a remedy will be supplied by the courts although the matter comes within the jurisdiction of the courts. The application of the doctrine does not call for the dismissal of the case in the court but only for its suspension until after the matters within the competence of the administrative body are threshed out and determined. 10. Addition Hills Mandaluyong Civic & Social Organization vs Megaworld Properties & Holdings Inc DOCTRINE OF EXHAUSTION OF ADMINISTRATIVE REMEDIES AND; DOCTRINE OF PRIMARY JURISDICTION; RULE 8 FACTS MEGAWORLD was the registered owner of a parcel of land located in Barangay Addition Hills, Mandaluyong City. MEGAWORLD conceptualized the construction of a residential

condominium complex onsaid parcel of land called the WackWack Heights Condominium. MEGAWORLD thereafter secured the necessary clearances, licenses and permits for the project. The plaintiff AHMCSO filed a complaint to annul the Building and Development Permits; and to permanently enjoinbuilding officials from issuing licenses and permits to MEGAWORLD. Thus the trial court declared the certificates and licenses issued as void and of no effect. MEGAWORLD is directed to rectify its Wack Wack Heights Project for it to conform to the requirements of an R-2 zone of Mandaluyong City and of the Metro Manila Zoning Ordinance. This RTC ruling was REVERSED by the CA on appeal. The CA found that the petitioners failed to exhaust administrative remedies before seeking redress from the courts.

filed before the Housing and Land Use Arbiter (HLA).The decision of the HLA may be brought to the Board of Commissioners by Petition for Certiorari and the decision of the Board of Commissioners [is] appealable to the Office of the President.

11. PILAR DEVELOPMENT CORPORATION vs. SPOUSES VILLAR JURISDICTION OF TRIAL COURTS V. HLURB FACTS: An ejectment suit originating from the MeTC of Las Pias City, was decided in favor of herein petitioner Pilar Development Corporation (PDC). However, on appeal, the RTC of Las Pias City reversed and set aside that of the MeTC and ordered the dismissal of the case, allegedly for want of jurisdiction thereon on the part of the MeTC. The RTC held that it is the HLURB, not the regular courts, which has jurisdiction over the suit. Directly elevating the issue to this Court on pure question of law, this petition for review on certiorari seeks the reversal of the RTC decision and the reinstatement of that of the MeTC. A Contract to Sell was executed between the petitioner and the respondents whereby the former sold to the latter a house and lot located at B.F. Resort Village Subdivision for P960K payable on installment with a downpayment of P289Kand the balance of P672K in 120 monthly amortizations at P13K/month. The TCT was issued in the name of the petition only after the execution of the subject contract and the consolidation and re-subdivision of a number of parcels of land enumerated in the contract. Respondents paid the required downpayment and some monthly amortizations after which they defaulted in the payment of the succeeding monthly amortizations. The petitionercancelled the subject contract thru a Notice of Cancellation. The petitioner, however, did not refund the cash surrender value to the respondents.Despite demands to vacate, the respondents still refused to surrender possession of subject premises to the petitioner. In their Answer, the respondents primarily assailed the jurisdiction of the court a quo over the subject matter and the propriety of the cancellation of the subject contract. Further, the respondents put in issue the identity of the property covered by the TCT, alleging that there was no showing that the residential lot stated therein subject of the complaint is similar to that provided in the contract. ISSUE: W/N it is the HLURB or the regular courts that has jurisdiction over the subject matter of the case

ISSUE W/N the complaint should be dismissed because the petitioners failed to exhaust administrative remedies before seeking judicial intervention from the courts. YES HELD The courts must allow administrative agencies to carry out their functions and discharge their responsibilities within the specialized areas of their respective competence RATIO The general rule is that before a party may seek the intervention of the court, he should first avail of all the means afforded him by administrative processes. The issues which administrative agencies are authorized to decide should not besubmitted to a court without first giving such administrative agency the opportunity to dispose of the same after due deliberation. Corollary to the doctrine of exhaustion of administrative remedies is the doctrine of primary jurisdiction; that is, courts cannot or will not determine a controversy involving a question which is within the jurisdiction of the administrative tribunal prior to the resolution of that question by the administrative tribunal, where the question demands the exercise of sound administrative discretion requiring the special knowledge, experience and services of the administrative tribunal to determine technical and intricate matters of fact. The petitioner unjustifiably failed to exhaust the administrative remedies available with the Housing and Land Use Regulatory Board (HLURB) before seeking recourse with the trial court. Under the rules of the HLURB, a complaint to annul any permit issued by the HLURB may be

HELD: The METC has JURISDICTION over the case. RATIO The RTC cited Presidential Decree (P.D.) No. 1344, which defines the jurisdiction of the HLURB (formerly National Housing Authority), as follows: Section 1. In the exercise of its functions to regulate the real estate trade and business and in addition to its powers provided for in Presidential Decree No. 957, the National Housing Authority shall have exclusive jurisdiction to hear and decide cases of the following nature: (a) Unsound real estate business practices; (b) Claims involving refund and any other claims filed by subdivision lot or condominium unit buyer against the project owner, developer, dealer, broker or salesman; and (c) Cases involving specific performance of contractual and statutory obligations filed by buyers of subdivision lot or condominium unit against the owner, developer, dealer, broker or salesman. In the case at bar, it was filed by the subdivision owner and not the buyer of a subdivision lot, and the cause of action is one for recovery of possession of the property on account of the cancellation of the parties contract to sell for nonpayment by the respondent spouses of the monthly amortizations pursuant to the terms and conditions stated in their written contract. The respondent spouses, as buyers of the subdivision lot in question, had no cause of action against petitioner PDC as subdivision owner. No jurisdiction could, therefore, be possibly vested upon the HLURB. Jurisdiction on the legal issue involving the right of possession over the subject lot rightfully belongs to the regular courts, in this case the MeTC of Las Pias City. The MeTC correctly ruled that petitioner PDC has the right to possess the subject property upon the effectivity of the cancellation of the contract to sell, pursuant to the terms and conditions specified therein, insofar as those terms and conditions are not contrary to the pertinent provisions of Republic Act (R.A.) No. 6552, otherwise known as Realty Installme nt Buyer Act. According to R.A. 6552, the cash surrender value, which in this case is equivalent to 50% of the total payment made by the respondent spouses, should be returned to them by the petitioner upon the cancellation of the contract to sell for the cancellation to take

effect. Admittedly, no such return was ever made by the petitioner. Thus, the said cash surrender value is hereby ordered deducted from the award owing to the petitioner based on the MeTC judgment, and cancellation takes effect by virtue of this judgment.

12. RIDGEWOOD ESTATE, INC. (Erroneously sued as Camella Homes) v. EXPEDITO BELAOS JURISDICTION OF TRIAL COURTS V. HLURB FACTS: Respondent Belaos entered into a contract to sell with Petitioner Subdivision Developer for the purchase of a house and lot. Pursuant thereto, Respondent issued several postdated checks in favor of petitioner as amortization for the property. Petitioner, however, failed to construct the house. Thus, respondent rescinded the contract and demanded the return of the amounts he had paid to petitioner, as well as the postdated checks. Petitioner remitted to respondent a portion of the amount of checks withdrawn BUT nonetheless continued to encash the other postdated checks, to the prejudice of respondent. Respondent filed before the RTC a complaint for damages against Camella Homes.

ISSUE: Petitioner filed a petition for certiorari before the Court of Appeals, raising the argument that the trial court had no jurisdiction over the suit, as the subject matter of the complaint was within the exclusive jurisdiction of the Housing and Land Use Regulatory Board (HLURB). Correct? HELD: The trial court correctly assumed jurisdiction over the complaint filed by respondent against petitioner. RATIO: The Court held in Roxas v. Court of Appeals that the mere relationship between the parties, i.e., that of being subdivision owner/developer and subdivision lot buyer, does not automatically vest jurisdiction in the HLURB. For an action to fall within the exclusive jurisdiction of the HLURB, the decisive element is the nature of the action as enumerated in Section 1 of P.D. No. 1344. The HLURB has jurisdiction over complaints aimed at compelling the subdivision developer to comply with its contractual and statutory obligations. Section 1 of Presidential Decree No. 1344 provides for the jurisdiction of HLURB (then National Housing Authority), thus:

Sec. 1. In the exercise of its function to regulate the real estate trade and business and in addition to its powers provided for in Presidential Decree No. 957, the National Housing Authority shall have exclusive jurisdiction to hear and decide the cases of the following nature: a. b. Unsound real estate business practices; Claims involving refund and any other claims filed by subdivision lot or condominium unit buyer against the project owner, developer, dealer, broker or salesman; and Cases involving specific performance of contractual and statutory obligations filed by buyers of subdivision lot or condominium unit against the owner, developer, dealer, broker or salesman.

c.

The complaint filed by respondent against petitioner was one for damages. It prayed for the payment of moral, actual and exemplary damages by reason of petitioners malicious encashment of the checks even after the rescission of the contract to sell between them. Respondent claimed that because of petitioners malicious and fraudulent acts, he suffered humiliation and embarrassment in several banks, causing him to lose his credibility and good standing among his colleagues. Such action falls within the jurisdiction of regular courts, not the HLURB. 13. ORTIGAS & COMPANY, LIMITED PARTNERSHIP v. CA, HON. JESUS G. BERSAMIRA, ET AL. JURISDICTION OF TRIAL COURTS V. HLURB FACTS Petitioner Ortigas & Company, Limited Partnership (Ortigas), a realty company, developed the Ortigas Center that straddled the three cities of Mandaluyong, Quezon, and Pasig. This case concerns the Pasig City side of the commercial district known as the Ortigas Center, formerly known as Capitol VI Subdivision. In 1994 respondent City of Pasig (the City) filed a complaint against Ortigas and Greenhills Properties, Inc. (GPI) for specific compliance before the Regional Trial Court (RTC) of Pasig. The City alleged that Ortigas failed to comply with Municipal Ordinance 5 (MO 5) which required it to designate appropriate recreational and playground facilities at its former Capitol VI Subdivision (regarded as a residential site), now the Pasig City side of the Ortigas Center. Further, the City alleged that despite the fact that the plan

was only approved by the Municipal Council as to layout, petitioner proceeded to develop the property without securing a final approval. The City impleaded GPI as the party to whom Ortigas sold a piece of property within the subdivision. In answer, Ortigas alleged that its development plan for the subject land was for a commercial subdivision, outside the scope of MO 5 that applied only to residential subdivision. Ortigas further alleged that only in 1984, 15 years after the approval of its plan, that the National Housing Regulatory Commission imposed the open space requirement for commercial subdivisions through its Rules and Regulations for Commercial Subdivision and Commercial Subdivision Development. The case was heard on pre-trial but before it could be terminated, Ortigas filed a motion to dismiss the case on the ground that the RTC had no jurisdiction over it, such jurisdiction being in the Housing and Land Use Regulatory Board (HLURB) for unsound real estate business practices. The RTC denied the motion to dismiss. With the denial of its motion for reconsideration, Ortigas filed a petition for certiorari before the Court of Appeals (CA) to challenge the RTCs actions. The CA rendered judgment, affirming the RTCs denial of the motion to dismiss. Since the City was not a buyer or one entitled to refund for the price paid for a lot, the dispute must fall under the jurisdiction of the RTC pursuant to Section 19 of The Judiciary Reorganization Act of 1980. The CA denied Ortigas motion for reconsideration, prompting it to file the present petition for review.

ISSUE W/N the CA erred in affirming the lower courts ruling that jurisdiction over the Citys action lies with the RTC, not with the HLURB HELD NO RATIO Ortigas maintains that the HLURB has jurisdiction over the complaint since a land developer's failure to comply with its statutory obligation to provide open spaces constitutes unsound real estate business practice that Presidential Decree (P.D.) 1344 prohibits. Executive Order 648 empowers the HLURB to hear and decide claims of unsound real estate business practices against land developers. Ultimately, whether or not the HLURB has the authority to hear and decide a case is determined by the nature of the cause of action, the subject matter or property involved, and the parties. Section 1 of P.D. 1344 vests in the HLURB the exclusive jurisdiction to hear and decide the following cases: (a) unsound real estate business practices;

(b) claims involving refund and any other claims filed by subdivision lot or condominium unit buyer against the project owner, developer, dealer, broker, or salesman; and (c) cases involving specific performance of contractual and statutory obligations filed by buyers of subdivision lots or condominium units against the owner, developer, dealer, broker or salesman. Unlike paragraphs (b) and (c) above, paragraph (a) does not state which party can file a claim against an unsound real estate business practice. But, in the context of the evident objective of Section 1, it is implicit that the unsound real estate business practice would, like the offended party in paragraphs (b) and (c), be the buyers of lands involved in development. The policy of the law is to curb unscrupulous practices in real estate trade and business that prejudice buyers. This position is supported by the Courts statement in Delos Santos v. Sarmiento that not every case involving buyers and sellers of subdivision lots or condominium units can be filed with the HLURB. Its jurisdiction is limited to those cases filed by the buyer or owner of a subdivision lot or condominium unit and based on any of the causes of action enumerated in Section 1 of P.D. 1344. Obviously, the City had not bought a lot in the subject area from Ortigas which would give it a right to seek HLURB intervention in enforcing a local ordinance that regulates the use of private land within its jurisdiction in the interest of the general welfare. It has the right to bring such kind of action but only before a court of general jurisdiction such as the RTC.

facilities at its former Capitol VI Subdivision (regarded as a residential site), now the Pasig City side of the Ortigas Center. Further, the City alleged that despite the fact that the plan was only approved by the Municipal Council as to layout, petitioner proceeded to develop the property without securing a final approval. The City impleaded GPI as the party to whom Ortigas sold a piece of property within the subdivision. In answer, Ortigas alleged that its development plan for the subject land was for a commercial subdivision, outside the scope of MO 5 that applied only to residential subdivision. Ortigas further alleged that only in 1984, 15 years after the approval of its plan, that the National Housing Regulatory Commission imposed the open space requirement for commercial subdivisions through its Rules and Regulations for Commercial Subdivision and Commercial Subdivision Development. The case was heard on pre-trial but before it could be terminated, Ortigas filed a motion to dismiss the case on the ground that the RTC had no jurisdiction over it, such jurisdiction being in the Housing and Land Use Regulatory Board (HLURB) for unsound real estate business practices. The RTC denied the motion to dismiss. With the denial of its motion for reconsideration, Ortigas filed a petition for certiorari before the Court of Appeals (CA) to challenge the RTCs actions. The CA rendered judgment, affirming the RTCs denial of the motion to dismiss. Since the City was not a buyer or one entitled to refund for the price paid for a lot, the dispute must fall under the jurisdiction of the RTC pursuant to Section 19 of The Judiciary Reorganization Act of 1980. The CA denied Ortigas motion for reconsideration, prompting it to file the present petition for review.

14. ORTIGAS & COMPANY, LIMITED PARTNERSHIP v. CA, HON. JESUS G. BERSAMIRA, ET AL. JURISDICTION OF TRIAL COURTS V. HLURB FACTS Petitioner Ortigas & Company, Limited Partnership (Ortigas), a realty company, developed the Ortigas Center that straddled the three cities of Mandaluyong, Quezon, and Pasig. This case concerns the Pasig City side of the commercial district known as the Ortigas Center, formerly known as Capitol VI Subdivision. In 1994 respondent City of Pasig (the City) filed a complaint against Ortigas and Greenhills Properties, Inc. (GPI) for specific compliance before the Regional Trial Court (RTC) of Pasig. The City alleged that Ortigas failed to comply with Municipal Ordinance 5 (MO 5) which required it to designate appropriate recreational and playground

ISSUE W/N the CA erred in affirming the lower courts ruling that jurisdiction over the Citys action lies with the RTC, not with the HLURB HELD NO RATIO Ortigas maintains that the HLURB has jurisdiction over the complaint since a land developer's failure to comply with its statutory obligation to provide open spaces constitutes unsound real estate business practice that Presidential Decree (P.D.) 1344 prohibits. Executive Order 648 empowers the HLURB to hear and decide claims of unsound real estate business practices against land developers. Ultimately, whether or not the HLURB has the authority to hear and decide a case is determined by the nature of the cause of action, the subject matter or property involved, and the parties. Section 1 of P.D. 1344 vests in the HLURB the exclusive jurisdiction to hear and decide the following cases:

(a)

unsound real estate business practices;

(b) claims involving refund and any other claims filed by subdivision lot or condominium unit buyer against the project owner, developer, dealer, broker, or salesman; and (c) cases involving specific performance of contractual and statutory obligations filed by buyers of subdivision lots or condominium units against the owner, developer, dealer, broker or salesman. Unlike paragraphs (b) and (c) above, paragraph (a) does not state which party can file a claim against an unsound real estate business practice. But, in the context of the evident objective of Section 1, it is implicit that the unsound real estate business practice would, like the offended party in paragraphs (b) and (c), be the buyers of lands involved in development. The policy of the law is to curb unscrupulous practices in real estate trade and business that prejudice buyers. This position is supported by the Courts statement in Delos Santos v. Sarmiento that not every case involving buyers and sellers of subdivision lots or condominium units can be filed with the HLURB. Its jurisdiction is limited to those cases filed by the buyer or owner of a subdivision lot or condominium unit and based on any of the causes of action enumerated in Section 1 of P.D. 1344. Obviously, the City had not bought a lot in the subject area from Ortigas which would give it a right to seek HLURB intervention in enforcing a local ordisnance that regulates the use of private land within its jurisdiction in the interest of the general welfare. It has the right to bring such kind of action but only before a court of general jurisdiction such as the RTC.

the Baguio Dairy Farm, a government reservation under the supervision of the Department of Agriculture (DA). On June 29, 2003, respondents filed a petition for injunction (with prayer for the issuance of a temporary restraining order [TRO] and/or a writ of preliminary injunction) with the NCIP seeking to enjoin the mayor of Baguio City and the head of the citys demolition team from implementing Demolition Order No. 17, series of 2003. The order, issued by the office of the mayor upon complaint of the DA, directs the demolition of shanties and other structures within the premises of the Baguio Dairy Farm belonging to private respondents which were then undergoing construction or were recently built without the required permits. In response, public respondent Brain Masweng, regional hearing officer of the NCIP in the Cordillera Administrative Region, issued a 72-hour TRO. In a resolution dated July 21, 2003, respondent Masweng granted the application for a writ of preliminary injunction. The case was elevated to the Court of Appeals (CA) via a petition for certiorari filed by petitioners. Petitioners contend that injunction, as an original and principal action, falls within the jurisdiction of the regular courts. The NCIP may issue TROs and writs of preliminary injunction only as an auxiliary remedy to a pending case before it. Petitioners also assert that there was no factual and legal basis for the NCIPs issuance of a writ of preliminary injunction.

ISSUE/S: 1. Whether or not the issuance by the NCIP of the writ of preliminary injunction was valid. HELD: 1. NO. RATIO: 1. The court held that the NCIP may issue temporary restraining orders and writs of injunction without any prohibition against the issuance of the writ when the main action is for injunction. The power to issue temporary restraining orders or writs of injunction allows parties to a dispute over which the NCIP has jurisdiction to seek relief against any action which may cause them grave or irreparable damage or injury. Private respondents base their claim to the disputed area on an alleged time-immemorial possession and a survey plan awarded to their forebears by the Director of Lands in 1920. In 1940, Proclamation No. 603 withdrew the contested area from sale or settlement and reserved the same for animal breeding station

15. THE CITY MAYOR OF BAGUIO, ET.AL., V. ATTY. BRAIN MASWENG, ETC.ET.AL. JURISDICTION OF COURTS V. NCIP FACTS: Respondents Judith K. Cario, Jacqueline Cario and the other heirs of Mateo Cario and Bayosa Ortega are members of the Ibaloi tribe Their ancestors were grantees of a survey plan approved by the Director of Lands in 1920. Currently, they have pending petitions before the NCIP for the validation of ancestral land claims covering a parcel of land in Resident Section J in Baguio City and Tuba, Benguet. A portion of the land being claimed by petitioners overlaps with

purposes, subject to private rights. The claim of respondents on the subject land is still pending before the NCIP. Thus, their rights are mere expectations, not the present and unmistakable right required for the grant of the provisional remedy of injunction. The structures subject of the demolition order were either built or being constructed without the requisite permit at the time the demolition order was issued in 2003. Hence, private respondents were not entitled to the preliminary injunction issued by the NCIP.

Whether or not an RTC, acting as Special Agrarian Court, has jurisdiction over just compensation cases involving agricultural lands located outside its regular jurisdiction but within the province where it is designated as an agrarian court under the Comprehensive Agrarian Reform Law of 1998? HELD: Yes. RULING: The RTC, Branch 32 based its order on Deputy Court Administrator (DCA) Zenaida Elepaos opinion that single sala courts have jurisdiction over agrarian cases involving lands located within its territorial jurisdiction. An RTC branch acting as a special agrarian court, she claimed, did not have expanded territorial jurisdiction. Being a single sala court, the Regional Trial Court, Branch 64, Guihulngan, Negros Oriental, has jurisdiction over all cases, including agrarian cases, cognizable by the Regional Trial Court emanating from the geographical areas within its territorial jurisdiction. The law that confers jurisdiction on Special Agrarian Courts designated by the Supreme Court in every province is R.A. 6657 or the CARL. R.A. 6657 requires the designation by the Supreme Court before an RTC Branch can function as a Special Agrarian Court. The Supreme Court has not designated the single sala courts of RTC, Branch 64 of Guihulngan City and RTC, Branch 63 of Bayawan City as Special Agrarian Courts. Consequently, they cannot hear just compensation cases just because the lands subject of such cases happen to be within their territorial jurisdiction. Since RTC, Branch 32 of Dumaguete City is the designated Special Agrarian Court for the province of Negros Oriental, it has jurisdiction over all cases for determination of just compensation involving agricultural lands within that province, regardless of whether or not those properties are outside its regular territorial jurisdiction.

16. HEIRS OF CANDIDO DEL ROSARIO v. MONICA DEL ROSARIO


(PENDING) SEE FOLDER 5 JURISDICTION OF COURTS V. DARAB 17. LANDBANK OF THE PHILIPPINES v. CORAZON M. VILLEGAS and LANDBANK OF THE PHILIPPINES V. HEIRS OF CATALINO NOEL AND PROCULA SY COURTS OF GENERAL AND SPECIAL JURISDICTION- JURISDICTION OF SPECIAL AGRARIAN COURT FACTS: Land Bank filed cases for determination of just compensation against Corazon Villegas in Civil Case 2007-14174 and heirs of Catalino Noel and Procula Sy in Civil Case 2007-14193 before the RTC of Dumaguete City, Branch 32, sitting as a Special Agrarian Court for the province of Negros Oriental. Villegas property was in Hibaiyo, Guihulngan City, Negros Oriental, while respondent heirs land was in Nangca, Bayawan City, Negros Oriental. These lands happened to be outside the regular territorial jurisdiction of RTC Branch 32 of Dumaguete City. RTC, Branch 32 dismissed Civil Case 2007-14174 for lack of jurisdiction. It ruled that, although it had been designated Special Agrarian Court for Negros Oriental, the designation did not expand its territorial jurisdiction to hear agrarian cases under the territorial jurisdiction of the RTC, Branch 64 of Guihulngan City where respondent Villegas property can be found. RTC, Branch 32 also dismissed Civil Case 2007-14193 for lack of jurisdiction. It pointed out that RTC, Branch 63 of Bayawan City had jurisdiction over the case since respondent heirs property was within the latter courts territorial jurisdiction. ISSUE:

18. IRENE SANTE v. HONORABLE CLARAVALL JURISDICTION OF RTC v. MTC over damages FACTS: Respondent Vita Kalashian filed before the RTC of Baguio City a complaint for damages against Petitioner for the latters remarks against her in the presence of police officers and other persons while they were inside the Police Station of Natividad, Pangasinan. Respondent prayed that Petitioner be held liable to pay moral damages in the amount of P 300,000; P50,000 as exemplary damages;P50,000 attorneys fees; P20,000 litigation expenses; and costs of suit.

Petitioner then filed a Motion to Dismiss on the ground that it was the MTCC not the RTC which has jurisdiction over the case. She argued that the amount of the claim for moral damages was not more than the jurisdictional amount of P300,000, because the claim for exemplary damages should be excluded in computing the total claim. The RTC of Baguio denied the Motion to Dismiss alleging that the total claim of respondent amounted to P420,000 which was above the jurisdictional amount for MTCCs outside Metro Manila. Aggrieved, the Petitioner filed a Petition for Certiorari and Prohibition docketed as CA-G.R. SP No. 85465, before the Court of Appeals. Meanwhile, the Respondent filed an amended complaint increasing the claim for moral damages to P1,000,000. This action was the subject of another Motion to Dismiss, which was denied by the RTC, and another Petition for Certiorari and Prohibition docketed as CA-G.R. SP No. 87563. In CA-G.R. SP No. 85465, the CA ruled that the case is clearly one that falls under the MTCC since the amount to be considered in determining jurisdiction is only the claim for moral damages with the other claims merely as incidental. On the other hand, CA-G.R. SP No. 87563 was decided in the opposite when it held that the total or aggregate amount demanded in the complaint constitutes the basis of jurisdiction thus leading to the validity of the amendment of the claim for moral damages. ISSUE: Did the RTC acquire jurisdiction over the case? HELD: Yes RULING: It has been provided in Sec. 19 (8) of B.P. Blg. 129, as amended by R.A. No. 7691, that Regional Trial Courts shall exercise exclusive original jurisdiction, in all other cases in which the demand, exclusive of interest, damages of whatever kind, attorneys fees, litigation expenses, and costs or the value of the property in controversy exceeds Three hundred thousand pesos (P300,000) or, in such other cases in Metro Manila, where the demand, exclusive of the abovementioned items exceeds Four hundred thousand pesos (P400,000). But where damages is the main cause of action, should the amount of moral damages prayed for in the complaint be the sole basis for determining which court has jurisdiction or should the total amount of all the damages claimed regardless of kind and nature, such as exemplary damages, nominal damages, and attorneys fees,etc., be used? In this regard, Administrative Circular No. 09-94 is instructive, 2. The exclusion of the term damages of whatever kind in determining the jurisdictional amount under Section 19 (8) and Section 33 (1) of B.P. Blg. 129, as amended by R.A. No. 7691, applies to cases where the damages are merely incidental to or a consequence of the main cause of action. However, in cases where the claim for

damages is the main cause of action, or one of the causes of action, the amount of such claim shall be considered in determining the jurisdiction of the court. In the instant case, the complaint filed is for the recovery of damages for the alleged malicious acts of petitioners. The complaint principally sought an award of moral and exemplary damages, as well as attorneys fees and litigation expenses, for the alleged shame and injury suffered by respondent by reason of petitioners utterance while they were at the police station. It is settled that jurisdiction is conferred by law based on the facts alleged in the complaint since the latter comprises a concise statement of the ultimate facts constituting the plaintiffs causes of action. It is clear based on the allegations of the complaint that respondents main action is for damages. Hence, the other forms of damages being claimed by respondent, e.g., exemplary damages, attorneys fees and litigation expenses, are not merely incidental to or consequences of the main action but constitute the primary relief prayed for in the complaint. Thus, the aggregate amount of the damages prayed for shall determine jurisdiction. Considering that the total amount of damages claimed was P420,000, the Court of Appeals was correct in ruling that the RTC had jurisdiction over the case. By having acquired jurisdiction over the original complaint, the RTC committed no grave abuse of discretion in allowing an amendment of the amount of moral damages claimed since the amendment in this case was a matter of right.

19. HEIRS OF GENEROSO SEBE v. HEIRS OF VERONICO SEVILLA G.R. No. 174497 | October 12, 2009 JURISDICTION OVER THE SUBJECT MATTER Facts: Plaintiff spouses Generoso and Aurelia Sebe and their daughter, Lydia Sebe, (the Sebes) filed with the RTC of Dipolog City a complaint against defendants VeronicoSevilla and Technology and Livelihood Resources Center for Annulment of Document, Reconveyance and Recovery of Possession of two lots, which had a total assessed value of P9,910.00, plus damages. The Sebes claimed that they owned the subject lots but, through fraud, defendant Sevilla got them to sign documents conveying the lots to him. In his Answer Sevilla insisted that he bought the lots from the Sebes in a regular manner. While the case was pending before the RTC, plaintiff GenerosoSebe and defendant VeronicoSevilla died so they were substituted by their heirs as respondents in this case. The RTC dismissed the case for lack of jurisdiction over the subject matter considering that the ultimate relief that the Sebes

sought was the reconveyance of title and possession over two lots that had a total assessed value of less than P20,000.00. The RTC contended that it has jurisdiction over such actions when the assessed value of the property exceeds P20,000.00, otherwise, jurisdiction shall be with the first level courts. The RTC concluded that the Sebes should have filed their action with the Municipal Trial Court (MTC) of Dipolog City. On the other hand, the Sebes pointed out that the RTC mistakenly classified their action as one involving title to or possession of real property when, in fact, it was a case for the annulment of the documents and titles that defendant Sevilla got. Since such an action for annulment was incapable of pecuniary estimation, it squarely fell within the jurisdiction of the RTC as provided in Section 19 of Batas Pambansa 129, as amended.

necessary consequence of the defendants lack of title to real property. Further, although the certificate of title may have been lost, burned, or destroyed and later on reconstituted, title subsists and remains unaffected unless it is transferred or conveyed to another or subjected to a lien or encumbrance. The Sebes alleged that defendant Sevilla violated their right of ownership by tricking them into signing documents of absolute sale, rather than just a real estate mortgage to secure the loan that they got from him. Assuming that the Sebes can prove that they have title to or a rightful claim of ownership over the two lots, they would then be entitled, first, to secure evidence of ownership or certificates of title covering the same and, second, to possess and enjoy them. The court, in this situation, may in the exercise of its equity jurisdiction and without ordering the cancellation of the Torrens titles issued to defendant Sevilla, direct the latter to reconvey the two lots and their corresponding Torrens titles to them as true owners. The present action is, therefore, not about the declaration of the nullity of the documents or the reconveyance to the Sebes of the certificates of title covering the two lots. These would merely follow after the trial court shall have first resolved the issue of which between the contending parties is the lawful owner of such lots, the one also entitled to their possession. Based on the pleadings, the ultimate issue is whether or not defendant Sevilla defrauded the Sebes of their property by making them sign documents of conveyance rather than just a deed of real mortgage to secure their debt to him. The action is, therefore, about ascertaining which of these parties is the lawful owner of the subject lots, jurisdiction over which is determined by the assessed value of such lots. Here, the total assessed value of the two lots subject of the suit is P9,910.00. Clearly, this amount does not exceed the jurisdictional threshold value of P20,000.00 fixed by law. The other damages that the Sebes claim are merely incidental to their main action and, therefore, are excluded in the computation of the jurisdictional amount. WHEREFORE, premises considered, the petition is DISMISSED. The Order of the Regional Trial Court of Dipolog City is AFFIRMED. 20. IGLESIA EVANGELICA METODISTA v. NATANAEL JUANE JURISDICTION OF MTC OR MTC OVER DETAINTER CASES

Issue: Whether or not the Sebess action involving the two lots valued at less than P20,000.00 falls within the jurisdiction of the RTC (That is, was the Sebess action one involving title to, or possession of, real property or any interest in it or one the subject of which is incapable of pecuniary estimation) Held: NO. The action is one involving title to, or possession of, real property or any interest therein. Ratio: Whether a court has jurisdiction over the subject matter of a particular action is determined by the plaintiffs allegations in the complaint and the principal relief he seeks in the light of the law that apportions the jurisdiction of courts. An action involving title to real property means that the plaintiffs cause of action is based on a claim that he owns such property or that he has the legal rights to have exclusive control, possession, enjoyment, or disposition of the same. Title is the legal link between (1) a person who owns property and (2) the property itself. Title is different from a certificate of title which is the document of ownership under the Torrens system of registration issued by the government through the Register of Deeds. While title is the claim, right or interest in real property, a certificate of title is the evidence of such claim. Title gives the owner the right to demand or be issued a certificate of title, the holder of a certificate of title does not necessa rily possess valid title to the real property. Thus, a plaintiffs action for cancellation or nullification of a certificate of title may only be a

FACTS: These are two consolidated cases arising from a Complaint in a civil proceeding, captioned "Unlawful Detainer," filed by

IglesiaEvangelicaMetodista en las Islas Filipinas (IEMELIF), Inc. against Reverend Natanael B. Juane (Juane), in the MeTC of Manila, Branch 26. IEMELIF is a religious corporation existing and duly organized under Philippine laws. It is the absolute and registered owner of land where the Cathedral of the IglesiaEvangelicaMetodista en las Islas Filipinas is located together with other improvements including the Pastors residence and the churchs school. By virtue of the appointment and assignment of defendant Juane as Resident Pastor of the Cathedral Congregation in Tondo, Manila, he was authorized to stay in and occupy the Pastors residence inside the cathedral complex. He had authority to stay in the premises, but it was eventually expired upon Juanes reassignment as Resident Pastor of the Sta. Mesa Congregation. IEMELIF issued a demand for Juane to vacate the premises, but the latter refused to do so. Due to Juanes unwarranted failure and unjust refusal to vacate the premises, IEMELIF was left without recourse but to file legal action to enforce its right to have physical possession of the Cathedral premises. This was filed in the MeTC of Manila. Juane filed a Motion to Dismiss (another civil case).He contends that the Complaint therein actually involved intra-corporate controversies, which, under Republic Act No. 8799, otherwise known as the Securities Regulation Code, fell within the jurisdiction of the Regional Trial Court (RTC), not the MeTC.The MeTC denied Juanes Motion to Dismiss. It held that the case did not involve the issue of removal of a corporate officer, but rather the right to possess the IEMELIF Cathedral in Tondo. Juane filed a Motion for Reconsideration, but the same was denied by the MeTC.

reassignment by IEMELIF, and could not be the subject of an action for unlawful detainer under Rule 70 of the Rules of Court. Citing the case of Abrin v. Campos, the Court held that it is a well-settled rule that what determines the nature of the action, as well as the Court which has jurisdiction over the case, is the allegation made by the Plaintiff in his complaint. To resolve the issue of jurisdiction, the Court must interpret and apply the law on jurisdiction vis-a-vis the averments of the complaint. The defenses asserted in the answer or motion to dismiss are not to be considered in resolving the issue of jurisdiction, otherwise the question of jurisdiction could depend entirely upon the defendant. The jurisdictional elements needed to be alleged in a Complaint for unlawful detainer were all present in the Complaint filed by IEMELIF. Furthermore, the Complaint never alleged as issues the validity of IEMELIFs actions of reassigning Juane to another church, and later removing him as pastor. The invalidity of Juanes removal as the Resident Pastor of the IEMELIF Tondo Congregation and his reassignment as the Resident Pastor of the IEMELIF Sta. Mesa Congregation was a defense set up by Juane in his Motion to Dismiss, which cannot be considered in resolving the issue of jurisdiction.The Complaint, having stated the jurisdictional elements in an unlawful detainer case, was properly filed with the Metropolitan Trial Court. 21. SPOUSES CLEMENCIO C. SABITSANA, JR. and MA. ROSARIO M. SABITSANAvs.JUANITO F. MUERTEGUI, represented by his Attorney-in-Fact DOMINGO A. MUERTEGUI, JR. JURISDICTION OF RTC OVER ACTION FOR QUIETING OF TITLE AND RULE 63! Facts: In 1981, Alberto Garcia executed an unnotarized Deed of Sale in favor of respondent JuanitoMuertegui (Juanito) over a 7,500-square meter parcel of unregistered land located in Biliran, Leyte del Norte covered by a Tax Declaration issued in Garcias name. Juanitos father Domingo Sr. and brother Domingo Jr. took actual possession of the lot and planted thereon coconut and ipil-ipil trees. They also paid the real property taxes on the lot for the years 1980 up to 1998. In 1991, Garcia sold the lot to the Muertegui family lawyer, petitioner Atty. Sabitsana, through a notarized deed of absolute sale. The sale was registered with the Register of Deeds and a new Tax Declaration was issued in Atty. Sabitsanas name. Although Domingo Jr. and Sr. paid the real estate taxes, Atty. Sabitsana also paid real property

ISSUE: Whether the Complaint filed by IEMELIF against Juane constitutes an intra-corporate dispute beyond the jurisdiction of the MeTC. HELD: No. Therefore, the petition is GRANTED. The unlawful detainer case was properly filed in the MeTC of Manila. RATIO: The primary and ultimate purpose of IEMELIF in filing the Complaint in the civil case was to seek recovery of physical possession over the subject property, a matter within the jurisdiction of the MeTC.The CA was wrong in holding that the most contentious issue raised in the Complaint of IEMELIF, wasJuanes removal from office and reassignment, which was within the realm of intra-corporate controversies and the exclusive jurisdiction of the RTC. Juanes purported loss of the right to possess the subject property was merely incidental to his removal from office and

taxes in 1992, 1993, and 1999. In 1996, he introduced concrete improvements on the property, which shortly thereafter were destroyed by a typhoon. When Domingo Sr. passed away, his heirs applied for registration and coverage of the lot under the Public Land Act. Atty. Sabitsana then wrote a letter dated April 24, 1998to Department of Environment and Natural Resources CENRO/PENRO office opposing the application, claiming that he was the true owner of the lot. He asked that the application for registration be held in abeyance until the issue of conflicting ownership has been resolved. In 2000, Juanito, through his attorney-in-fact Domingo Jr., filed a Civil Case for quieting of title and preliminary injunction, against herein petitioners Atty. Sabitsana and his wife, Rosario, claiming that they bought the lot in bad faith and are exercising acts of possession and ownership over the same, which acts thus constitute a cloud over his title. The Complaint prayed, among others, that the Sabitsana Deed of Sale, the letter to CENRO/PENRO, and the TD in the name of Atty. Sabitsana be declared null and void and of no effect; that petitioners be ordered to respect and recognize Juanitos title over the lot; and that moral and exemplary damages, attorneys fees, and litigation expenses be awarded to him. In their Answer with Counterclaim, petitioners asserted mainly that the sale to Juanito is null and void absent the marital consent of Garcias wife; that they acquired the property in good faith and for value; and that the Complaint is barred by prescription and laches. They likewise insisted that the RTC of Naval, Biliran did not have jurisdiction over the case, which involved title to or interest in a parcel of land the assessed value of which is merely P1,230.00.

declaratory relief, which properly falls within the jurisdiction of the RTC pursuant to Rule 63 of the Rules. NOTE: Respondent has a better right to the lot. The sale to respondent Juanito was executed on September 2, 1981 via an unnotarized deed of sale, while the sale to petitioners was made via a notarized document only on October 17, 1991, or ten years thereafter. Thus, Juanito who was the first buyer has a better right to the lot, while the subsequent sale to petitioners is null and void, because when it was made, the seller Garcia was no longer the owner of the lot. 22. SULTAN YAHYA TOMAWIS V. HON. BALINDONG JURISDICTION OF SHARIA DISTRICT COURT FACTS: Private respondents filed with the Sharia District Court (SDC) an action for quieting of title of a parcel of land located in Banggolo, Marawi City, against petitioner. In his answer, petitioner debunked the sisters claim of ownership and raised, as one of his affirmative defenses treated by the court as a motion to dismiss, SDCs lack of jurisdiction over the subject matter of the case. As argued, the regular civil court, not SDC, had such jurisdiction pursuant to B.P. 129 or the Judiciary Reorganization Act of 1980. Respondent judge denied the motion. Petitioner repaired to the Court of Appeals (CA), Mindanao Station, on a petition for certiorari, mandamus, and prohibition under Rule 65 to nullify, on jurisdictional grounds, the SDC Orders. CA dismissed the petition on the ground that the CA was not empowered to resolve decisions, orders or final judgments of the SDCs. ISSUE and HELD: 1. Is the CA correct in dismissing the petition? NO. However, since the instant petition involves only a question of law on the jurisdiction of the SDC over a complaint for quieting of title, then it is properly instituted before this Court. 2. Does SDC have jurisdiction over the quieting of title case? YES. RATIO:

Issue:WON Regional Trial Court has jurisdiction over the suit for quieting of title. Held:Yes, it is clear under the Rules that an action for quieting of title may be instituted in the RTCs, regardless of the assessed value of the real property in dispute. Under Rule 63 of the Rules of Court, an action to quiet title to real property or remove clouds therefrom may be brought in the appropriate RTC. Ratio: It must be remembered that the suit for quieting of title was prompted by petitioners April 24, 1998 letter-opposition to respondents application for registration. Thus, in order to prevent a cloud from being cast upon his application for a title, respondent filed the Civil Case to obtain a declaration of his rights. In this sense, the action is one for

1. Prefatorily, the Court acknowledges the fact that decades after the enactment in 1989 of the law creating the Sharia Appellate Court and after the Court, per Resolution of June 8, 1999,authorized its creation, the Sharia Appellate Court has yet to be organized with the appointment of a Presiding Justice and two Associate Justices.Until such time that the

Sharia Appellate Court shall have been organized, however, appeals or petitions from final orders or decisions of the SDC filed with the CA shall be referred to a Special Division to be organized in any of the CA stations preferably composed of Muslim CA Justices. 2. On February 4, 1977, PD 1083created the Sharia courts, i.e., the SDC and the Sharia Circuit Court, both of limited jurisdictions. The Administrative Code of 1987 classified Sharia courts as regular courts, meaning they are part of the judicial department. Art. 143 of PD 1083 vests SDCs, in certain cases, with exclusive original jurisdiction and with concurrent original jurisdiction over certain causes of action. As far as relevant, Art. 143 reads as follows: (2) Concurrently with existing civil courts, the Sharia District Court shall have original jurisdiction over: XXX (b) All other personal and real actions not mentioned in paragraph 1 (d) wherein the parties involved are Muslims except those for forcible entry and unlawful detainer, which shall fall under the exclusive original jurisdiction of the Municipal Circuit Court. On August 14, 1981, BP 129 took effect. Sec. 19 of BP 129, as later amended by RA 7691, defining the jurisdiction of the RTCs, provides: exclusive original jurisdiction XXX (2) In all civil actions which involve the title to, or possession of, real property, or any interest therein, where the assessed value of the property involved exceeds P20,000,00 or, for civil actions in Metro Manila, where such value exceeds P50,000.00 except actions for forcible entry into and unlawful detainer of lands or buildings, original jurisdiction over which is conferred upon the Metropolitan Trial Courts, Municipal Trial Courts, and Municipal Circuit Trial Courts. As things stood prior to the effectivity date of BP 129, the SDC had, by virtue of PD 1083, original jurisdiction, concurrently with the RTCs and MTCs, over all personal and real actions outside the purview of Art. 143(1)(d) of PD 1083, in which the parties involved were Muslims, except those for ejectment. On the other hand, BP 129, as amended, vests the RTC or the municipal trial court with exclusive original jurisdiction in all civil actions that involve the title to or possession of real property, or any interest in it, and the value of the property subject of the case or the jurisdictional amount, determining whether the case comes within the jurisdictional competence of the RTC or the MTC. A reading of the pertinent provisions of BP 129 and PD 1083 shows that the former, a law of general application to civil courts, has no application to, and does not repeal, the provisions found in PD 1083, a special law,

which only refers to Sharia courts. A look at the scope of BP 129 clearly shows that Sharia courts were not included in the reorganization of courts that were formerly organized under RA 296. BP 129 was enacted to reorganize only existing civil courts and is a law of general application to the judiciary. In contrast, PD 1083 is a special law that only applies to Sharia courts. We must read and construe BP 129 and PD 1083 together, then by taking PD 1083 as an exception to the general law to reconcile the two laws. This is so since the legislature has not made any express repeal or modification of PD 1083, and it is well-settled that repeals of statutes by implication are not favored.In order to give effect to both laws at hand, we must continue to recognize the concurrent jurisdiction enjoyed by SDCs with that of RTCs under PD 1083. While we recognize the concurrent jurisdiction of the SDCs and the RTCs with respect to cases involving only Muslims, the SDC has exclusive original jurisdiction over all actions arising from contracts customary to Muslims to the exclusion of the RTCs, as the exception under PD 1083, while both courts have concurrent original jurisdiction over all other personal actions. Said jurisdictional conferment, found in Art. 143 of PD 1083, is applicable solely when both parties are Muslims and shall not be construed to operate to the prejudice of a non-Muslim, who may be the opposing party against a Muslim. 23. Sps. LYDIA FLORES-CRUZ and REYNALDO I. CRUZ vs. Sps. LEONARDO and ILUMINADA GOLI-CRUZ, et al. JURISDICTION DETERMINED SOLELY BY THE ALLEGATIONS IN THE COMPLAINT FACTS: Sometime in 1999, Petitioner Spouses purchased a parcel of land from a relative. The property was situated in Pulong Yantok, Angat, Bulacan. They paid real estate taxes but never occupied the property. Petitioners sold portions to third parties. Petitioners discovered sometime in 2000 that Respondents were occupying a section of the land. Petitioners offered to sell the land to them. Since they could not agree on the price, Petitioners demanded that Respondents vacate the land. Petitioners filed a case for recovery of possession of the land in the Regional Trial Court (RTC) of Malolos, Bulacan in 2001. Respondents filed a motion to dismiss claiming, that the RTC had no jurisdiction over the case as it should have been filed in the MTC since it was a summary action for ejectment under Rule 70 of the Rules of Court. RTC denied the motion to dismiss and eventually decided in favor of Petitioners. On appeal, the CA ruled in favor of Respondents and dismissed

the complaint. It held that the RTC had no jurisdiction over the action for recovery of possession because petitioners had been dispossessed of the property for less than a year. It held that the complaint was one for unlawful detainer which should have been filed in the MTC. ISSUE: Whether or not the RTC has jurisdiction over the case. HELD: No; RTC did not have jurisdiction over the case. It is axiomatic that jurisdiction is determined solely by the allegations in the complaint and not by evidence adduced during trial. One cannot advert to anything not set forth in the complaint. In this case, the complaint alleged that the Petitioners tolerated the occupation of the Respondents and filed the case with the RTC in less than 1 year from the demand to vacate. When the case was filed in 2001, Congress had already approved Republic Act No. 7691 which expanded the MTCs jurisdiction to include other actions involving title to or possession of real property ( accion publiciana and reinvindicatoria) where the assessed value of the property does not exceed P20,000 (or P50,000, for actions filed in Metro Manila). The complaint did not contain any allegations as to the value of the property. Thus, the Court could not determine where jurisdiction lies. Wherefore, petition is denied.

ISSUE Whether or not Legarda complied with the KatarungangPambaranngay Law HELD Legarda complied with the KatarungangPambaranggay Law which provides for conciliation before any complaint, petition, action or proceeding involving any matter within the authority of the Lupon of the Baranggay shall be file or instituted in the court. Records show that the respondent referred the dispute to the barangay for conciliation proceedings. However, no settlement took place or was possible because the petitioners in the instant case repeatedly refused to meet Legarda or her representative despite the issuance of several summons by the barangay chairman. The Certification to File Action was therefore issued to serve as a basis for the filing of the corresponding complaint. Also, the petitioners did not object to the presentation of the Certification during the hearing. They did not also question the certificate. It was deemed admitted for failure to deny the same under oath in their Answer, as argued by the respondent. Assuming that respondent did not really refer the dispute to the barangay for conciliation proceedings, the trial court can still take cognizance of the case due to the petitioners failure to object to such lack of conciliation proceedings during the hearing. Further, the petitioners alleged that the real owner of the property is not the father of the respondent but Benito Legarda Incorporated. Assuming this is true, the SC said that the barangay conciliation will become truly unnecessary since of the parties to the case is a juridical person. Non-compliance with a condition precedent under the KatarungangPambaranggay Law does not prevent a court of competent jurisdiction to take cognizance of a case where the defendant fails to object to the courts exercise of jurisdiction. Such objection should be seasonably made before the court taking first cognizance if the complaint and must be raised in the Answer, or in such other pleading allowed by under the Rules of Court.

24. ESPINO v LEGARDA BARANGGAY CONCILIATION FACTS Legarda filed complaints for accionpubliciana in the RTC of Manila against Espino. The latter allegedly entered the premises of Legardas lots and constructed houses thereon. The Espinos refused to vacate even after demands to do so. The Respondent in the instant case also went to the barangay which summoned the Espinos who ignored the summons. In the Answer of Espino, he claimed that they cannot be evicted because, among other reasons, respondent Legarda did not resort to conciliation proceedings before the barangay prior to the filing of the complaint a rule provided by the KatarungangPambaranggay Law. The RTC ruled in favour of the respondent, ordering Espino to vacate. This decision was appealed to the CA which affirmed the RTC ruling since Legarda showed a Certificate from the barangay chairman that conciliation proceedings took place, and that the Espinos did not object to the presentation or challenge the certification itself.

25. ZAMORA V IZQUIERDO GR NO. 146195 NOVEMBER 18, 2004 BARANGAY CONCILIATION

FACTS In 1973, Carmen Izquierdo and Pablo Zamora entered into a verbal stipulation whereby the former leased to the latter one of her apartment units in Caloocan City for residence purposes (for one family only) at a rate of 3k per month. After the death of Carmen in 1996, the heirs, herein respondents, through the formers attorney-in-fact, prepared a new contract of lease, increasing the rent to 3,600. Petitioners refused to sign it. After Pablo died in 1997, his wife, Avelina, and their children, herein petitioners, continued to reside in the apartment unit although refusing to pay the increased rental and persisted in operating a photocopying business in the same apartment. Avelina applied with MWSS for a water line installation in the premises and sought to get the required written consent from the owner. Respondents declined because of the aforementioned refusal to pay the increased rent. Avelina then filed a complaint against the attorney-in-fact, Punzalan, with the Office of the Punong Barangay. The following day, Punzalan sent a letter to Avelina, informing her that the lease is being terminated and demanding that petitioners vacate the premises within 30 days from notice. Despite several barangay conciliation sessions, the parties failed to settle their dispute amicably. Hence, the Bgy Chairman issued a Certification to File Action. Respondents filed with the MTC a complaint for unlawful detainer and damages, which the petitioners sought to dismiss on the ground that the controversy was not referred to the barangay for conciliation, the first action pertaining to another dispute (the non-signing of the respondents). Petitioners also contend that when the parties failed to reach an amicable settlement before the Lupong Tagapamayapa, the Punong Barangay did not constitute the Pangkat ng Tagapagkasundo before whom mediation or arbitration proceedings should have been conducted, in violation of the Local Govt Code. MTC, RTC, and CA ruled in favor of petitioners. ISSUE Whether the case should be dismissed HELD YES

before the Lupon Chairman or the Pangkat as a precondition to filing a complaint in court. To wit: Section 410. Procedure for Amicable Settlement (b) Mediation by lupon chairman upon receipt of the complaint, the lupon chairman shall, within the next working day, summon the respondents, with notice to the complainants for them and their witnesses to appear before him for a mediation of their conflicting interests. If he fails in his mediation effort within 15 days from the first meeting of the parties before him, he shall forthwith set a date for the constitution of the pangkat in accordance with the provisions of this chapter. In the case at bar, the Punong Barangay as Chairman of the Lupong Tagapamayapa conducted conciliation proceedings to resolve the dispute between the parties. Contrary to the petitioners contention, the complaint does not only allege, as a cause of action, the refusal of respondents attorney-in-fact to give her consent to the installation of water facilities in the premises, but also petitioners violation of the terms of the lease, specifically their use of a portion therein for their photocopying business and their failure to pay the increased rental. While the Pangkat was not constituted, the parties met 9 times at the Office of the Barangay Chairman for conciliation. It is thus manifest that there was substantial compliance with the law which does not require strict adherence thereto. Additionally, the petitioners motion to dismiss is proscribed by Section 19(a) of the 1991 Revised Rule on Summary Procedure, which permits the filing of such pleading only when the ground for dismissal of the complaint is anchored on lack of jurisdiction over the subject matter, or failure by the complainant to refer the subject matter of his complaint to the Lupon for conciliation prior to its filing with the court. As discussed earlier, the case was referred to the Lupon Chairman for conciliation. PETITION DENIED. 26. REPUBLIC V. CA (PENDING) See Page 12 SUMMARY PROCEEDINGS UNDER THE FAMILY CODE

27. REPUBLIC OF THE PHILIPPINES vs. GLORIA BERMUDEZLORINO SUMMARY PROCEEDINGS UNDER THE FAMILY CODE FACTS:

https://www.facebook.com/groups/286534214811733/4184049216 24661/?ref=notif&notif_t=group_comment_reply
RATIO Pursuant to PD 1508 (The Katarungang Pambarangay Law), now included under the LGC, the parties are required to undergo a conciliation process

A verified petition was filed by herein petitioner through counsel alleging that she married Francisco Lorino, Jr. on June 12, 1987 but because of the violent character of his husband, she decided to go back to her parents and lived separately from her husband and went abroad to work. After nine (9) years, there was absolutely no news about him and she believes that he is already dead and is now seeking through this petition for a Court declaration that her husband is judicially presumed dead for the purpose of remarriage. Finding the said petition to be sufficient in form and substance, the same is hereby set for hearing before this Court on September 18, 2000 at 8:30 oclock in the morning at which place, date and time, any or all persons who may claim any interest thereto may appear and show cause why the same should not be granted. A copy of the order was then published and posted in accordance with the requirements set by law. Pieces of evidence of the required publications and affidavits were there submitted to the court. RTC, finding merit in the summary petition, rendered judgment granting the petition. Despite the judgment being immediately final and executory under the provisions of Article 247 of the Family Code, thus:(Art. 247. The judgment of the court shall be immediately final and executory)the Office of the Solicitor General, for the Republic of the Philippines, nevertheless filed a Notice of Appeal. So the RTC had the records elevated to the Court of Appeals. the Court of Appeals, treating the case as an ordinary appealed case under Rule 41 of the Revised Rules on Civil Procedure, denied the Republics appeal and accordingly affirmed the appealed RTC decision. Without filing any motion for reconsideration, petitioner Republic directly went to SC via the instant recourse under Rule 45, maintaining that the petition raises a pure question of law that does not require prior filing of a motion for reconsideration. ISSUES: 1. WHETHER OR NOT THE COURT OF APPEALS DULY ACQUIRED JURISDICTION OVER THE APPEAL ON A FINAL AND EXECUTORY JUDGMENT OF THE REGIONAL TRIAL COURT? HELD: 1. NO, SC ruled against the Republic. RATIO: 1.Article 238 of the Family Code, under Title XI: SUMMARY JUDICIAL PROCEEDINGS IN THE FAMILY LAW, sets the tenor for cases covered by these rules, to wit:
Art. 238. Until modified by the Supreme Court, the procedural rules in this Title shall apply in all cases provided for in this Code requiring summary court proceedings. Such cases shall be decided in an expeditious manner without regard to technical rules.

In Summary Judicial Proceedings under the Family Code, there is no reglementary period within which to perfect an appeal, precisely because judgments rendered thereunder, by express provision of Section 247, Family Code, supra, are immediately final and executory. It was erroneous, therefore, on the part of the RTC to give due course to the Republics appeal and order the transmittal of the entire records of the case to the Court of Appeals. the right to appeal is not a natural right nor is it a part of due process, for it is merely a statutory privilege. Since, by express mandate of Article 247 of the Family Code, all judgments rendered in summary judicial proceedings in Family Law are immediately final and executory, the right to appeal was not granted to any of the parties therein. The Republic of the Philippines, as oppositor in the petition for declaration of presumptive death, should not be treated differently. It had no right to appeal the RTC decision. Nothing is more settled in law than that when a judgment becomes final and executory it becomes immutable and unalterable. The same may no longer be modified in any respect, even if the modification is meant to correct what is perceived to be an erroneous conclusion of fact or law, and whether made by the highest court of the land.

But, if only to set the records straight and for the future guidance of the bench and the bar, let it be stated that the RTCs decision, was immediately final and executory upon notice to the parties. It was erroneous for the OSG to file a notice of appeal, and for the RTC to give due course thereto. The Court of Appeals acquired no jurisdiction over the case, and should have dismissed the appeal outright on that ground. ADDITIONAL INFO.: This judgment of denial was elevated to this Court via a petition for review on certiorari under Rule 45. Although the result of the Court of Appeals denial of the appeal would apparently be the same, there is a big difference between having the supposed appeal dismissed for lack of jurisdiction by virtue of the fact that the RTC decision sought to be appealed is immediately final and executory, and the denial of the appeal for lack of merit. In the former, the supposed appellee can immediately ask for the issuance of an Entry of Judgment in the RTC, whereas, in the latter, the appellant can still raise the matter to this Court on petition for review and the RTC judgment cannot be executed until this Court makes the final pronouncement. (CA should have dismissed the case due to lack of jurisdiction rather than due to lack of merits of the case, both CA and RTC committed reversible error for giving due course on the appeal of OSG.) 28. CECILIO HERNANDEZ v. SAN JUAN-SANTOS

WRIT OF HABEAS CORPUS, RULE 92, RULE 103! FACTS: Petitioners are the half-brother, half-sister, and step mother of Lulu Hernandez while Respondent is the latters maternal first cousin and legal guardian. Lulus mother died while giving birth to her and since then, she was raised by her maternal uncle, Sotero San Juan while her father remarried. Lulu has inherited valuable properties from her mother side as the only child of her mom and as the sole testate heir of Sotero. She is considerably weak minded and has a tendency to be violent which lead to her not finishing elementary education. Due to this, her father Felix and, after his death, the petitioners informally managed her properties and made some shady deals. To add insult to injury, the petitioners maltreated her to the extent that she became terribly unhygienic and extremely ill. As such, she sought the assistance of Respondent to help her manage her properties which was being dissipated by the Petitioner for their own personal gain. Respondent filed a Petition for Guardianship in the RTC of San Mateo, Rizal. In this, the Petitioners filed a Motion to Intervene and to oppose the same. The RTC granted Respondents petition and she was appointed legal guardian of Lulu. This decision was appealed by the Petitioners to the CA which affirmed the RTC. However, during the pendency of the appeal in the CA, the Petitioners abducted Lulu thereby compelling Respondent to seek assistance from the PNP and forced her to file a Petition for Habeas Corpus in the CA. The CA granted the Petition for Habeas Corpus. Thus, the present case. ISSUE: 1) Is Lulu Hernandez an incompetent person who requires the appointment of a legal guardian over her person and property? 2) Was the Grant of the Petition for Habeas Corpus valid? HELD: 1) Yes; 2) Yes RULING: 1) It is being argued by the petitioners that the opinions of Lulus attending physicians, which were used by the Respondent in obtaining guardianship over the person and property of Lulu, are inadmissible as evidence as they were not experts in psychiatry. Respondent therefore failed to prove that Lulu's illnesses rendered her an incompetent. She should have been presumed to be of sound mind and/or in full possession of her mental capacity. This however is clearly without merit. Under Section 50, Rule 103 of the Rules of Court, an ordinary witness may give his opinion on the mental sanity of a person with whom he is sufficiently acquainted. Lulu's attending physicians spoke and interacted with her. Such occasions allowed them to thoroughly observe her behavior and conclude that her intelligence level was below average and her mental stage below normal. Their opinions were admissible in evidence.

Furthermore, where the sanity of a person is at issue, expert opinion is not necessary. The observations of the trial judge coupled with evidence establishing the person's state of mental sanity will suffice. Here, the trial judge was given ample opportunity to observe Lulu personally when she testified before the RTC. Under Section 2, Rule 92 of the Rules of Court, persons who, though of sound mind but by reason of age, disease, weak mind or other similar causes are incapable of taking care of themselves and their property without outside aid, are considered as incompetents who may properly be placed under guardianship. The RTC and the CA both found that Lulu was incapable of taking care of herself and her properties without outside aid due to her ailments and weak mind. The Court adopted these factual findings of the RTC as affirmed by the CA. Similarly, there was no compelling reason to reverse the trial and appellate courts finding as to the propriety of respondent's appointment as the judicial guardian of Lulu since she was the only person the latter trusts and due to the fact that Guardianship is a trust relationship. 2) Inasmuch as respondents appointment as the judicial guardian of Lulu was proper, the issuance of a writ of habeas corpus in her favor was also in order. This writ extends to all cases of illegal confinement or detention or by which the rightful custody of person is withheld from the one entitled thereto. Respondent, as the judicial guardian of Lulu, was duty-bound to care for and protect her ward. For her to perform her obligation, respondent must have custody of Lulu. Thus, she was entitled to this writ after she was unduly deprived of the custody of her ward. 29. NAVIA V. PARDICO G.R. NO. 184467, JUNE 19, 2012 WRIT OF AMPARO Facts: A vehicle of Asia Land arrived at the house of Lolita M. Lapore (Lolita) located at Grand Royale Subdivision, Malolos City. The arrival of the vehicle awakened Lolitas son, Enrique Lapore (Bong), and Benhur Pardico (Ben), who were then both staying in her house. When Lolita went out to investigate, she saw two uniformed guards disembarking from the vehicle. One of them immediately asked Lolita where they could find her son Bong. Before Lolita could answer, the guard saw Bong and told him that he and Ben should go with them to the security office of Asian Land because a complaint was lodged against them for theft of electric wires and lamps in the subdivision. The brothers were then taken to the security office and underwent investigation headed by Navia. Later on, Lolita was instructed to sign an entry in the guards logbook

where she undertook not to allow Ben to stay in her house anymore. Thereafter, Navia again asked Lolita to sign the logbook. Upon Lolitas inquiry as to why she had to sign again, Navia explained that they needed proof that they released her son Bong unharmed but that Ben had to stay as the latters case will be forwarded to the barangay. Since she has poor eyesight, Lolita obligingly signed the logbook without reading it and then left with Bong. Moments after Lolita and Bong reached their house, Buising arrived and asked Lolita to sign the logbook again. Lolita asked Buising why she had to sign again when she already twice signed the logbook at the headquarters. Buising assured her that what she was about to sign only pertains to Bongs release. Since it was dark and she has poor eyesight, Lolita took Buisings word and signed the logbook without, again, reading what was written in it. The following morning, Virginia went to the Asian Land security office to visit her husband Ben, but only to be told that petitioners had already released him together with Bong the night before. She then looked for Ben, asked around, and went to the barangay. Since she could not still find her husband, Virginia reported the matter to the police. In the course of the investigation on Bens disappearance, it dawned upon Lolita that petitioners took advantage of her poor eyesight and naivete. They made her sign the logbook as a witness that they already released Ben when in truth and in fact she never witnessed his actual release. The last time she saw Ben was when she left him in petitioners custody at the security office.

State or a political organization, followed by a refusal to acknowledge the same or give information on the fate or whereabouts of said missing persons, with the intention of removing them from the protection of the law for a prolonged period of time. Simply put, the petitioner in an amparo case has the burden of proving by substantial evidence the indispensable element of government participation. Furthermore, the court is aware that under Section 1 of the Rule on Writ of Amparo, a writ of amparo may lie against a private individual or entity. But even if the person sought to be held accountable or responsible in an amparo petition is a private individual or entity, still, government involvement in the disappearance remains an indispensable element. Here, petitioners are mere security guards at Grand Royale Subdivision in Brgy. Lugam, Malolos City and their principal, the Asian Land, is a private entity. They do not work for the government and nothing has been presented that would link or connect them to some covert police, military or governmental operation. 30. COL. DELA MERCED vs. GSIS and Sps. MANLONGAT LIS PENDENS (NOTE: I CANT DECIDE WHERE TO CLASSIFY THIS PROPERLY) BUT IN RIANOs BOOK, LIS PENDENS IS DISCUSSED UNDER ACTION IN REM AND PERSONAM A transferee pendente lite of registered land, whose title bears a notice of a pending litigation involving his transferors title to the said land, is bound by the outcome of the litigation, whether it be for or against his transferor. Given this principle, the modification of the final decision against the transferor in order to include the transferee pendente lite does not violate the doctrine of immutability of final judgments. His inclusion does not add to or change the judgment; it is only a legal consequence of the established doctrine that a final judgment binds the privy of a litigating party. FACTS: Five registered parcels of land (Lots 6, 7, 8 and 10 of Block 2 and Lot 8 of Block 8) in Antonio Subdivision, Pasig were originally owned by Jose C. Zulueta as evidenced by TCT No. 26105. Later, Zulueta spouses mortgaged several lots to the GSIS, including the subject properties. GSIS eventually foreclosed them and ownership was consolidated in its favor. Upon learning about the foreclosure, Francisco Dela Merced, Col. Dela Merceds predecessor filed a complaint praying for the nullity of the foreclosure on the ground that he is the one who owns the lots at the time of the foreclosure. Dela Merced also impleaded Victor and Milagros Manlongat, who were claiming Lot 6, Block 2 by virtue of a sale executed by the GSIS in their daughters (Elizabeth Ma nlongat)

Issue: Whether or not the Writ of Amparo filed by Virginia was sufficient in form and substance to be considered valid. Held: Virginia's Petition for Writ of Amparo is fatally defective and must perforced be dismissed. Ratio: The Rule on the Writ of Amparo was promulgated to arrest the rampant extralegal killings and enforced disappearances in the country. Its purpose is to provide an expeditious and effective relief to any person whose right to life, liberty and security is violated or threatened with violation by an unlawful act or omission of a public official or employee, or of a private individual or entity. Sect ion 1 of the Rule on Writ of Amparo provides "The petition for a writ of amparo is a remedy available to any person whose right to life, liberty and security is violated or threatened with violation by an unlawful act or omission of a public official or employee, or of a private individual or entity. The writ shall cover extralegal killings and enforced disappearances or threats thereof." the protective writ of amparo to issue, allegation and proof that the persons subject thereof are missing are not enough. It must also be shown and proved by substantial evidence that the disappearance was carried out by, or with the authorization, support or acquiescence of, the

favor. Dela Merced argued that, due to the nullity of GSISs foreclosure, it had no ownership right that could be transferred to Elizabeth Manlongat. Dela Merced caused the annotation of lis pendenson GSISs title in order to protect his interests in the subject properties. Dela Merced died and was substituted by his heirs. The SC rendered decision in favor of petitioner and nullified the foreclosure because these lots were never part of its mortgage agreement with the Zulueta spouses. Judgment became final and petitioners filed a Motion for Execution19 with Branch 160 of the RTC of Pasig City. ISSUE: Whether a final and executory judgment against GSIS and Manlongat can be enforced against their successors-in-interest or holders of derivative titles HELD: YES RATIO: Enforcement of judgment against transferees pendente lite A notice of lis pendens is an announcement to the whole world that a particular real property is in litigation, serving as a warning that one who acquires an interest over said property does so at his own risk, or that he gambles on the result of the litigation over the said property. Once a notice of lis pendens has been duly registered, any cancellation or issuance of the title of the land involved as well as any subsequent transaction affecting the same, would have to be subject to the outcome of the litigation. There can be no risk of losing the property or any part thereof as a result of any conveyance of the land or any encumbrance that may be made thereon posterior to the filing of the notice of lis pendens. Dela Merced caused the annotation of lis pendens on GSISs title. This was cancelled with respect to two lots and new individual titles, which carried over the notice of lis pendens, were issued to the transferees. These two transferees had notice of the litigation involving GSISs ownership over the subject properties, and were bound by the outcome of the litigation. Such transferee undertakes to respect the outcome of the litigation. The existence of these entries on the transferees titles bars any defense of good faith against petitioners and effectively makes them mere privies of GSIS and subject to whatever rights GSIS might have in the subject properties, which (as it turns out) is none at all. What they possess are derivative titles of the GSISs title over the subject lands which are null and void. The title they obtained affords them no special protection and they cannot

invoke the rights of a purchaser in good faith and cannot acquire better rights than those of the predecessor-in-interest. The Court cannot accept GSISs theory that the dispositive portion of a previous Decision is enforceable only against GSISs title because it does not contain the phrase "and all its derivative titles." GSISs narrow interpretation would render nugatory the principle that a final judgment against a party is binding on his privies and successors-ininterest. In a case, the Court explained that an action is binding on the privies of the litigants even if such privies are not literally parties to the action. Their inclusion in the writ of execution does not vary or exceed the terms of the judgment. In the same way, the inclusion of the "derivative titles" in the writ of execution will not alter the order of the cancellation of GSISs title.

31. HEIRS OF JOSE SY BANG, et al. v. ROLANDO SY, et al. LIS PENDENS, RULE 14! NOTE: This is a long case guys, and almost purely civpro. But in the outline its just under lis pendens, so thats what youll find here. For the other civpro-related issues, refer to the original case. FACTS: This case is a consolidation of two Petitions for Review. But for the pertinent issue of lis pendens, only the first petition is included here. Respondent Rolando Sy filed a Complaint for Partition in the RTC against spouses Jose Sy Bang and Iluminada Tan, etc. Respondents are children of Sy Bang from his second marriage, while petitioners are also children of Sy Bang but from his first marriage as well as his current marriage with Iluminda Tan. Sy Bang died intestate, leaving behind real and personal properties, including several businesses. On the Third Partial Decision based on the above complaint, the RTC Judge Puno ruled in favor of Rolando and the other respondents, declaring them heirs of Sy Bang and entitled to his estate. Subsequently, the Judge issued two Orders, the relevant one of which is the Second Order in which the Judge ordered the immediate cancellation of the lis pendens annotated at the back of the certificates of title in the names of Bartolome Sy, Rosalino Sy and Rolando Sy (the respondents). On appeal, the CA rendered its assailed decision. It held that Judge Puno acted correctly in issuing the assailed Third Partial

Decision. The CA found proper the Second Order to cancel the notice of lis pendens. In this Petition for Review, petitioners question, among others, the Second Order canceling the notice of lis pendens ex parte and without any showing that the notice was for the purpose of molesting the adverse parties, or that it was not necessary to protect the rights of the party who caused it to be recorded.

The CA found, and the SC affirmed, that Rosalino, Bartolome and Rolando were able to prove that the notice was intended merely to molest and harass the owners of the property, some of whom were not parties to the case. It was also proven that the interest of Oscar Sy, who caused the notice to be annotated, was only 1/14 of the assessed value of the property. Moreover, Rosalino, Bartolome and Rolando were ordered to post a P50,000.00 bond to protect whatever rights or interest Oscar Sy may have in the properties under litis pendentia.

ISSUE: Whether or not the notice of lis pendens was properly cancelled HELD: YES RATIO: The SC makes reference to Sec. 77 of PD 1529. The filing of a notice of lis pendens has a two-fold effect: (1) to keep the subject matter of the litigation within the power of the court until the entry of the final judgment in order to prevent the final judgment from being defeated by successive alienations; and (2) to bind a purchaser, bona fide or not, of the land subject of the litigation to the judgment or decree that the court will promulgate subsequently. While the trial court has an inherent power to cancel a notice of lis pendens, such power is to be exercised within the express confines of the law. As provided in Section 14, Rule 13 of the 1997 Rules of Civil Procedure, a notice of lis pendens may be cancelled on two grounds: (1) when the annotation was for the purpose of molesting the title of the adverse party, or (2) when the annotation is not necessary to protect the title of the party who caused it to be recorded. The notice is but an incident in an action, only an extrajudicial one. It does not affect the merits thereof. It is intended merely to constructively advise, or warn, all people who deal with the property that they so deal with it at their own risk, and whatever rights they may acquire in the property in any voluntary transaction are subject to the results of the action, and may well be inferior and subordinate to those which may be finally determined and laid down therein. The cancellation of such a precautionary notice is therefore also a mere incident in the action, and may be ordered by the Court having jurisdiction of it at any given time. And its continuance or removal-like the continuance or removal of a preliminary attachment of injunction-is not contingent on the existence of a final judgment in the action, and ordinarily has no effect on the merits thereof. 32. LHUILLIER V. BRITISH AIRWAYS JURISDICTION; RULE 14! FACTS: Edna Lhuillier filed a complaint for damages against British airways before the RTC of Makati City. She alleged that there were 2 rude flight attendants in the plane she boarded to Rome from London. She claims moral, nominal and exemplary damages. Summons together with a copy of the complaint were served to the General Manager of Euro-Philippine Airline Services, Inc. Respondent filed a motion to dismiss on the grounds of lack of jurisdiction over the case and person of the respondent, claiming that London or Rome has the jurisdiction over the case pursuant to the Warsaw convention. It was likewise alleged that summons was erroneously served on Euro Phil, which is not its resident agent in the Philippines. ISSUES/ HELD: 1. W/N the Philippine courts have jurisdiction? NO. 2. W/N the filing of motion to dismiss may be deemed as having submitted to the jurisdiction of the court? NO. RATIO: 1. Warsaw Convention governs actions arising from international air travel and provides the exclusive remedy for conduct which falls within its provisions. Under Article 28(1) of the Warsaw Convention, the plaintiff may bring the action for damages before

1. Court where the carrier is domiciled; 2. Court where the carrier has its principal place of business; 3. Court where the carrier has an establishment by which the contract has been made; or 4. Court of the place of destination In this case: (1) the domicile of respondent is London, United Kingdom; (2) the principal office of respondent airline is likewise in London, United Kingdom; (3) the ticket was purchased in Rome, Italy; and (4) the place of destination is Rome, Italy. 2. A special appearance before the courtchallenging its jurisdiction over the person through a motion to dismiss even if the movant invokes other groundsis not tantamount to estoppel or a waiver by the movant of his objection to jurisdiction over his person; and such is not constitutive of a voluntary submission to the jurisdiction of the court.

determined. Issue: W/N the case should be dismissed? Held: NO Ratio: 1. GENERAL RULE: A pleading which does not specify in the prayer the amount sought shall not be admitted or shall be expunged, and that a court acquires jurisdiction only upon payment of the prescribed docket fee. EXCEPTION: While the payment of prescribed docket fee is a jurisdictional requirement, even its non-payment at the time of filing does not automatically cause the dismissal of the case, as long as the fee is paid within the applicable prescriptive or reglementary period, more so when the party involved demonstrates a willingness to abide by the rules prescribing such payment. Thus, when insufficient filing fees were initially paid by the plaintiffsand there was no intention to defraud the government, the Manchester rule does not apply. The building constructed on respondents leased premises was specifically constructed to house a bank, hence, the idle period before another occupant with like business may opt to lease would be difficult to project. 2. ALSO, Metrobank raised the issue of lack of jurisdiction rather late. If the plaintiff fails to comply with this requirement, the defendant should timely raise the issue of jurisdiction or else he would be considered in estoppel. In the latter case, the balance between the appropriate docket fees and the amount actually paid by the plaintiff will be considered a lien on any award he may obtain in his favor Metrobank raised the issue of jurisdiction only before the appellate court after it and its co-petitioner participated in the proceedings before the trial court. TAKE NOTE THAT Respondent is STILL liable for the balance between the actual fees paid and the correct payable filing fees. SEE Rule 41 Sec 4 RE: PAYMENT OF DOCKETS IN CASE OF FILING OF AN APPEAL AND LASALETTE COLLEGE CASE UNDER SAID RULE FOLDER 12 RULE 1 GENERAL PROVISIONS

Consequently, there being no valid substituted services of summons made, the SB did not acquire jurisdiction over the persons of petitioner and her children. And perforce, the proceedings in the subject forfeiture cases, insofar as petitioner and her three children are concerned, are null and void for lack of jurisdiction.

33. MBTC v. Perez PAYMENT OF DOCKET FEES JURISDICTIONAL; RULE 1 Facts: Solidbank Corporation (Solidbank) forged a lease with Perez over a land. Solidbank was to, construct a one-storey building specifically suited for bank premises. Solidbank was later acquired by Metrobank. Metrobank sent a notice of termination of the lease. Perez filed a complaint for breach of contract and damages praying that, they be ordered to pay her the would be unrealized income for the ens uing idle months of the said building. During the course of the trial, Perez was unable to pay the complete amount of filing fees.What she paid did not cover her prayer for unrealized income for the ensuing idle months, for at the time of filing and payment, the period that the building would be idle could not yet be

SEE MBTC V. PEREZ (UNDER JURISDICTION) PAYMENT OF DOCKET FEES JURISDICTIONAL; RULE 1

34. GOV. ORLANDO FUA V COA Rule 1 FACTS The SangguniangPanlalawigan of the Province of Siquijor adopted a Resolution and passed an Appropriation Ordinance allocating a sum of money for the grant of extra Christmas Bonus to its employees and officers. The Governor, petitioner in this case, wrote a letter to former President Gloria MAcapagal Arroyo to approve the appropriation. On the marginal note, the President only wrote no objection. The provincial government, relying on the resolutions and Arroyos marginal note released the extra bonus and the legality of which was challenged by the provincial auditors and then by the regional cluster director of COA as a violation of the Local Government Code provision which imposes a limitation on Personal Service expenditure. The decision of the Regional Cluster Director to issue a Notice of Disallowance was appealed by the petitioner to the Commission on Audit-Legal and Adjudication Office (COA-LAO-Local) which only affirmed the Regional Cluster Directors Notice of Disallowance. Hence, this petition for certiorari to the SC. ISSUE Whether or not the petition should be given due course by the SC HELD No, the petition should not be given due course because of the petitioners failure to observe the doctrine of exhaustion of administrative remedies. The 1997 Revised Rules of Procedure of the COA states that the party aggrieved by a final order or decision of the Director may appeal to the Commission Proper. Also, any decision, order or resolution of the Commission may be brought to the Supreme Court on certiorariwithin 30 days from receipt of a copy thereof The immediate filing of the present petition for certiorari by the petitioner is a failure to exhaust the administrative remedies available to him. The general rule is that before a party may seek the intervention of the court, he should first avail himself of all the means afforded him by administrative processes. The issues which administrative agencies are authorized to decide should not be summarily taken from them and submitted to the court without first giving such administrative agency the opportunity to dispose of the same after due deliberation.

Also, the petitioner claims that the case is cognizable by the SC to which the latter says no. This petition cannot be considered as an exception to the general rule mentioned above since the issue presented is not a purely legal one. The special civil action of certiorari is a limited form of review and is a remedy of last recourse. It lies only when there is no appeal or plain, speedy and adequate remedy in the ordinary course of law. The Commission Proper, which is the tribunal possessing special knowledge, experience and tools to determine technical and intricate matters of fact involved in the conduct of audit, would still be the best body to determine whether the marginal note on the letter of request to the president is tantamount to the required approval. The petitioner should have appealed the Decision of the Director to the Commission Proper. He is not entitled to a writ of certiorari.

RULE 2 CAUSE

OF

A CTION

35. PHILIPPINE CHARTERINSURANCE CORPORATION (PCIC) vs. PHILIPPINE NATIONALCONSTRUCTION CORPORATION (PNCC) CAUSE OF ACTION ELEMENTS FACTS: PNCC conducted a public bidding for the supply of labor, materials, tools, supervision, equipment, and other incidentals necessary for the fabrication and delivery of 27 tollbooths to be used for the automation of toll collection along the expressways. Orlando Kalingo (Kalingo) won in the bidding and was awarded the contract. PNCC issued in favor of Kalingo Purchase Order (P.O.) No. 71024L for 25 units of tollbooths and P.O. No.71025L for two units of tollbooths. Each P.O. is covered by a surety bond with the following conditions: (1) the liability of PCIC under the bonds expires on March 16, 1998; and (2) a written extrajudicial demand must first be tendered to the surety, PCIC, within 15 days from the expiration date; otherwise PCIC shall not be liable thereunder and the obligee waives the right to claim or file any court action to collect on the bond. Kalingodefaulted. PNCC filed awrittenextrajudicial claim against PCIC,which went unheeded despite repeated demands, so PNCC filed with the RTC a complaint for collection of a sum of money against

Kalingo and PCIC.PNCC's complaint against PCIC called solely on PCIC Bond No. 27547; itdid not raise or plead collection under PCIC Bond No. 27546 which secured the down payment on P.O. No. 71025L. The trial court made no ruling on PCICs liability under PCIC Bond No. 27546, a claim that was not pleaded in the complaint. The appellate court, on the other hand, held that PCIC, as surety, is liable jointly and severally with Kalingo for the amount of the two bonds (27564 and 27546) securing the two POs to Kalingo. ISSUE and HELD: Whether the CA erred in ruling that PCIC should also be held liable under Bond No. 276546, collection under which was NOT subject of PNCCscompaint for collection of sum of money? YES. RATIO: The fundamental rule is that reliefs granted a litigant are limited to those specifically prayed for in the complaint; other reliefs prayed for may be granted only when related to the specific prayer(s) in the pleadings and supported by the evidence on record. Necessarily, any such relief may be granted only where a cause of action therefor exists, based on the complaint, the pleadings, and the evidence on record. Section 2, Rule 2 of the 1997 Rules of Civil Procedure defines a cause of action as the act or omission by which a party violates the right of another. Its essential elements are as follows: (1)A right in favor of the plaintiff by whatever means and under whatever law it arises or is created; (2) An obligation on the part of the named defendant to respect or not to violate such right; and (3) Act or omission on the part of such defendant in violation of the right of the plaintiff or constituting a breach of the obligation of the defendant to the plaintiff for which the latter may maintain an action for recovery of damages or other appropriate relief. Only upon the occurrence of the last element does a cause of action arise, giving the plaintiff the right to maintain an action in court for recovery of damages or other appropriate relief. The 2 bonds state that PCIC will not be liable for any claim not presented to it in writing within 15 days from the expiration of the bond, and that the obligee (PNCC) thereby waives its right to claim or file any court action against the surety (PCIC) after the termination of 15 days from the time its cause of action accrues. This written claim provision creates a condition precedent for the accrual of: (1) PCICs obligation to comply with its promise under the particular bond, and of (2) PNCC's right to collect or sue on these bonds. The records reveal that PNCC complied with the written claim provision, but only with respect to PCIC Bond No. 27547. Nothing in the records

shows that PNCC ever complied with the provision with respect to PCIC Bond No. 27546. Under the circumstances, PNCCs cause of action with respect to PCIC Bond No. 27546 did not and cannot exist, such that no relief for collection thereunder may be validly awarded. Hence, the trial courts decision finding PCIC liable solely under PCIC Bond No. 27547 is correct not only because collection under the other bond, PCIC Bond No. 27546, was not raised or pleaded in the complaint, but for the more important reason that no cause of action arose in PNCCs favor with respect to this bond. Consequently, the appellate court was in error for including liability under PCIC Bond No. 27546. PNCCs argument alluding to Section 2(c), Rule 7 o f the Rules of Court is misplaced. A general prayer for other reliefs just and equitable appearing on a complaint or pleading normally enables the court to award reliefs supported by the complaint or other pleadings, by the facts admitted at the trial, and by the evidence adduced by the parties, even if these reliefs are not specifically prayed for in the complaint. We cannot, however, grant PNCC the other relief of recovering under PCIC Bond No. 27546 because of the respect due the contractual stipulations of the parties. While it is true that PCICs liability under PCIC Bond No. 27546 would have been clear under ordinary circumstances (considering that Kalingo's default under his contract with PNCC is now beyond dispute), it cannot be denied that the bond contains a written claim provision, and compliance with it is essential for the accrual of PCICs liability and PNCCs right to collect under the bond. For its failure to file a written claim with PCIC within 15 days from the bonds expiry date, PNCC clearly waived its right to collect under PCIC Bond No. 27546. That, wittingly or unwittingly, PNCC did not collect under one bond in favor of calling on the other creates no other conclusion than that the right to collect under the former had been lost. Consequently, PNCCs cause of action with respect to PCIC Bond No. 27546 cannot juridically exist and no relief therefore may be validly given. 36. UNICAPITAL v. RAFAEL CONSING CAUSE OF ACTION FACTS: Petitioner obtained for himself and his mother various loans amounting to P18,000,000 from Unicapital. The loans were secured by Real Estate Mortgage (REM) constituted on parcel of land in Cavite registered under the name of the mother of petitioner (Cecilia Dela Cruz). In pursuance with the option to purchase on the mortgaged property, Unicapital agreed to purchase one half of the property. The payment was set off by the loans of the petitioner while the other half was purchased by Plus

Builders, Inc, a joint venture partner of the Unicapital. Before both companies could develop the land, they discovered that the property was in the names of Po Willie Yu and Juanito Tan Teng. Thus, Unicapital demanded the return of the total amount of P41,377,851.48 as of April 19, 1999 that had been paid to and received by de la Cruz and Consing, but the latter ignored the demands.

respondent judge in issuing the orders referred to in the instant petition was not made with grave abuse of discretion. The case in Unicapital involved fraud and it is well settled that a civil action based on defamation, fraud and physical injuries may be independently instituted pursuant to Article 33 of the Civil Code, and does not operate as a prejudicial question that will justify the suspension of a criminal.

On July 22, 1999, Petitioner Consing filed Civil Case in the Pasig City Regional Trial Court to enjoin Unicapital from proceeding against him for the collection of the sum of money on the ground that he had acted as a mere agent of his mother. On the same date, Unicapital initiated a criminal complaint for estafa through falsification of public document against Petitioner Consing and the mother in the Makati City Prosecutors Office. Finding probable cause, City Prosecutor filed an information for estafa against Petioner. In 2000, Petitioner moved to defer arraignment because of the existence of a prejudicial question. RTC granted and suspended the proceedings in Makati Criminal Case. The State thus assailed in the CA the last two orders of the RTC in the Makati criminal case via petition for certiorari. It was found that indeed, the resolution of the pasig civil case is prejudicial to the Cavite and Makati Criminal Cases. Meanwhile, Plus Builders commenced an action for damages against Petitioner Consing in RTC Manila. Petitioner again filed a motion to defer the arraignment on the ground of the existence of a prejudicial question. CA Ruled that there is NO Prejudicial question. Thus, petitioner filed before SC.

37. ARTHUR DEL ROSARIO V. HELLENOR DONATO SECTION 5 JOINDER OF CAUSES OF ACTIONS FACTS: Philip Morris Products, Inc. wrote the National Bureau of Investigation , requesting assistance in curtailing the proliferation of fake Marlboro cigarettes in Angeles City, Pampanga. After doing surveillance work in that city, respondent HellenorDonato, Jr., the NBI agent assigned to the case, succeeded in confirming the storage and sale of such fake cigarettes at the house that belonged to petitioner Alexander del Rosario. Donato applied for a search warrant with Branch 57 of the Regional Trial Court (RTC) of Angeles City to search the subject premises. But it took a week later or on March 12, 2002 for the RTC to hear the application and issue the search warrant. Although Donato felt that the delayed hearing compromised the operation, the NBI agents led by respondent Rafael V. Gonzaga proceeded to implement the warrant. Their search yielded no fake Marlboro cigarettes. Alexander and Arthur del Rosario (the Del Rosarios) filed a complaint for P50 million in damages against respondents NBI agents NBI agents answered the complaint with a motion to dismiss on the grounds of: a) the failure of the complaint to state a cause of action; b) forum shopping; and c) the NBI agents immunity from suit, they being sued as such agents, but it was denied. ISSUES: 1. Whether or not the CA correctly ruled that the complaint of the Del Rosarios did not state a cause of action; and

ISSUE: Whether or not there is a prejudicial question that would justify the suspension of the proceedings n the Criminal Case

HELD: NO. There was no prejudicial question. The fact remains that both the crime charged in the information in the criminal case and the eleventh cause of action in the civil case are based upon fraud, hence both the civil and criminal cases could proceed independently of the other pursuant to Article 33 of the new Civil Code which provides: In cases of defamation, fraud and physical injuries, a civil action for damages, entirely separate and distinct from the criminal action shall proceed independently of the criminal prosecution, and shall require only a preponderance of evidence. (j) That, therefore, the act of

2. Whether or not the CA correctly ruled that the Del Rosarios were guilty of forum shopping. HELD: 1. The CA held that the Del Rosarios complaint before the RTC failed to state a cause of action against respondents NBI agents. Such complaint said that the NBI agents unlawfully procured and enforced the search warrant issued against the Del Rosarios but it failed to state the ultimate facts from which they drew such conclusion. Their allegation that the NBI agents used an unlawfully obtained search warrant is a mere conclusion of law. While a motion to dismiss assumes as true the facts alleged in the complaint, such admission does not extend to conclusions of law. Statements of mere conclusions of law expose the complaint to a motion to dismiss on ground of failure to state a cause of action. The allegation that the search warrant in this case was served in a malicious manner is also not sufficient. Allegations of bad faith, malice, and other related words without ultimate facts to support the same are mere conclusions of law. 2. Rather than file a separate action for damages, the Del Rosarios should have filed their claim for compensation in the same proceeding and with the same court that issued the writ of search and seizure. The Del Rosarios were thus guilty of forum shopping. 38. LILIA B. ADA, LUZ B. ADANZA, FLORA C. BAYLON, REMO BAYLON, JOSE BAYLON, ERIC BAYLON, FLORENTINO BAYLON, and MA. RUBY BAYLONvs.FLORANTE BAYLON SEC 6 MISJOINDER OF CAUSE OF ACTIONS; RULE 10! FACTS: Petitioners filed with the RTC a Complaint for partition, accounting and damages against Florante, Rita and Panfila. They alleged that after the death of Spouses Baylon, Rita took possession of the said parcels of land and appropriated for herself the income from the same and used it to buy 2 parcels of land (4709 and 4706). During the pendency of the case, Rita donated 4709 and half of 4706 to Florante. Learning of the said donation inter vivos in favor of Florante, the petitioners filed a Supplemental Pleadingpraying that the said donation in favor of the respondent be rescinded in accordance with Article 1381(4) of the Civil Code. They further alleged that Rita was already sick and very weak when the donation was made and, thus, could not have validly given her consent

thereto. ISSUE and HELD: 1. Can the actions for partition and rescission be joined in a single action? NO. 2. May a misjoined cause of action, if notsevered upon motion of a party orby the court suasponte, beadjudicated by the court togetherwith the other causes of action? YES. 3. May a supplemental pleading raisea new cause of action? YES, as long as ithas some relation to theoriginalcause of action set forth in theoriginal complaint.

RATIO: 1. By a joinder of actions, or more properly, a joinder of causes of action is meant the uniting of two or more demands or rights of action in one action, the statement of more than one cause of action in a declaration. Nevertheless, while parties to an action may assert in one pleading, in the alternative or otherwise, as many causes of action as they may have against an opposing party, such joinder of causes of action is subject to the condition, inter alia, that the joinder shall not include special civil actions governed by special rules. Here, there was a misjoinder of causes of action. An action for partition is a special civil action governed by Rule 69 of the Rules of Court while an action for rescission is an ordinary civil action governed by the ordinary rules of civil procedure. 2. Misjoinder of causes of action and parties do not involve a question of jurisdiction of the court to hear and proceed with the case. They are not even accepted grounds for dismissal thereof. Instead, under the Rules of Court, the misjoinder of causes of action and parties involve an implied admission of the courts jurisdiction. The courts have the power, acting upon the motion of a party to the case or suasponte, to order the severance of the misjoined cause of action to be proceeded with separately. However, if there is no objection to the improper joinder or the court did not motuproprio direct a severance, then there exists no bar in the simultaneous adjudication of all the erroneously joined causes of action. It should be emphasized that the foregoing rule only applies if the court trying the case has jurisdiction over all of the causes of action therein notwithstanding the misjoinder of the same. If the court trying the case has no jurisdiction over a misjoined cause of action, then such misjoined cause of action has to be severed from the other causes of action, and if

not so severed, any adjudication rendered by the court with respect to the same would be a nullity. 3. See Sec. 6, Rule 10 of Rules of Court. XXX A supplemental pleading may properly allege transactions, occurrences or events which had transpired after the filing of the pleading sought to be supplemented, even if the said supplemental facts constitute another cause of action, and founded on the same cause of action. We further stressed therein that a supplemental pleading maynot be used to try a new cause of action. While a matter stated in a supplemental complaint should have some relation to the cause of action set forth in the original pleading, the fact that the supplemental pleading technically states a new cause of action should not be a bar to its allowance but only a matter that may be considered by the court in the exercise of its discretion. In such cases, we stressed that a broad definition of "cause of action" should be applied. Here,the petitioners prayer for the rescission of the said donation inter vivos in their supplemental pleading is germane to, and is in fact, intertwined with the cause of action in the partition case.The petitioners supplemental pleading merely amplified the original cause of action. Thus, the principal issue raised by the petitioners in their original complaint remained the same.

Writ of Execution drawing petitioner to file a petition for Certiorari and Prohibition with prayer for injunctive relief before the Davao RTC Davao RTC issued a TRO to stay the execution of the MTCC decision Branch 8 of Davao RTC affirmed the MTCC decision and held that since respondent was duly represented the requirement in the LGC was substantially complied with. This prompted petitioner to elevate the case to the CA which issued the challenged decision dismissing without prejudice respondents complaint for unlawful detainer on the ground of lack of cause of action, he having failed to comply with the barangay conciliation proceeding Petitioner filed a motion for reconsideration alleging that during the pendency of the appeal, she was dispossessed from the premises and prays that she be restored thereto. HOWEVER, CA, in noting that the respondents claim was dismissed without prejudice, the petitioners cause of action shall be ventilated in a separate action In this petition for certiorari, petitioner contends that the requirement of filing a separate action runs contrary to the intent of the Rules to provide a just, speedy and inexpensive disposition of cases. ISSUE: IS THE REMEDY OF AN APPEAL AVAILABLE FROM AN ORDER DISMISSING AN APPEAL WITHOUT PREJUDICE?

39. POSITOS VS. CHUA

RULE 2; RULE 16!


Facts: A dispute arose between petitioner, occupant of a parcel of land and respondent, owner of the said land. Conflict was referred for conciliation before the Lupon following the provisions of Act 7160 - LGC. However, respondent did not appear but sent a representative to his behalf. No settlement was reached which prompted respondent to file a complaint against petitioner for unlawful detainer before the MTCC Davao City MTCC rendered a judgment in favor of herein respondent however, petitioner appealed to the RTC of Davao and as she did not file a supersedeas bond to stay the execution of its decision, MTCC issued a

RULING: NO! Section 1, Rule 41 of the Rules of Court provides that the remedy of appeal is not available from an order dismissing an action without prejudice. Aggrieved party instead may file for a special civil action under Rule 65 Present petition prays for the modification of the appellate courts decision. However, this Court cannot treat it as one of certiorari, the allegations of which are not constitutive of GADLEJ 40. Dotmatrix Trading as represented by its proprietors namely Romy Yap Chua, et al. vs. Rommel B. Legaspi, et al. Rule 2 FACTS:

Rommel Legaspi, as the proprietor of Big J Farms and RBL Farm, was the petitioners supplier of day-old chicks from September to December 2001. Respondent sent a demand letter to the petitioners for the payment of delivered day-old chicks. The petitioners, thru petitioner Cadiz, replied that they have paid P1,360,000.00, but the respondent was able to deliver only P1,136,150.00 worth of day-old chicks, leaving a deficiency of P223,850.00 worth of day-old chicks. The petitioners demanded the delivery of the deficiency, or the return of the overpayment made. When the parties refused to comply with each others demands, both went to court for judicial relief. The petitioners (the buyers of the chicks) filed before RTC-Tarlac a complaint for sum of money and damages against the respondent, docketed as Civil Case No. 9354. The petitioners sought the return of the overpayment made, plus moral and exemplary damages, and attorneys fees. The respondent (the seller of the chicks) filed before RTCMalolos, Bulacan a complaint for sum of money and damages against the petitioners, docketed as Civil Case No. 489-M-2002. The respondent alleged that he delivered P1,368,100.00 worth of dayold chicks, but the petitioners only paid P1,150,000.00. Thus, the respondent prayed for the payment of the balance of P218,100.00. Shortly upon receipt of the summons and complaint in Civil Case No. 9354, the respondent filed a motion to dismiss Civil Case No. 9354 before RTC-Tarlac on the ground of litis pendentia because it is merely anticipatory and defensive of the respondents claim for collection in Civil Case No. 489-M-2002 before RTC-Malolos.

ISSUE: Whether or not Civil Case No. 9354 should be dismissed on the ground of litis pendentia? HELD: Yes. Civil Case No. 489-M-2002 is the appropriate case to determine the rights of the parties. RULING:

Facts: S.C. Megaworld Construction and Development Corporation (SC Megaworld) bought electrical lighting materials from Genlite Industries, a sole proprietorship owned by Engr Luis Parada for its Read-Rite project in Canlubang, Laguna. SC Megaworld was unable to pay for the above purchase on due date, but blamed it on its failure to collect under its sub-contract with the EnviroKleen. Engr. Parada then filed a suit in the RTC to collect from the petitioner the said balance, plus damages, costs and expenses. SC Megaworld in its answer denied liability, claiming that it was released from its indebtedness to Engr. Parada by reason of the novation of their contract, which took place when the latter accepted the partial payment of EnviroKleen in its behalf, and thereby acquiesced to the substitution of EnviroKleen as the new debtor in the petitioners place. RTC rendered judgment in favor Engr. Parado. On appeal to the CA, SC Megaworld maintained that the trial court erred in ruling that no novation of the contract took place and for the first time, it further argued that the trial court should have dismissed the complaint for failure of Engr. Parado to implead Genlite Industries as a proper party in interest, as provided in Sec 2 of Rule 3 of the 1997 Rules of Civil Procedure. In Sec 1(g) of Rule 16, it is also provided that the defendant may move to dismiss the suit on the ground that it was not brought in the name of or against the real party in interest, with the effect that the complaint is then deemed to state no cause of action. CA dismissed the appeal noting that at no stage in the proceedings did SC Megaworld raise the question of whether the suit was brought in the name of the real party in interest. Litigants cannot raise an issue for the first time on appeal as this would contravene the basic rules of fair play and justice. Moreover, Engr. Parada is the sole proprietor of Genlite Industries, and therefore the real party-plaintiff. CA concurred with the RTC. On motion for reconsideration, the SC Megaworld raised for the first time the issue of the validity of the verification and certification of non-forum shopping attached to the complaint. CA denied the said motion for lack of merit. Issues: 1.W/N the complaint should have been dismissed outright by the trial court for an invalid non-forum shopping certification. 2. W/N Gentile Industries should have been impleaded as a party plaintiff.

RULE 3 PARTIES

TO

CIVIL ACTIONS

41. S.C. Megaworld v. Engr. Parada G.R. No. 183804, Sept. 11, 2013 SEC 1 WHO MAY BE PARTIES; SECS 4 AND 5 RULE 7

Held: 1. No 2. No Ratio:

1. The verification and certification of non-forum shopping in the complaint is not a jurisdictional but a formal requirement, and any objection as to non-compliance therewith should be raised in the proceedings below and not for the first time on appeal. It is mainly intended to secure an assurance that the allegations therein made are done in good faith or are true and correct and not mere speculation. The Court may order the correction of the pleading, if not verified, or act on the unverified pleading if the attending circumstances are such that a strict compliance with the rule may be dispensed with in order that the ends of justice may be served. Further, in rendering justice, courts have always been, as they ought to be, conscientiously guided by the norm that on the balance, technicalities take a backseat vis--vis substantive rights, and not the other way around It is well-settled that no question will be entertained on appeal unless it has been raised in the proceedings below. Points of law, theories, issues and arguments not brought to the attention of the lower court, administrative agency or quasi-judicial body, need not be considered by a reviewing court, as they cannot be raised for the first time at that late stage. Basic considerations of fairness and due process impel this rule. Any issue raised for the first time on appeal is barred by estoppel. Through a SPA, the Engr. Luis Parada authorized Engr. Leonardo A. Parada (Leonardo), the eldest of his 3 children, to perform the following acts in his behalf: a) to file a complaint against the petitioner for sum of money with damages; and b) to testify in the trial thereof and sign all papers and documents related thereto, with full powers to enter into stipulation and compromise. SC Megaworld reiterates its argument before the CA that the above verification is invalid, since the SPA did not specifically include an authority for Leonardo to sign the verification and certification of nonforum shopping, thus rendering the complaint defective for violation of Secs 4 and 5 of Rule 7. It was previously held that the question of forum shopping cannot be raised in the CA and in the Supreme Court, since such an issue must be raised at the earliest opportunity in a motion to dismiss or a similar pleading. The high court even warned that [i]nvoking it in the later stages of the proceedings or on appeal may result in the dismissal of the action. 2. A sole proprietorship has no juridical personality separate and distinct from that of its owner, and need not be impleaded as a party-plaintiff in a civil case. Sec 1 of Rule 3 provides that only natural or juridical persons or entities authorized by law may be parties in a civil case. Art 44 of the NCC enumerates who are juridical persons:chanrobles virtua1aw 1ibrary Art. 44. The following are juridical persons: (1) The State and its political subdivisions;

(2) Other corporations, institutions and entities for public interest or purpose, created by law; their personality begins as soon as they have been constituted according to law; (3) Corporations, partnerships and associations for private interest or purpose to which the law grants a juridical personality, separate and distinct from that of each shareholder, partner or member. Genlite Industries is merely the DTI-registered trade name or style of Engr. Parada by which he conducted his business. As such, it does not exist as a separate entity apart from its owner, and therefore it has no separate juridical personality to sue or be sued. As the sole proprietor of Genlite Industries, there is no question that Engr. Parada is the real party in interest who stood to be directly benefited or injured by the judgment in the complaint below. There is then no necessity for Genlite Industries to be impleaded as a party-plaintiff, since the complaint was already filed in the name of its proprietor, Engr. Luis U. Parada. To heed the SC Megaworlds sophistic reasoning is to permit a dubious technicality to frustrate the ends of substantial justice.

42. RP v. COALBRINE INTERNATION PHILIPPINES, INC SEC 2 PARTIES-IN-INTEREST FACTS: The Export Processing Zone Authority (EPZA), predecessor of the Philippine Economic Zone Authority, is the owner of the Bataan Hilltop Hotel and country club within the Bataan Export Processing Zone. Dante M. Quindoza is the Zone Administrator of the Bataan economic Zone. In 1994 EPZA, now PEZA, and respondent Coalbrine International Philippines, Inc. entered into a contract in which Coalbrine would rehabilitate and lease the Bataan Hilltop Hotel for 25 years which would commence in 1994 and would be renewable, at the option of coalbrine, for another 25 years. Respondent Sheila Neri was the Managing director of the hotel. In 1996, the PEZA board passed a resolution rescinding the contract to rehabilitate and lease on the ground that Coalbrine committed several violations and non-performance of its obligations as provided in the contract. PEZA then sent a notice to Coalbrine to vacate the premises and pay its outstanding obligations. In 1998 Coalbrine filed with the RTC of Manila a complaint for specific performance with prayer for the issuance of a TRO and/or writ of preliminary injunction with damages against PEZA and/or Bataan Economic Zone wherein Coalbrine sought to declare that PEZA had no valid cause to rescind the contract and

to enjoin PEZA from taking over the hotel and country club and from disconnecting the water and electric services of the hotel. In 2002, respondents Coalbrine and Neri filed with the RTC of Balanga, Bataan, a complaint for damages with prayer for the issuance of a TRO and/or writ of preliminary prohibitory/mandatory injunction against Zone Administrator Quindoza. Coalbrine alleged that in 2001, Quindoza started to harass the hotels operations by causing the excavation of the only road leading to the hotel. This excavation was supposedly for the putting up of a steel pipe however the project came to a halt and therefore this paralyzed the hotels operation. Neri put up a narrow access ramp in order for the hotel guests to pass through however Quindoza had also placed a ROAD CLOSED sign near the hotel. It gave the impression that the hotel, too, was closed. In the last week of March, Quindoza also cut the pipelines that supplied water to the hotel. This caused great inconvenience of respondents and the hotel guests. The pipelines were later on reconnected. Respondents prayed for the payment of damages caused by this disconnection. They also prayed that Quindoza be enjoined from further acts of harassment and that a reasonable access road be constructed at Quindozas expense. Quindoza, through the Sol Gen, filed a motion to dismiss on facts that the court had no jurisdiction, that the venue is improper and that Coalbrine is guilty of forum shopping. With respect to Neri, the latter has no cause of action against the Quindoza and that the complaint is fatally defective for being unauthorized. The motion to dismiss was denied. A motion for reconsideration was filed however was also denied. Quindoza, in his capacity as Zone Administrator filed with the CA a petition for certiorari seeking to annul the RTC orders. This petition was also denied for lack of merit. Petitioner is now filing a petition for review on certiorari raising the lone issue of respondent Neris lack of proof of authority to file the complaint in the RTC of Balanga. Respondents, in their comment, argued that the RP was not a party to the civil case hence it has no personality to file the instant petition for review; the order denying the motion to dismiss was a mere interlocutory order and thus not appealable and not a proper subject of a patition for certiorari.

2.

attached to the complaint filed by respondents in the RTC was defective since Neri had not authority to sign on behalf of the corporation. W/N the RP is a real-party-in-interest

RULING: 1. NO. Neri is not a real party in interest and therefore has no authority to sign the verification and certification. Section 2, Rule 3 of the Rules on Civil Procedure provide that A real party-in-interest is the party who stands to be benefitted or injured by the judgement in the suit, or the party entitled to the avails of the suit. Unless otherwise authorized by law or these rules, every action must be prosecuted or defended in the name of the real party in interest. Interest, within the meaning of the rule means material interest, an interest in issue and to be affected by the decree, as distinguished from mere interest in the question involved, or a mere incidental interest. The RTC erred in ruling that Neri was a proper party in interest. The interests which respondent Neri has as the hotels managing director are merely incidental. Thus, Respondent Neri has no cause of action against petitioner. The plaintiff in this case would only be Coalbrine. Respondent, as a corporation, cannot file a certification or verification and must do so by its duly authorized officers. In this case there was no proof that Neri was authorized to sign the verification and the certification against nonforum shopping. The requirement regarding verification of pleading, however is FORMAL and NOT JURISDICTIONAL. Non-compliance with which does not necessarily render the pleading fatally defective. The court may order the correction of the pleading if verification is lacking or act on the pleading although it is not verified, if the attending circumstances are such that strict compliance with the ruled may be dispensed with in order that the ends of justice may thereby be served. While there have been instances where the court has allowed the filing of a certificate against non-forum shopping by someone on behalf of the corporation without the accompanying proof of authority at the time of filing, the court did so for compelling reasons. Moreover, there was subsequent compliance to the requirement of written authorization. In this case, while Neri testified that she was authorized by the corporate secretary, there was no showing that there was a valid board resolution authorizing the corporate secretary to file the action and to authorize respondent Neri to file the action. In fact, such proof of authority had not been submitted even belatedly to show subsequent compliance. Thus, there was no reason for the relaxation of the rule. 2. YES. The RP is a real party-in-interest

ISSUES: 1. (Relevant issues) W/N respondent Neris signature in the verification and certification against non-forum shopping

As to respondents claim that petitioner Republic of the Philippines was not a party to the civil case subject of this petition since Administrator Quindoza was the sole defendant therein and, thus, has no personality to file this petition, their claim is not persuasive. Administrator Quindoza was sued in his official capacity as Zone Administrator of the Bataan Export Processing Zone. Therefore, the complaint is in the nature of suit against the State and thus the Republic has the personality to file the petition. Petition is GRANTED 43. NEMESIO GOCO V. CA, CATLY Municipalitys lessee Syllabus: Parties; Cause of Action; One having no material interest to protect cannot invoke the jursidiction of the court as the plaintiff in an action, in which case, it is dismissible on the ground of lack of cause of action. FACTS: In 1952, Municipality of calapan acquired a interest over Lot 2042 in a suit against Alveyra however the entire Lot 2042 was included in only one TCT thus an action to quiet title was instituted to determine the extent of the Municipalitys interest. During the pendency of the case, Alveyra sold of their interest over the land to Respondents Catly. The lot was divided into LOT 2042-A for Alveyra and LOT 2042-B for the municipality Catly found out that the Petitioners Goco were occupying a portion of the said Lot thus they filed an ejectment case against them but it was dismissed. After the dismissal, Catly now filed an action for recovery of possession against petitioners. Allegedly to defend themselves against the cases filed, Gocofiled a complaint for declaration of nullity of the TCTs issued in favor of Catlys names. They claim that they are the occupant s of Lot 2042 since 1946 as lessees of the Municipality and that the inclusion of thelot they are leasing in Catlys Lot was prejudicial to their interest as actual occupants. ISSUE:WHETHER OR NOT Gocos are real parties-in-interest to assail and seek the annulment of Catlys Title. NO RATIO: Under Sec 2 Rule 3 there are two requisites that must be followed these are: (1) to institute an action, the plaintiff must be the real party in interest; and (2) the action must be prosecuted in the name of the real party in interest.Interest means material interest or an interest in issue to be affected by decree or judgment of the case as distinguished from mere curiousity about the question involved.Having no material interest, the case may be dismissed for lack of cause of action.

An action for annulment of CTCs must be instituted by the real party-in-interest who is the personclaimingtitle or ownershipadversetothatof the registered ownerin this case the Municipality. The interest of the party must be personal and not one based on a desire to vindicate the constitutional right of some third and unrelated party. In this case, Gocos were only lessees and not the real owners of the lot therefore they are not parties with material interest to commence such action to nullify the TCTs. 44. ROGER NAVARRO V. HON JOSE ESCOBIDA G.R. NO. 153788 FEBRUARY 10, 2010 SEC 2; SEC 4 SPOUSES AS PARTIES FACTS Navarro entered two lease contracts with Kargo Enterprises, a sole proprietorship, represented by its manager Glenn Go to rent two motor vehicles. The parties agreed that Navarro would deliver postdated checks to Kargo Enterprise as payment for the monthly rentals. When three of these postdated checks were dishonored, Kargo Enterprises made demands on Navarro to either pay the rentals due or return the leased motor vehicles. When Navarro neither paid nor returned the leased vehicles, Karen Go as the sole proprietor of Kargo Enterprises, filed two complaints for replevin and/or sum of money against Navarro. ISSUES 1. WON Karen is not the real party-in-interest to file the complaints against him. 2. WON the complaints for replevin must, on their face, show that demand on the lessee was first made since the lease contracts make demand a prerequisite before a lessee can be compelled to return the leased vehicles 3. WON the Court erred in ruling that Kargo Enterprises is conjugal property, as this was not established by the facts alleged in the complaints. 4. WON the lower court gravely abused its discretion in issuing the order directing Karen to amend her complaints to include Glenn as coplaintiff because according to Navarro, this was tantamount to the court a quo curing Karen's lack of capacity to sue. HELD The court finds no merit in such contentions.

45. BPI v. LEE RATIO 1. On their face, the lease contracts expressly provide that Glenn entered these contracts representing Kargo Enterprises as its Manager. In other words, Glenn signed the contract merely as Kargo Enterprises's agent. Since Kargo Enterprises is a sole proprietorship, it has no identity of its own and relies completely on the identity of its owner, Karen. Simply put, Karen Go is Kargo Enterprises, and is therefore the best party to file the replevin complaints against Navarro for the motor vehicles he leased from Kargo Enterprises. 2. We cannot understand Navarro's issue with prior notice and demand. To begin with, Karen clearly and specifically alleged in her complaint that "demands, written and oral, were made on defendant ROGER NAVARRO to pay the amount of xxx or to return the subject motor vehicle as also provided for in the LEASE AGREEMENT WITH OPTION TO PURCHASE, but said demands were, and still are, in vain to the great damage and injury of herein plaintiff". Secondly, Navarro already admitted that he received the demand letter sent by Karen's counsel before the replevin complaints were filed. As for Article 1669 of the Civil Code, which Navarro cites in support of his claim that prior notice and demand are a condition precedent to the filing of a replevin action, this provision clearly cannot apply to this case because it falls under the Chapter on Lease of Rural and Urban Lands. 3. Art. 116 of the Family Code provides that "all property acquired during the marriage, whether the acquisition appears to have been made, contracted or registered in the name of one or both spouses, is presumed to be conjugal unless the contrary is proved." In determining whether this legal presumption applies or not, the essential factor to consider is when the trade name was acquired. In the present case, the trade name Kargo Enterprises was registered in Karen's married name, signifying that the trade name was acquired during Karen and Glenn's marriage. Kargo Enterprises, as a trade name and as a business concern, is therefore presumed to be conjugal property. As Navarro did not adduce any proof that Kargo Enterprises was acquired solely by Karen's efforts, it is deemed that all assets carried under this name are conjugal property. 4. Glenn is not indispensable to the complaint but a merepro-forma party. Even if Glenn is an indispensable party, his non-inclusion in the complaints is not a ground to dismiss the action. As we held in Commissioner Domingo v. Scheer parties may be added by order of the court on motion of the party or on its own initiative at any stage of the action and/or such times as are just. SEC 2 PARTIES-IN-INTEREST VIRTUAL PARTY; Rule 19 Intervenor; Rule 39 FACTS: Carlito Lee filed a complaint for sum of money with damages and application for the issuance of a writ of attachment against Trendline and Buelva seeking to recover his total investment in the amount of P5.8 million. Lee alleged that he was enticed to invest his money with Trendline upon Buelvas misrepresentation that she was its duly licensed investment consultant or commodity saleswoman RTC issued a writ of preliminary attachment whereby the Check-O-Matic Savings Accounts of Trendline with Citytrust Banking Corporation, Ayala Branch, in the total amount of P700,962.10 were garnished. Subsequently, the RTC rendered a decision finding defendants jointly and severally liable to Lee for the full amount of his investment plus legal interest, attorneys fees and costs of suit Citytrust filed before the RTC an Urgent Motion and Manifestation5 seeking a ruling on defendants' request to release the amount of P591,748.99 out of the garnished amount for the purpose of paying Trendlines tax obligations. Citytrust and BPI merged, with the latter as the surviving corporation. The Articles of Merger provide, among others, that all liabilities and obligations of Citytrust shall be transferred to and become the liabilities and obligations of BPI in the same manner as if the BPI had itself incurred such liabilities or obligations. BPI Manager Samuel Mendoza, Jr. denied having possession, control and custody of any deposits or properties belonging to defendants, prompting Lee to seek the production of their records of accounts with BPI. It found BPI liable to deliver to the RTC the garnished bank deposit of Trendline Neither did it consider BPI a stranger to the case, holding it to have become a party- in-interest upon the approval by the Securities and Exchange Commission (SEC) of the parties Articles of Merger.

ISSUE: Wheter or not Bpi is a party-in-interest HELD: Yes. Upon the merger of Citytrust and BPI, with the latter as the surviving corporation, and with all the liabilities and obligations of Citytrust transferred to BPI as if it had incurred the same, BPI undoubtedly became a party interested in sustaining the proceedings, as it stands to be prejudiced by the outcome of the case. It is a settled rule that upon service of the writ of garnishment, the garnishee becomes a virtual party or forced intervenor to the case and the trial court thereby acquires jurisdiction to bind the garnishee to comply with its orders and processes. Citytrust, therefore, upon service of the notice of garnishment and its acknowledgment that it was in possession of defendants' deposit accounts in its letter-reply dated June 28, 1988, became a virtual party to or a forced intervenor in the civil case. By virtue of its merger with BPI, BPI, as the surviving corporation, effectively became the garnishee, thus the virtual party to the civil case. 46. JUANITO VICTOR C. REMULLA, Petitioner, v.ERINEO S. MALIKSI, etc. et.al. LOCUS STANDI (Compare with SEC 2 PARTIES-IN-INTEREST) FACTS: Marietta OHara de Villa (de Villa), in her personal capacity and as administratix of the estate of her late husband Guillermo, ceded, through a deed of donation(1957 deed of donation), 134,957 square meters (sq. m.) (donatedportion) of their 396,622 sq. m. property (subject property) in favor of the Province of Cavite, on which now stands various government offices and facilities. The Province of Cavite respectively filed a Complaintseeking to expropriate, for the amount of P215, 050.00, the remaining 261,665 sq. m. of the subject property which the former intends to develop as the Provincial Capitol Site. De Villa, through her Answer, opposed the expropriation proceedings, claiming that there are still areas within the

donated portion, which the Province of Cavite failed to develop.She also alleged that the fair market value of the subject property should be pegged at the amount of P11, 272,500.00, or at P45.00 per sq. m. While the expropriation case was still pending, de Villa sold, for the amount of P2, 000,000.00,the 261,665 sq. m. portion of the subject property to Goldenrod, Inc., a joint venture company owned by Sonya G. Mathay and Eleuterio M. Pascual, Jr. Then Cavite Governor ErineoS.Maliksiissued Executive Order No. 004authorizing the creation of a committee which recommended the terms and conditions for the proper settlement of the expropriation case. The said committee thereafter submitted its Committee Reportrecommending that: (a) the just compensation be pegged at the amount of P495.00 per sq. m. plus 6% annual interest for 22 years,for a total net consideration of P50,000,000.00, which amount shall be equally shouldered by the Province of Cavite and TreceMartires City; Compromise agreement was entered into by Maliksi, TreceMartires City Mayor De Sagun and Mathay and Pascual. Remulla, in his personal capacity as taxpayer and as then ViceGovernor and, hence, Presiding Officer of the SangguniangPanlalawigan of the Province of Cavite,filed a petition for annulment of judgmentunder Rule 47 of the Rules of Court before the CA, arguing that the subject compromise is grossly disadvantageous to the government because: (a) the agreed price for the subject property was excessive as compared to its value at the time of taking in 1981;(b) the government stands to lose prime lots; and (c) it nullifies/amends the 1957 deed of donation.Moreover, Maliksi entered into the subject compromise without authority from the SangguniangPanlalawigan of the Province of Cavite and sans any certification on the availability of funds as required by law. Remulla claimed that extrinsic fraud tainted the expropriation proceedings considering that there was collusion between the parties and that respondent Ignacio deliberately withheld crucial information regarding the property valuation and certain incidents prior to the expropriation case when he presented the subject compromise for ratification before the SangguniangPanlalawigan of the Province of Cavite.

ISSUE/S: 1. Whether or not Remulla had legal standing to seek a petition for annulment of compromise agreement HELD: 1. Remulla filed his petition for annulment of judgment in two capacities: first, in his personal capacity as a taxpayer; and,

second, in his official capacity as then presiding officer of the SangguniangPanlalawigan of the Province of Cavite. RATIO: 1. A taxpayer may be allowed to sue where there is a claim that public funds are illegally disbursed or that public money is being deflected to any improper purpose, or that public funds are wasted through the enforcement of an invalid or unconstitutional law or ordinance.34 In this case, public funds of the Province of Cavite stand to be expended to enforce the compromise judgment. As such, Remulla being a resident-taxpayer of the Province of Cavite has the legal standing to file the petition for annulment of judgment and, therefore, the same should not have been dismissed on said ground. A taxpayer need not be a party to the contract in order to challenge its validity, or to seek the annulment of the same on the ground of extrinsic fraud. Indeed, for as long as taxes are involved, the people have a right to question contracts entered into by the government. 2. Remulla equally lodged the petition for annulment of judgment in his official capacity as then Vice-Governor and Presiding Officer of the SangguniangPanlalawigan of the Province of Cavite. As such, he represents the interests of the province itself which is, undoubtedly, a real party in interest since it stands to be either benefited or injured40 by the execution of the compromise judgment.

Issues: 1. W/N the complaint must be dismissed on the ground that venue was not properly laid. 2. W/N Atty. Aceron is a real party in interest, such that he is considered a plaintiff for purposes of determining the venue. Held: 1. Yes. The petitioners complaintshould have been filed in the RTC of Bacolod City, thecourt of the place where therespondents reside, and notin RTC of Quezon City. 2. No. Atty. Aceron is not a real party in interest in the case below; thus, his residence is immaterial to the venue of the filing ofthe complaint. Ratio: 1. The petitioners complaint for collection of sum of money against the respondents is a personal action as it primarily seeks the enforcement of a contract. The Rules give the plaintiff the option of choosing whether to file it in the place (1) where he himself or any of them resides, or (2) where the defendant or any of the defendants resides or may be found. However, if the plaintiff does not reside in the Philippines, the complaint in such case may only be filed in the court of the place where the defendant resides. 2. A real party in interest is the party who, by the substantive law, has the right sought to be enforced. It is clear that Atty. Aceron is not a real party in interest in the case below as he does not stand to be benefited or injured by any judgment therein. He was merely appointed by the petitioners as their attorney-in-fact for the limited purpose of filing and prosecuting the complaint against the respondents. Such appointment, however, does not mean that he is subrogated into the rights of petitioners and ought to be considered as a real party in interest. 48. ATTY. FE Q. PALMIANO-SALVADOR v. ANGELES, substituted by LUZ G. ANGELES CONSTANTINO

47. THEODORE AND NANCY ANG V. SPS. ALAN AND EMANG REAL PARTY-IN-INTEREST AND SEC 3 REPRESENTATIVE AS PARTIES; VENUE! Facts: In Sept. 1992, respondent spouses Alan and EmAng obtained a loan from petitioners Nancy and Theodore Ang, amounting to USD 300,000, evidenced by a promissory note subject to interest rate of 10% p.a. and payable upon demand. Despite repeated demands, respondents were unable to pay. As of August 28, 2006, the debt, inclusive of the interest, reached USD 719,671.23. The petitioners reside in USA, while the respondents reside in Bacolod. Petitioners appointed Atty. Aceron to represent them in filing a Complaint for collection of sum of money, which the lawyer filed with the RTC of Quezon City, as he is a resident of Quezon City.

SEC 3 REPRESENTATIVE AS PARTIES EFFECT OF COMPLAINT FILED BY ONE WITHOUT AUTHORITY TO DO SO FACTS: Respondent Angeles is one of the registered owners of a parcel of land. It is occupied by Jelly Galiga as lessee. Petitioner Salvador alleged that she bought the land from Galiga who represented that he was the owner. Hence, Salvador remaind in possession of said property up to the present.

Angeles made a formal demand to Salvador to vacate, who refused. Thus Angeles, through Rosario Diaz, filed a complaint for ejectment with the MeTC. The MeTC rendered its decision in favour of Angeles. Salvador appealed, alleging, among others, that Diaz, who filed for the complaint, had no authority from Angeles at the time of filing of the suit. This was denied by the RTC, and again by the CA.

proceedings before it were null and void. Legally, there was no complaint to speak of, and the courts jurisdiction cannot be deemed to have been invoked at all.

ISSUE/S: Whether or not Diaz had authority from respondent Angeles to file the complaint HELD: NO RATIO: The SC pointed out that this basic issue was ignored by the MeTC and the RTC, while the CA absolutely failed to address it, despite petitioners insistence on it from the very beginning, i.e. in her Answer filed with the MeTC. The complaint before the MeTC was filed in the name of respondent, but it was Diaz who executed the verification and certification, alleging he was the respondents attorney-in-fact. There was, however, no copy of any document attached to the complaint to prove Diazs authorit y. More than a year after the complaint was filed, respondent attached a document entitled Special Power of Attorney supposedly executed by respondent. However, this SPA was executed more than a month after the complaint was filed, notarized by a Robert McGuire from California. There was no certification by the Philippine Consulate General in California, USA, that said person is indeed a notary public of California. Thus, the SC cannot give full faith and credit to the official acts of McGuire, and hence, no evidentiary weight or value can be attached to the document designated as SPA. There is nothing then on record to show that Diaz had been authorized by respondent to initiate the action against petitioner. The SC has previously ruled that if a complaint is filed for and in behalf of the plaintiff by one who is not authorized to do so, the complaint is not deemed filed. An unauthorized complaint does not produce any legal effect. Hence, the Court should dismiss the complaint on the ground that it has no jurisdiction over the complaint and the plaintiff. Courts acquire jurisdiction over the plaintiff upon filing of the complaint, and to be bound by a decision, a party should first be subjected to the courts jurisdiction. Clearly, since no valid complaint was ever filed with the MeTC, the same did not acquire jurisdiction over the person of respondent. All 49. REGNER V. LOGARTA SEC 7 INDISPENSABLE PARTIES FACTS: Luis Regner had 3 daughters with his 1st wife Anicita: Cynthia, Teresa (the respondents) and Melinda. The petitioner is the 2nd wife Victoria. During the lifetime of Luis, he acquired several properties, among which is a share at Cebu Country Club Inc. Luis extended a Deed of Donation in favor of Teresa and Cynthia of the share at the Cebu Country Club. Luis then passed away after a year. Victoria alleged that Cynthia and Teresa with the help of another sibling defrauded Luis, who was then very ill and was unable to write, into placing his thumbmark into a Deed of Donation. In said Deed, Luis purportedly donated a Proprietary Ownership Certificate pertaining to membership shares in the Cebu Country Club. Victoria alleged that said Deed is void because the placing of thumbmark by Luis was done without the latters free will and voluntariness considering his physical state; that it was done without Luiss lawyer; that the ratification made by Luis before he died is likewise void because of similar circumstances. The sheriff could not deliver the summonses against Cynthia and Teresa because although they are Filipinos, they are not residing here; they are residing in California. It was only in the year 2000 that one of the summonses was served to one of the sisters, Teresa, when she came back to the Philippines. Teresa immediately filed a motion to dismiss on the ground that Victoria failed to prosecute her case for an unreasonable length of time. Victoria opposed the MTD. Teresa, in her rejoinder, alleged that the case should be dismissed because Cynthia, who is an indispensable party, was not issued any summons, hence, since an indispensable party is not served with summons, without her who has such an interest in the controversy or subject matter there can be no proper determination of the case. The trial court ruled in favor of Teresa; this was affirmed by the Court of Appeals. ISSUE:

1. WON a co-donee is an indispensable party in an action to declare the nullity of the deed of donation 2. WON delay in the service of summons upon one of the defendants constitutes failure to prosecute that would warrant dismissal of the complaint. HELD: 1. YES. 2. YES. RATIO: 1. A Court must acquire jurisdiction over the persons of indispensable parties before it can validly pronounce judgments personal to the parties. Courts acquire jurisdiction over a party plaintiff upon the filing of the complaint. Jurisdiction over the person of a party defendant is assured upon the service of summons in the manner required by law or otherwise by his voluntary appearance. As a rule, if a defendant has not been summoned, the court acquires no jurisdiction over his person, and a personal judgment rendered against such defendant is null and void. A decision that is null and void for want of jurisdiction on the part of the trial court is not a decision in the contemplation of law and, hence, it can never become final and executory. 2. Rule 3, Section 7 of the Rules of Court, defines indispensable parties as parties-in-interest without whom there can be no final determination of an action. As such, they must be joined either as plaintiffs or as defendants. 3. The general rule with reference to the making of parties in a civil action requires the joinder of all necessary parties where possible, and the joinder of all indispensable parties under any and all conditions, their presence being a sine qua non for the exercise of judicial power. It is precisely when an indispensable party is not before the court that the action should be dismissed. 4. The purpose of this provision was to prevent multiplicity of suits by requiring the person asserting a right against the defendant to include with him, either as co-plaintiffs or as co-defendants, all persons standing in the same position, so that the whole matter in dispute may be determined once and for all in one litigation. 5. Cynthia is an indispensable party in Civil Case No. CEB 23927 without whom the lower court is barred from making a final adjudication as to the validity of the entire donation. Without the presence of indispensable parties to a suit or proceeding, a judgment therein cannot attain finality. The trial court must also acquire jurisdiction over Cynthias person through the proper service of summons. 6. In the case at bar, Cynthia was never served any summons in any of the manners authorized by the Rules of Court. The summons served to Teresa cannot bind Cynthia. It is incumbent upon Victoria to

compel the court to authorize the extraterritorial service of summons against Cynthia. Her failure to do so for a long period of time constitutes a failure to prosecute on her part. To allow petitioner to wait until such time that summonses were served on respondents would frustrate the protection against unreasonable delay in the prosecution of cases and violate the constitutional mandate of speedy dispensation of justice which would in time erode the peoples confidence in the judiciary. We take a dim vi ew of petitioners complacent attitude and sustain the dismissal by the trial court of the petitioners complaint for failure to prosecute for a period of more than one year. 50. JOSEPHINE MARMO V. MOISES ANACAY SEC 7 INDISPENSABLE PARTIES FACTS: Anacay and his wife authorized petitioner Josephine to sell a 50-square meter parcel of land and the house built thereon; petitioner Josephine sold the subject property to petitioner Danilo for P520,000.00, payable in monthly installments of P8,667.00 from May 2001 to June 2006; petitioner Danilo defaulted in his installment payments from December 2002 onwards; the respondent subsequently discovered that his title had been cancelled and a new title was issued in petitioner Josephines name by virtue of a falsified Deed of Absolute Sale; petitioner Josephine subsequently transferred her title to petitioner Danilo; her title was cancelled and a new one was issued in petitioner Danilos name. Thus, Moises O. Anacay filed a case for Annulment of Sale, Recovery of Title with Damages against the petitioners and the Register of Deeds of the Province of Cavite. In her Answer, petitioner Josephine averred, among others, that the respondents children, as co-owners of the subject property, should have been included as plaintiffs because they are indispensable parties. Following the pre-trial conference, the petitioners filed a Motion to Dismiss the case for the respondents failure to include his children as indispensable parties. The RTC denied the petitioners motion to dismiss. On appeal, CA dismissed the petition. The petitioners moved but failed to secure a reconsideration of the CA Decision; hence, the present petition. Issue Whether or not the respondents children are indispensable parties Held No.Respondents children, as co-owners of the subject property, are not indispensable parties to the resolution of the case. Ratio Section 7, Rule 3 of the Revised Rules of Court defines indispensable parties as parties-in-interest without whom there can be no final determination of an action and who, for this reason, must be joined either as plaintiffs or as defendants.Jurisprudence further holds that a

party is indispensable, not only if he has an interest in the subject matter of the controversy, but also if his interest is such that a final decree cannot be made without affecting this interest. He is a person whose absence disallows the court from making an effective, complete, or equitable determination of the controversy between or among the contending parties. When the controversy involves a property held in common, Article 487 of the Civil Code explicitly provides that "any one of the co-owners may bring an action in ejectment." Where the suit is brought without repudiating the co-ownership, then the suit is presumed to be filed for the benefit of the other co-owners. However, where the co-owner repudiates the co-ownership by claiming sole ownership of the property, his co-owners are indispensable parties and must be impleaded as partydefendants. In the present case, the respondent, as the plaintiff in the court below, never disputed the existence of a co-ownership nor claimed to be the sole or exclusive owner of the litigated lot. Moreover and more importantly, the respondents claim in his complaint is personal to him and his wife, i.e., that his and his wifes signatures in the Deed of Absolute Sale in favor of petitioner Josephine were falsified.The issue therefore is falsification, an issue which does not require the participation of the respondents co-owners at the trial; it can be determined without their presence because they are not parties to the document. 51. ANICIA VALDEZ-TALLORIN v. HEIRS OF JUANITO TARONA SEC 7 INDISPENSABLE PARTIES FACTS: Respondents Carlos, Rogelio, and Lourdes Tarona (Taronas) filed an action before the RTC of Balanga, Bataan, against petitioner Anicia Valdez-Tallorin for the cancellation of her and two other womens tax declaration over a parcel of land. Taronas alleged that, unknown to them, the Assessors Office of Morong cancelled Tax Declaration 463 in the name of their father, Juanito Tarona, covering 6,186 square meters of land in Morong. The cancellation was said to be based on an unsigned though notarized affidavit that Juanito allegedly executed in favor of petitioner Tallorin and two others, namely, Margarita Pastelero Vda. de Valdez and Dolores Valdez, who were not impleaded in the action. In place of the cancelled one, the Assessors Office issued Tax Declaration 6164 in the names of the latter three persons. Juanitos affidavit had been missing and no copy could be found among the records of the Assessors Office. The Taronas alleged that, without their fathers affidavit on file, it followed that his tax declaration had been illegally cancelled and a new one illegally issued in favor of Tallorin; and that the others with her and the unexplained disappearance of the affidavit from official files, coveredup the falsification or forgery that caused the substitution.

The Taronas asked the RTC to annul Tax Declaration 6164, reinstate Tax Declaration 463, and issue a new one in the name of Juanitos heirs. The RTC rendered judgment: a) annulling the tax declaration in the names of Tallorin, Margarita Pastelero Vda. de Valdez, and Dolores Valdez; b) reinstating the tax declaration in the name of Juanito; and c) ordering the issuance in its place of a new tax declaration in the names of Juanitos heirs. The trial court also ruled that Juanitos affidavit authorizing the transfer of the tax declaration had no binding force since he did not sign it. Tallorin appealed the above decision to the CA: 1) that the land covered by the tax declaration in question was titled in her name and in those of her two co-owners; 2) that Juanitos affidavit only dealt with the surrender of his tenancy rights and did not serve as basis for canceling Tax Declaration 463 in his name; 3) that, although Juanito did not sign the affidavit, he thumbmarked and acknowledged the same before a notary public; and 4) that the trial court erred in not dismissing the complaint for failure to implead Margarita Pastelero Vda. de Valdez and Dolores Valdez who were indispensable parties in the action to annul Juanitos affidavit and the tax declaration in their favor. The CA affirmed the trial courts decision and rejected all of Tallorins arguments. Since she did not assign as error the order declaring her in default and since she took no part at the trial, the CA pointed out that her claims were in effect mere conjectures, not based on evidence of record. The CA did not address the issue Tallorin raised regarding the Taronas failure to implead Margarita Pastelero Vda. de Valdez and Dolores Valdez as indispensable party-defendants, their interest in the cancelled tax declarations having been affected by the RTC judgment ISSUE: Whether or not the CA erred in failing to dismiss the Taronas complaint for not impleading Margarita Pastelero Vda. de Valdez and Dolores Valdez in whose names, like their co-owner Tallorin, the annulled tax declaration had been issued. HELD: Yes RATIO: The rules mandate the joinder of indispensable parties. Sec. 7. Compulsory joinder of indispensable parties. Parties in interest without whom no final determination can be had of an action shall be joined either as plaintiffs and defendants. Indispensable parties are those with such an interest in the controversy that a final decree would

necessarily affect their rights, so that the courts cannot proceed without their presence. Joining indispensable parties into an action is mandatory, being a requirement of due process. Without their presence, the judgment of the court cannot attain real finality. Judgments do not bind strangers to the suit. The absence of an indispensable party renders all subsequent actions of the court null and void. Indeed, it would have no authority to act, not only as to the absent party, but as to those present as well. And where does the responsibility for impleading all indispensable parties lie? It lies in the plaintiff. 52. JOANIE SURPOSA UY vs. JOSE NGO CHUA RULE 3 FACTS: Jose, who was then married, had an illicit relationship with Irene Surposa then had two children, namely, Joanie and her brother, Allan. Jose attended to Irene when the latter was giving birth to Joanie on, and instructed that petitioners birth certificate be filled out with the following names: "ALFREDO F. SURPOSA" as father and "IRENE DUCAY" as mother. Actually, Alfredo F. Surposa was the name of Irenes father, and Ducay was the maiden surname of Irenes mother. Jose financially supported petitioner and Allan, and had consistently and regularly given Joanie allowances before she got married. He also provided her with employment and when petitioner was still in high school, respondent required her to work at the Cebu Liberty Lumber, a firm owned by his family. She was later on able to work at the Gaisano-Borromeo Branch through Joses efforts. Joanie and Allan were introduced to each other and became known in the Chinese community as respondents illegitimate children. Jose denied that he had an illicit relationship with Irene, and that petitioner was his daughter. The parties eventually entered into a Compromise Agreement in a special proceeding, which was approved by RTC in a decision dated 21 February 2000. JOANIE SURPOSA UY declares, admits and acknowledges that there is no blood relationship or filiation between petitioner and her brother Allan on one hand and JOSE NGO CHUA on the other. As a gesture of goodwill and by way of settling Joanie and her brothers civil, monetary and similar claims but without admitting any liability, JOSE NGO CHUA hereby binds himself to pay the petitioner the sum of P2,000,000.00 and another P2,000,000.00 to her brother, ALLAN SURPOSA.

Joanie filed on 15 April 2008 her Opposition to Joses Demurrer to Evidence in a special proceeding. Thereafter, RTC-Branch 24 issued its now assailed Resolution dated 25 June 2008 in special proceeding, granting respondents Demurrer. ISSUE: Whether or not the compromise agreement entered into by the parties herein before the Regional Trial Court, Branch 09 of Cebu City effectively bars the filing of the case. HELD: NO RULING: A compromise is a contract whereby the parties, by making reciprocal concessions, avoid a litigation or put an end to one already commenced. In Estate of the late Jesus S. Yujuico v. Republic, the Court pronounced that a judicial compromise has the effect of res judicata. A judgment based on a compromise agreement is a judgment on the merits. Any compromise agreement that is contrary to law or public policy is null and void, and vests no rights in and holds no obligation for any party. It produces no legal effect at all. The Compromise Agreement between petitioner and respondent is obviously intended to settle the question of petitioners status and filiation, and is covered by the prohibition under Article 2035 of the Civil Code. In sum, the special proceeding before RTC-Branch 24 is not barred by res judicata, since RTC-Branch 9 had no jurisdiction to approve, in its decision in a special proceeding since the compromise agreement, which was contrary to law and public policy. Nevertheless, the Court must clarify that even though the Compromise Agreement between petitioner and respondent is void for being contrary to law and public policy, the admission which Joanie made therein may still be appreciated against her. RTC-Branch 24 is only reminded that while petitioners admission may have evidentiary value, it does not, by itself, conclusively establish the lack of filiation.

53. HADJI PANGSAYAN T. ABDULRAHMAN vs. THE OFFICE OF THE OMBUDSMAN FOR MINDANAO AND GUIAMALUDIN A. SENDAD G.R. No. 175977, August 19, 2013

SECTION 11 MISJOINDER OF PARTIES DCOTRINE: Neither the misjoinder nor the non-joinder of parties is a ground for the dismissal of an action. FACTS: Petitioner was a Land Management Inspector of the Community Environment and Natural Resources Office (CENRO) of Kalamansig, Sultan Kudarat. Private respondent reported the alleged illegal activities of petitioner who solicited from him the total amount of PP5,450 as consideration for the titling in private respondents name of lands covered by the homestead applications. In a Resolution dated 14 March 1995, the Ombudsman recommended the dismissal of petitioner, Sayutin, and Alcoriza from service. It found Sayutin and Alcoriza guilty of gross neglect of duty and petitioner of grave misconduct. On appeal, the CA dismissed the petition citing among others the petitioners failure to implead private respondent. ISSUE: Whether the CA correctly dismissed the case on the ground that the petitioner failed to implead private respondent HELD: No. In this case, it was an error for the CA to dismiss the petition for failure to comply with Section 5, Rule 65 of the Rules of Court, which states:

an action. If it was truly necessary to implead Guiamaludin Sendad, what the CA should have done was to order petitioner to add him as private respondent to the case.

54. LITTIE SARAH AGDEPPA V. HEIRS OF BONETE SECTION 11 MISJOINDER OF PARTIES FACTS: Dorotea Bonete, widow of Ignacio and mother of five obtained a loan from DBP Cotabato in the amount of P55,000 in order to buy farm implements. A parcel of agricultural land, in the name of Dorotea, was used as collateral to secure the said loan. Respondents received a notice of collection from DBP. Counsel Atty. Littie Sarah Agdeppa accompanied Dorotea to BDP and obligated herself to pay the loan. Dorotea was allegedly made to sign a document as Littie Sarahs security for the amount which the latter paid to DBP. Further, respondents alleged that Agdeppa and her representatives had been gradually easing them out of the land and ordered to stop the cultivation of their ricefields. Eventually, they were forcibly ejected. Further, Agdeppa planted corn and put up duck-raising projects on the subject property. Respondents found out from the RD that the title of the land had already been cancelled and transferred to Agdeppas name. Dorotea said that Agdeppa took advantage of her by letting her sign a contract which Agdepa said was a security but turned out to be a deed of sale. Thus, respondents filed a Complaint for Recovery of Ownership and Possession and/or Annulment of Deed of Sale of the Subject Property with Damages before the RTC. Agdeppa filed a Motion to Dismiss the Complaint based on the ff: 1) that respondents had no legal capacity to sue; 2) that respondents were not the real parties in interest; 3) that the Complaint stated no cause of action; and 4) that the claim or demand set forth in the Complaint had already been waived and extinguished. RTC dismissed the Complaint, holding that the Complaint did not show the character and representation that respondents claimed to have. The title of the land was not in the name of the late Ignacio Bonete but in Dorotea's name. It held that respondents were not real parties in interest. It further held that respondents lacked the personality to sue; thus, a valid basis to grant the motion to dismiss on the ground that the complaint did not state a cause of action.

Section 5. Respondents and costs in certain cases. When the petition filed relates to the acts or omissions of a judge, court, quasi-judicial agency, tribunal, corporation, board, officer or person, the petitioner shall join, as private respondent or respondents with such public respondent or respondents, the person or persons interested in sustaining the proceedings in the court; and it shall be the duty of such private respondents to appear and defend, both in his or their own behalf and in behalf of the public respondent or respondents affected by the proceedings, and the costs awarded in such proceedings in favor of the petitioner shall be against the private respondents only, and not against the judge, court, quasi-judicial agency, tribunal, corporation, board, officer or person impleaded as public respondent or respondents. (Emphasis supplied) Section 11, Rule 3 of the Rules of Court, states that neither the misjoinder nor the non-joinder of parties is a ground for the dismissal of

CA reversed and set aside the RTC Order, and remanded the case to the RTC for further proceedings because Dorotea, being the former owner of the subject property, was a real party in interest.

ISSUE: W/N Dorotea was a real party in interest. HELD: Yes. RATIO: Misjoinder of parties does not warrant the dismissal of the action (Rule 3, Section 11 of the Rules of Court). It bears stressing that the title covering the land was issued in the name of Dorotea. This is established by the record, and petitioners themselves admit this fact. However the title allegedly issued in favor of Agdeppa, and the purported deed of sale, allegedly executed by Dorotea in favor of Littie Sarah, are not on record. Considering the allegations in the pleadings, it is best that a trial on the merits be conducted. The Court fully agrees with the CA ruling. Dorotea as former owner of the land, she is a real party in interest and has the legal capacity to file the instant case for reconveyance and annulment of deed of sale. Why the property became the subject of the deed of sale which is being disputed by Dorotea should be threshed out in a full-blown trial on the merits in order to afford the contending parties their respective days incourt. As held in a case, the complaint is not supposed to contain evidentiary matters as this will have to be done at the trial on the merits of the case. A liberal construction of the Rules is apt in situations involving excusable formal errors in a pleading, as long as the same do not subvert the essence of the proceeding, and they connote at least a reasonable attempt at compliance with the Rules. The Court is not precluded from rectifying errors of judgment, if blind and stubborn adherence to procedure would result in the sacrifice of substantial justice for technicality. To deprive respondents, particularly Dorotea, of their claims over the subject property on the strength of sheer technicality would be a travesty of justice and equity.

55. RIOFERIO V. CA SEC 16 DEATH OF PARTY Facts: Alfonso P. Orfinada, Jr. died without a will in Angeles City leaving several personal and real properties located in Angeles City, Dagupan City and Kalookan City. He also left a widow, respondent Esperanza P. Orfinada, whom he

married on July 11, 1960 and with whom he had seven children who are the herein respondents, , namely: Lourdes P. Orfinada, Alfonso Clyde P. Orfinada, Nancy P. Orfinada-Happenden, Alfonso James P. Orfinada, Christopher P. Orfinada, Alfonso Mike P. Orfinada (deceased) and Angelo P. Orfinada. The demise of the decedent left in mourning his paramour and their children. They are petitioner Teodora Riofero, who became a part of his life when he entered into an extra-marital relationship with her during the subsistence of his marriage to Esperanza sometime in 1965, and co-petitioners Veronica, Alberto and Rowena. Respondents Alfonso James and Lourdes found out that Teodora and her children executed an Extrajudicial Settlement of Estate of a deceased person with quitclaim involving the property of the estate of the decedent located in Dagupan City and accordingly, the Registry of Deeds in Dagupan issued corresponding CTCs. Furthermore, the respondents also found out that petitioners were able to obtain a loan of P700,000 from the Rural Bank of Mangaldan by executing a Real Estate Mortgage over the said properties. Respondent Alfonso Clyde P. Orfinada III filed a Petition for Letters of Administration docketed as S.P. Case No. 5118 before the Regional Trial Court of Angeles City, praying that letters of administration encompassing the estate of Alfonso P. Orfinada, Jr. be issued to him. Respondents filed a Complaint for the Annulment/Rescission of Extra Judicial Settlement of Estate of a Deceased Person with Quitclaim, Real Estate Mortgage and Cancellation of Transfer Certificate of Titles with Nos. 63983, 63985 and 63984 and Other Related Documents with Damages against petitioners, the Rural Bank of Mangaldan, Inc. and the Register of Deeds of Dagupan City before the Regional Trial Court, Branch 42, Dagupan City. In their Answer, Petitioners raised the affirmative defense that respondents are not the real parties-in-interest but rather the Estate of Alfonso O. Orfinada, Jr. in view of the pendency of the administration proceedings. Petitioners filed a Motion to Set Affirmative Defenses for Hearing, which the trial court denied on the ground that respondents, as heirs, are the real parties-in-interest especially in the absence of an administrator who is yet to be appointed in S.P. Case No. 5118. Issue: 1. Whether the trial court exercised a grave abuse of discretion in denying the petitioners' Motion to Set Affirmative Defense in Hearing. 2. Whether the heirs have legal standing to prosecute the rights belonging to the deceased subsequent to the commencement of the administration proceedings. Held: 1. No. The Rules of Court clearly provides that the holding of a

2.

preliminary hearing on an affirmative defense lies in the discretion of the court. Yes. Pending the filing of administration proceedings, the heirs without doubt have legal personality to bring suit in behalf of the estate of the decedent.

56. ANTONE v BERONILLA SEC 22 SOLICITOR GENERAL LEGAL STANDING; RULE 7 SEC 4VERIFICATION FACTS Antone file a complaint for bigamy against her husband, Beronilla, for contracting a second marriage when the marriage between him and Antone has not been dissolved. The Pasay City trial court dismissed the complaint. The case was brought to the CA via petition for certiorari which was dismissed because, according to the CA, the petition is fatally infirm in form and substance because: (1) the verification is defective as it does not include the assurance that the allegations in the petition are based on authentic records, and (2) the petitions should have been filed in behalf of the People of the Philippines by the Office of the Solicitor General. ISSUE Whether or not the dismissal by the CA for the reasons provided lacks merit HELD YES. The petition should be given due course despite the defect in the pleading and the question of legal standing to bring the action. The Rules of Court provide that a pleading required to be verified which lacks a proper verification shall be treated as an unsigned pleading. However, in numerous cases, a liberal construction of the rule was use in order that the ends of justice may be served. The defect is merely formal and not jurisdictional. The court may order the correction of the pleading, or even act on the pleading if the attending circumstances are such that strict compliance with the rule may be dispensed with in order that the ends of justice may be served. A pleading is required to be verified only to ensure that it was prepared in good faith and that the allegations were true and correct and not based on mere speculations. Also, there is no dispute that it is the OSG which has the authority to represent the government in a judicial proceeding before the CA. Generally, the summary dismissal of an action in the name of the Republic of the Philippines, when not initiated by the Solicitor General, is in order.

Ratio: 1. Rule 6, Sec. 5 provides "Pleadings grounds as affirmative defenses.Any of the grounds for dismissal provided for in this rule, except improper venue, may be pleaded as an affirmative defense, and a preliminary hearing may be had thereon as if a motion to dismiss had been filed." The incorporation of the word may in the provision is clearly indicative of the optional character of the preliminary hearing. The word denotes discretion and cannot be construed as having a mandatory effect. Subsequently, the electivity of the proceeding was firmed up beyond cavil by the 1997 Rules of Civil Procedure with the inclusion of the phrase in the discretion of the Court, apart from the retention of the word may in Section 6, in Rule 16 thereof. 2. Even if administration proceedings have already been commenced, the heirs may still bring the suit if an administrator has not yet been appointed. In fact, in the case of Gochan v. Young,[28] this Court recognized the legal standing of the heirs to represent the rights and properties of the decedent under administration pending the appointment of an administrator. Thus: "The above-quoted rules, while permitting an executor or administrator to represent or to bring suits on behalf of the deceased, do not prohibit the heirs from representing the deceased. These rules are easily applicable to cases in which an administrator has already been appointed. But no rule categorically addresses the situation in which special proceedings for the settlement of an estate have already been instituted, yet no administrator has been appointed. In such instances, the heirs cannot be expected to wait for the appointment of an administrator; then wait further to see if the administrator appointed would care enough to file a suit to protect the rights and the interests of the deceased; and in the meantime do nothing while the rights and the properties of the decedent are violated or dissipated." Evidently, the necessity for the heirs to seek judicial relief to recover property of the estate is as compelling when there is no appointed administrator, if not more, as where there is an appointed administrator but he is either disinclined to bring suit or is one of the guilty parties himself. the rule that the heirs have no legal standing to sue for the recovery of property of the estate during the pendency of administration proceedings has three exceptions, one being when there is no appointed administrator such as in this case.

However, there are exceptions to this rule. Due course were given to actions even when the respective interests of the government were not properly represented by the OSG. The exception operates when the challenged order affects the interests of the State or the plaintiff People of the Philippines. Even absent the imprimatur of the Solicitor General, the petition must not be dismissed on a technical ground.

obligation. However, HT failed to pay its obligation. The parties entered into a restructuring agreement, with a stipulation regarding venue. Venue- venue of any action or proceeding arising out of or connected with this Restructuring Agreement, the Note, the Collateral and any and all related documents shall be in Makati City, parties hereby waiving any other venue. However, despite the restructuring agreement, HT still failed to settle its obligation, thus the extrajudicial foreclosure of the REM. HT filed for TRO, injunction and prohibition assailing the validity of the foreclosure of the REM in RTC Cebu City. ISSUE: Whether Makati City is the proper venue to assail the validity of the foreclosure of REM. HELD: YES. Although Rule 4 Section 1 states that the proper venue of instituting real actions is where the property is situated, Rule 4 Section 4 provides that parties may stipulate an exclusive venue. Since an exclusive venue is stipulated in the agreement, the venue shall be mandatory. 59. HONORORIO BERNARDO VS. HEIRS OF VILLEGAS FACTS: Eusebio owns land in Rizal. Bernardo entered into possession of portion of land and constructed a house. Thus, Bernardo was sued for accion publiciana by the heirs of Eusebio. Earlier, the heirs had filed an ejectment case under MTC, which was dismissed for prescription (filed beyond the 1-year period). In the answer, bernardo denied possession of the property and that the cause of action is barred by the judgement in the previously dismissed ejectment case in the MTC. In the same answer, he questioned RTCs jurisdiction over the case. RTC contended that it has jurisdiction since the action is accion publiciana. On appeal, Bernardo questioned once again the jurisdiction of RTC over the subject matter. ISSUE: Whether estoppels bars the petitioner from raising the issue of lack of jurisdiction over the subject matter. HELD: Although appellant raised the issue of jurisdiction in his answer, he did not file a motion to dismiss. The appellant is now stopped to question jurisdiction over the subject matter and nature by filing various pleadings. Thus, RTC has jurisdiction over the case. 60. SPS. DANILO SAMONTE VS. CENTURY SAVINGS

RULE 4 VENUE

OF

ACTIONS

57. GENEROSA ALMEDA LATORRE V. LATORRE SECTION 1 FACTS: Luis and Ifzal entered into a Contract of Lease over a property at Dasmarinas Makati City, stating that Luis is the absolute owner of the property. Generosa said that this was not true because she is the coowner (and mother) or Luis. It was alleged that Generosa and Luis executed Deeds of Donation in favour of a foundation. Then, with the consent of the foundation, Deeds of Revocation were executed, although it was unregistered. Thus, the title is still with the foundation. Generosa demanded for part of the rentals from Ifzal, but the latter refused. Later on, Generosa discovered that Luis caused annotation of an adverse claim by virtue of a Deed of Absolute Sale, which was allegedly falsified. Generosa sued for collection. Luis filed a motion to dismiss, alleging that venue was improperly laid. ISSUE: Whether RTC Makati is the proper venue since the action is a real action. HELD: YES. Makati City is the proper venue. While the complaint was for collection and declaration of nullity of Deed of Absolute Sale, in truth, it is one involving a real action. Thus, RTC Makati City is the proper venue [Rule 4 section 1] and not RTC Muntinlupa. 58. PAGLAUM MANAGEMENT V. UNION BANK OF THE PHILS STIPULATION OF VENUE FACTS: Paglaum is the registered owner of 3 parcels of land in Ceby, co-owned by Dy, president of Health Marketing Technologies Inc. (HT). Union Bank extended HT a credit line of 10M. Paglaum executed a REM to secure the

[I dunno how this case became a case under Rule 4, because no issue on venue was raised AT ALL. I think this ones mi splaced, more of a case for Rule 70.] FACTS: Sps Samonte obtained a 1.5 M loan from Century, secured by REM over a property in Makati City. For failure to pay, the REM was extrajudicially foreclosed, Century as the highest bidder. The spouses failed to redeem the property within the redemption period, thus Centry leased the premises to the spouses. Spouses failed religiously pay the rentals. Thus, Century demanded them to pay the unpaid rentals and vacate the premises. Spouses refused. Spouses insisted on the nullity of the foreclosure proceedings to justify their failure to pay the rents/vacate. They insisted that the ejectment case should await the result of the separate action the instituted for the nullification of the foreclosure proceedings. ISSUE: Whether the instant ejectment case should be suspended pending the resolution of the action for the nullity of the foreclosure. HELD No. Under Rule 70 of the Rules of Court, ejectment suits are designed to restore physical possession to one who has been illegally/forcible deprived without prejudice for settlement of the parties opposing claims of juridical possession. Therefore, nullification of foreclosure proceedings does not suspend ejectment suit.

Infantesought the dismissal of the revival on the grounds that Muntinlupa RTC has no jurisdiction over the persons of the parties and that venue was improperly laid. CA ruled in favor of Aran Builders because accdg to CA the action for revival of judgment is an action in rem thus should be filed with RTC of the place where the real property was located.

ISSUE:WHETHER OR NOT an action for revival of judgment of an action involving title or possession of real property is an action IN REM. YES RATIO: Infanta avers that it is an action in personam therefore the complaint should be filed with the RTC of the place where either petitioner or respondent resides thus the case should be dismissed for improper venue. SC ruled that since in this case, the action to revive the judgmentaffects Arans interestoverrealproperty therefore it is an action in remthus Sec. 1 Rule 41applies in this case. Infantaeven argued using the case of Aldeguer vs. Gemelo, but that case cannot apply to the present case because in Aldeguer the revival sought was only a judgment for damages and it didnt affect any title or possession of real property or interest therein. SEE: THEODORE AND NANCY ANG V. SPS. ALAN AND EMANG UNDER RULE 3; VENUE

RULE 5 UNIFORM P ROCEDURE IN T RIAL C OURTS 61. INFANTE VS ARAN BUILDERS, INC Syllabus: Venue; Revival of judgment; The proper venue in an action for revival of judgment DEPENDS on the determination of whether the present action for revival of judgment is a real action or a personal action FACTS: On 2001, an action for revival of judgment was filed by Aran builders against Infante to RTC of MuntinlupaCity. The judgment sought to be revived was rendered by RTC in an action for specific performance + damages, it became final and executory on 1994(Territorial jurisdiction is in Makati because Muntinlupa had no RTC yet that time) The judgment sought to be revived involved the interest, possession,title, and the ownership of the parcel of land located in Muntinlupa city.

62. ANGELINA SORIENTE, ET. AL VS. THE ESTATE OF THE LATE ARSENIO E. CONCEPCION ET. AL., GR. NO. 160239 RULE 5; RULE 8; RULES ON SUMMARY PROCEDURE FACTS:

Section 1.Venue of real actions. Actions affecting title to or possession of real property, or interest therein, shall be commenced and tried in the proper court which has jurisdiction over the area wherein the real property involved, or a portion thereof, is situated.xxx

Respondent Nenita Concepcion established that she was the registered owner of the lot occupied by petitioner in Mandaluyong City, covered by TCT-A, which she inherited from her predecessor, Arsenio Concepcion. Arsenio Concepcion acquired the lot in 1978 and merely allowed and tolerated the occupancy of the lot by petitioner Soriente. Petitioner was allowed to stay on the lot for free, but on a temporary basis until such time that Concepcion and/or his family needed to develop the lot. After Arsenio Concepcion died, his family initiated steps to develop the lot, but petitioners occupancy of the lot pre vented them from pursuing their plan. Verbal demands to vacate the lot was made on petitioner, but she never left. Respondent filed against petitioner a complaint for unlawful detainer which stated that petitioner had no title to the property and her free occupancy thereof was merely tolerated by respondent. Petitioner Soriente did not file a separate Answer, but affixed her signature to the Answer filed by A DIFFERENT defendant in ANOTHER EJECTMENT CASE instituted by respondent against one, Alfredo Caballero over the same property. Pursuant to Section 7 of the 1991 Revised Rule on Summary Procedure which governs preliminary conferences and appearance of parties, the trial court set a preliminary conference which was rescheduled multiple times. When it finally pushed through, the Caballeros, counsel for the respondents, and a representative for petitioner Soriente were present. However, Sorientes representative failed to submit a SPA authorizing her to enter into a compromise agreement. In view of the absence of Sorientes representative, respondents counsel moved that the case be submitted for decision. The trial court rendered a decision in favor of respondent and ordering petitioner to vacate the premises and pay damages. Petitioner appealed on the grounds that 1.) petitioner did not have legal capacity to sue since she was not the registered owner of the lot; and 2.) that the court erred in holding that this instant case be decided in accordance with Section 7 of the Rules on Summary Procedure.

2. Is the case at hand subject to Section 7 of the Rules on Summary procedure? HELD: 1. Yes. Petitioner does have legal capacity to sue. 2. Yes. Section 7 which governs preliminary conferences, the appearance of parties, and the effects of non-appearance applies to the case at bar. RATIOS: 1. Section 4, Rule 8 of the 1997 Rules of Civil Procedure provides:

Sec. 4. Capacity. x x x A party desiring to raise an issue as to the legal existence of any party or the capacity of any party to sue or be sued in a representative capacity, shall do so by specific denial, which shall include such supporting particulars as are peculiarly within the pleaders knowledge. The SC ruled that under this rule, the petitioner should have at least SPECIFICALLY DENIED such capacity of the party in the Answer. The case records clearly disclose that no such specific denial was made by the appellant. Furthermore, as the successor-in-interest of the late Arsenio E. Concepcion and co-owner of the subject property, respondent Nenita Concepcion is entitled to prosecute the ejectment case not only in a representative capacity, but as a real party-in-interest, pursuant to Article 487 of the Civil code which states that Any one of the co-owners may bring an action in ejectment 2. Section 7 of the Rules on Summary Procedure provides: SEC.7. Preliminary conference; appearance of parties. Not later than thirty (30) days after the last answer is filed, a preliminary conference shall be held. The rules on pre-trial in ordinary cases shall be applicable to the preliminary conference unless inconsistent with the provisions of this Rule. The failure of the plaintiff to appear in the preliminary conference shall be a cause for the dismissal of his complaint. The defendant who appears in the absence of the plaintiff shall be

ISSUES: 1. Pursuant to Section 4, Rule 8 of the 1997 Rules of Civil Procedure, does respondent have legal capacity to sue?

entitled to judgment on his counterclaim in accordance with Section 6 hereof. All cross-claims shall be dismissed. If a sole defendant shall fail to appear, the plaintiff shall be entitled to judgment in accordance with Section 6 thereof. This Rule shall not apply where one of two or more defendants sued under a common cause of action who had pleaded a common defense shall appear at the preliminary conference. Section 6 of the Rules on Summary Procedure as stated in the above section provides: SEC. 6. Effect of failure to answer. Should the defendant fail to answer the complaint within the period above provided, the court, motu proprio, or on motion of the plaintiff, shall render judgment as may be warranted by the facts alleged in the complaint and limited to what is prayed for therein: Provided, however, That the court may in its discretion reduce the amount of damages and attorneys fees claimed for being excessive or otherwise unconscionable. This is without prejudice to the applicability of Section 4, Rule 18 of the Rules of Court, if there are two or more defendants. The SC ruled that the contentions of petitioners that the court should not have rendered judgment on her case based on the above cited sections of the Rules on Summary Procedure because she was able to affix her signature on the answer of Caballero, thus consolidating their cases, lacks merit. The Court holds that the ejectment case filed against petitioner was distinct from that of Caballero, even if the trial court consolidated the cases and, in the interest of justice, considered the Answer filed by Caballero as the Answer also of petitioner since she affixed her signature thereto. Considering that petitioner was sued in a separate case for ejectment form that of Caballero, petitioners failure to appear in the preliminary conference entitled respondent to the rendition of judgment by the trial court in the ejectment case filed against petitioner, in accordance with Section 7 of the 1991 Revised rules on Summary Procedure.

PROCEDURE I N REGIONAL T RIAL COURTS RULE 6 KINDS OF P LEADINGS 63. MARANAN v. MANILA BANKING CORPORATION Facts: Mandarin obtained from Manila Banking Corporation a loan in the amount of 10M Php and by virtue of which Petitioner Maranan et. al signed the surety agreement and held themselves solidarily liable with the corporation The outstanding loan obligation reached 30, 500,000Php in which Manila Banking Corporation filed a complaint for a sum of money against Pacific Enamel, Chua, other guarantors including herein petitioner Chua and Pacific Enamel filed a Motion to Dismiss on the ground that the complaint states no cause of action against them and accordingly, trial court dismissed the case as to them and dropped them off the case. Petitioner filed her answer alleging that the surety agreement did not express the true intention of the parties. Maranan claimed that Chua was the real borrower and that Mandarin was merely used as a business conduit of Chua. Furthermore, Maranan stated that she was only made to sign the agreement as an employee of the company and in compliance with the requirements set forth by the Central Bank Responden filed a Motion for Judgment on the Pleadings, trial court however, denied the same Two years later, petitioner filed an Amended Answer impleading Chua and Pacific Enamel as defendant in the counter claim. The Amended answer consists of new allegations that the surety agreement was void as an accessory contract because the PNs were not existing at the time of the agreement. Trial court denied the admission of the Amended Answer and declared that the same was done without prior leave of court. ISSUE: Did CA commit GAD in dismissing the petition for certiorari? RULING: Amended Answer as a matter of right

Petitioners answer was confined to the allegations that the surety agreement did not express the true intention of the parties. Answer merely purported to show that Chua was the real borrower and the actual recipient of the loan. The Amended Answer however, contained substantial amendments not found in her original answer. Sections 2 and 3 of Rule 10 substantially states that amendments will be a matter of right if the pleading is one in which no responsive pleading is permitted and furthermore, after the case is set for hearing substantial amendments may be made only upon leave of court ---> This petitioner failed to do. Her amended answer contained substantial allegations which were clearly not part of her prior answer and thus, the Rules provide that such substantial amendments must be done upon leave of court. On the resort to a counter-claim instead of a third party claim

Section 6 and 14 of Rule 6 provides the definitions of a counter claim and third party claim respectively in that a counterclaim is filed against an opposing party and a third party claim is flied against a person not a party to the action. Technicality suggests that Chua and Pacific Enamel are both dropped off as parties to the case and thus, are considered as persons not party to the action when Maranan filed a counterclaim. Even a plea for liberal construction of the rules would not work to the favor of Maranan since the error in the case at bar is not that which is excusable inadvertence. 64. RIMBUNAN HIJAU & NIUGINI LUMBER vs. ORIENTAL WOOD PROCESSING CORP. FACTS: Niugini and Rimbunan are foreign corporations duly organized and existing under the laws of Papua New Guinea (PNG) while respondent is a private domestic corporation. Petitioners are seeking to recover a certain sum of money. The amount represented the alleged remaining balance of the purchase price for the mixed species of PNG logs which petitioners sold and exported to respondent. Petitioners allege that they are nonresident foreign corporations, not doing business in the Philippines. Niugini is a subsidiary of plaintiff Rimbunan. They are engaged in the business of extraction and exportation of PNG round logs. They have no representative/liason offices, or branch offices in the Philippines and they are not licensed to do business here. They

do not engage in any business in this jurisdiction except for some isolated transactions. Plaintiffs sold and delivered to defendant 8Million cubic meters of mixed group species of PNG round logs. Defendant promised to open a letter of credit in favor of plaintiff Niugini to cover the transaction. After the vessel carrying the logs left the port of PNG the defendant, through fax, informed the plaintiffs that it would resort to telegraphic transfer directly to plaintiffs bank account to pay the said purchase price. They cited tight government regulations which allegedly made the processing of the latter of credit difficult. Partial payment was made through three telegraphic transfers and a balance of $393, 699.52 was left to be paid. Despite this, the plaintiffs, relying on the promise made by the defendant that it would immediately remit the remaining balance by telegraphic transfer, transmitted to the defendant the original copies of the Bill of Lading, Commercial Invoice, and Packing List to enable it to obtain the release of the goods from customs authorities. Defendant failed to remit by telegraphic transfer the remaining balance and through a telephone conversation defendant promised to settle the balance within one month. However they did not keep their promise and despite several demands, no payment was made. Several months later, though a letter, defendants expressly acknowledged their debt however they claimed that their revolving capital has been diminished and therefore they could not give a firm schedule of payment. Defendant was able to make a payment reducing the amount payable to $343, 741.52. However plaintiff Niugini, later on, demanded for the payment within 7 days. Despite this, defendant did not make any further payments. Plaintiff Niugini though its Philippine counsel Tan &Venturanza Law offices, sent a final demand letter which was again disregarded by the Defendants. The respondent filed a motion to dismiss on the grounds that petitioners have not legal capacity to sue in this jurisdiction. In response to this petitioners filed an opposition with a motion positing that respondent was estopped from questioning their capacity to sue. The trial court denied these motions. In denying the respondents Motion to dismiss the trial court ruled that the petitioners were not doing business in the Philippines but were merely suing on an isolated transaction. Thus petitioners were legally capacitated to institute and maintain an action against respondent. It also held that respondent is estopped from challenging the personality of petitioners after having acknowledged the same by entering into a contract with them.

Respondent filed an action for certiorari raising the defense that the trial court erred in claiming that Rimbunans actions were mere isolated transactions because in reality Rimbunan made numerous shipments of logs. The CA granted respondents petition for certiorari and ordered the dismissal of the complaint. The CA held that petitioners were proscribed from suing respondent in the Philippine courts. Petitioners are now challenging the decision of the CA, they aver that the CA had no proof on the alleged existence of the 14 transactions. As this was the basis used by the CA in concluding that petitioners were doing business in the Philippines it was vital for the parties to present evidence on the matter. Moreover, since the issue was factual, the same should have been threshed out before the trial court. Therefore the CA was wrong to have ruled on a question of law on the basis of alleged facts that had not even been established yet. And even if there were 14 transactions the petitioners would still have capacity to sue because the respondents are barred by estoppel since they have already benefited from the contract.

on an isolated transaction or was doing business in the Philippines. This requires at least a preponderant set of facts. The CA, in concluding that petitioner were doing business in the Philippines relied merely on respondents say-so. Allegations must be proven by sufficient evidence. Mere allegation is not evidence. Absent any evidence to prove respondents allegation in the case at bar, the SC fails to see how the CA could have concluded that petitioners were doing business in the Philippines. Based on this, the reinstatement of this case in the lower court for further proceedings, including trial on the merits, is necessary.Even without the foregoing ground, the reversal of the CA decision and the reinstatement of this case at the trial court level is still warranted. The lower court in its resolution correctly held that respondent is estopped from questioning petitioners capacity to sue in this jurisdiction. The trial court found that respondent had contracted with petitioners and had in fact made partial payment on its obligation. After contracting with a foreign corporation, a domestic firm is estopped from denying the formers capacity to sue. In the case of Antam Consolidated, Inc. v. CA, this court noted that it is a common ploy of defaulting local companies which are sued by unlicensed foreign corporations to invoke the latters lack of capacity to sue. This practice of domestic corporations is particularly reprehensible considering that in requiring a license, the law never intended to prevent foreign corporations from performing single or isolated acts in this country. Rather, the law was intended to bar foreign corporations from acquiring a domicile for the purpose of business without first taking the steps necessary to render them amenable to suits in the local courts. While the doctrine of lack of capacity to sue based on failure to first acquire local license may be resorted to in meritorious cases, it cannot be called upon when no evidence exists to support its invocation or the facts do not warrant its application. In this case, the respondent is estopped from challenging the petitioners capacity to sue and t he forthcoming trial before the lower court should weigh instead on the other defenses raised by the respondent.

ISSUE: W/N Petitioner has locus standi to sue respondent? HELD: The petition has Merit. The case is REMANDED to the lower court for further proceedings. RATIO: An order to dismiss is an interlocutory order which does not terminate nor finally dispose of a case. Therefore the general rule is that the denial of a motion to dismiss cannot be questioned in a civil action for certiorari (a remedy designed to correct errors of jurisdiction and not errors of judgment). An extraordinary remedy of certiorari may be granted, however, when the denial of the motion to dismiss has been tainted with great abuse of discretion. In the case at bar, no explanation was given by the CA to justify this grant of extraordinary remedy of certiorari. Facts show that the CA substituted its evaluation of the motion to dismiss and the opposition thereto for that of the trial courts, without regard to the evidence which was still to be presented on the issue of whether or not petitioner are doing business in the Philippines or merely suing on an isolated transaction. The question of petitioners capacity to sue is a factual question. An unlicensed foreign corporation is nonetheless permitted to bring suit in the Philippines if it is suing on an isolated transaction. Thus, it is integral that it is ascertained whether the foreign corporation was merely suing

65. SPOUSES MENDIOLA vs. COURT OF APPEALS, PILIPINAS SHELL and TABANGAO REALTY Rule 6 Compulsory Counterclaim Res judicata

Rule 37 Motion for Reconsideration Rule 41 Period ordering appeal! FACTS: Shell entered into an agreement for the distribution of Shell petroleum products with Pacific Management and Development (Pacific), a sole proprietorship owned by the Petitioners Mendiola. To secure performance of the obligations under the agreement, Petitioners executed a Real Estate Mortgage (REM) over their Paraaque properties in favor of Shell. Pacific defaulted forcing Shell to commence Extrajudicial Foreclosure (EJF-REM) proceedings. During the scheduled auction, Petitioners went to the announced venue in the Paraaque Municipal Hall. But Petitioners did not witness any auction take place and they found out later that an auction was held in Makati City Hall and that their properties had been sold to Tabangao Realty. After applying the proceeds of the auction sale to the obligations of the Petitioners, a deficiency remained. This prompted Shell to institute a case in the Manila RTC to recover the sum of money (SOM). Petitioners, in an Answer with Counterclaim contended that the EJF-REM was without basis in fact and in law and that it had been made fraudulently and in bad faith. Petitioners filed an action to annul the EJF-REM (Annulment Case) in Makati RTC with Shell and Tabangao Realty as defendants. Pending trial in the Annulment case in Makati, the Manila court rendered its judgement in favor of Shell in the SOM case, ordering Petitioners to pay Shell. Petitioners appealed the SOM case to the Court of Appeals. The Court of Appeals affirmed the Manila case, finding also that Petitioners are guilty of forum shopping for commencing the Makati case. The Supreme Court affirmed the findings of the CA. Meanwhile, the Makati Court, in the Annulment Case ruled in favor of the Petitioners and declared the auction sale null and void. Shell and Tabangao (Pvt. Respondents) moved to have the case reconsidered on the ground of res judicata since the Manila court already ruled on the validity of the auction sale. The Makati Court denied the Motion for Reconsideration in a resolution. Pvt. Respondents appealed the Motion for Reconsideration to the Court of Appeals. Petitioners, instead of filing their appellees' brief filed a motion to dismiss appeal on the ground that an appeal is proscribed as a remedy to the resolution of a lower court. The Court of Appeals denied the motion to dismiss. Petitioners brought these special civil actions for certiorari,

mandamus and prohibition claiming that CA committed a grave abuse of discretion in denying the motion to dismiss appeal. ISSUES: Whether or not an appeal may be taken from the denial of a motion for reconsideration of a decision. Whether or not the Makati Annulment Case should prosper independently of the Manila SOM case.

HELD: 1. Yes; appeal by Shell and Tabangao of the denial of their motion for reconsideration is not proscribed. It is true that the original text of Sec. 1, Rule 41 expressly limited appeal to a judgement or final order, and proscribed the taking of an appeal from and order denying a motion for new trial or reconsideration among others. Furthermore, Sec. 9, Rule 37 of the Rules of Court provide that an order denying a motion for new trial or reconsideration is not appealable, the remedy being an appeal from the judgement or final order. However, in a number of cases, the Supreme Court has categorized an order of dismissal of a complaint as not being a prohibited interlocutory order, but a final order as it puts an end to the particular matter resolved, or settles definitely the matter therein, leaving nothing for the court to do but to execute the order. Another consequence is that the reglementary period for the perfection of appeal should start to run from the day that a party receives the final order of a court denying their motion for reconsideration and not from the day they received an order dismissing their complaint. Thus, the appeal taken by Shell and Tabangao is proper. 2. No; Makati case is barred and should be dismissed on the ground of res judicata. The Supreme Court finds that the compulsory counterclaim filed by the Petitioners in the Manila SOM Case is virtually identical to their complaint in the Makati Annulment Case. The Makati case had logical relation to the Manila case because both arose out of the EJF-REM. Specifically, the right of Shell to demand SOM was predicated on the validity of the EJF-REM. The annulment of the EJF-REM was a true compulsory counterclaim in the Manila case. Thus Makati RTC should have dismissed the annulment case. The following elements of res judicata were present: 1) The former judgement or order must be final; 2) The judgement or order must be on the merits;

3) It must have been rendered by a court having jurisdiction over the subject matter and the parties; 4) there must be, between the first and second action, identity of parties, of subject matter and cause of action. It must be noted that the fourth element does not refer to absolute identity. The test to determine identity is to ascertain whether the same evidence will sustain the actions, or whether there is an identity in the fact essential to the maintenance of the actions. Wherefore, the petition for certiorari, prohibition and mandamus was dismissed.

RULE 7 PARTS

OF A

P LEADING

Sometime thereafter the HLURB issued a cease and desist order (CDO) enjoining the collection of amortization payments. This CDO was subsequently lifted. Thereafter, complainants went to the Sps. Genato with the intention of resuming their amortization payments. The latter however refused to accept their payments and instead demanded for a lump sum payment of all the accrued amortizations which fell due during the effectivity of the CDO. Housing Arbiter ordered that complainantsresume payment of amortization. HLURB Board of Commissioners, the Decision was modified by the additional directive for the complainants to pay 3% interest per month for the unpaid amortizations A Writ of Execution was issued and thus, the sheriff seized Rita Viola's two delivery trucks and 315 sacks of rice. Respondent Viola then filed an Urgent Motion to Quash with TRO. This was granted. Spouses argued that jurisdiction over viola was acquired through consent given by Viola, while Viola argues that the HLURB did not acquire jurisdiction over her person since she was not a party to the case as can be gleaned by the caption

RECALL ANTONE v BERONILLA

UNDER: RULE 3 SEC 22 SOLICITOR GENERAL LEGAL STANDING; FOR: RULE 7 SEC 4VERIFICATION

Issue: W/N HLURB acquired Jurisdiction over Viola even if her name was not in the caption Held: YES Ratio: It is not the caption of the pleading but the allegations therein that are controlling. The inclusion of the names of all the parties in the title of a complaint is a formal requirement under Section 3, Rule 7 of the Rules of Court. However, the rules of pleadings require courts to pierce the form and go into the substance. The non-inclusion of one or some of the names of all the complainants in the title of a complaint, is not fatal to the case, provided there is a statement in the body of the complaint indicating that such complainant/s was/were made party to such action. This is specially true before the HLURB where the proceedings are summary in nature without regard to legal technicalities obtaining in the courts of law and where the pertinent concern is to promote public interest and to assist the parties in obtaining just, speedy and inexpensive determination of every action, application or other proceedings. Respondent Viola, although her name did not appear in the title as a party, was one of the persons who caused the preparation of the complaint and who verified the same. The allegations in the body of the complaint indicate that she is one of the complainants. She categorically considered, and held out, herself as one of the complainants

RECALL S.C.

MEGAWORLD V. ENGR. PARADA G.R. NO. 183804, SEPT. 11, 2013 UNDER: RULE 3 SEC 1 WHO MAY BE PARTIES;

FOR: SECS 4 AND 5 RULE 7

66. SPS. WILLIAM GENATO VS. RITA VIOLA Rule 7 Facts: A complaint titled "VILLA REBECCA HOMEOWNERS ASSOCIATION, INC. versus MR. WILLIAM GENATO and spouse REBECCA GENATO" was filed with the HLURB. The said complaint was verified by 34 individuals, including Rita Viola, who referred to themselves as the "Complainants". The complaint stated that on various dates, complainants executed Contracts to Sell and/or Lease Purchase Agreements with the Sps. Genato pertaining to housing units in Villa Rebecca Homes Subdivision.

from the time of the filing of the complaint and up to the time the decision in the HLURB case became final and executory. For clarity, the complaint should have been amended to reflect in the title the individual complainants. There being a "defect in the designation of the parties", its correction could be summarily made at any stage of the action provided no prejudice is caused thereby to the adverse party 67. COMMISSION ON APPOINTMENTS V. CELSO PALER Rule 7 Sec 3 Facts: Respondent Celso M. Paler was a supervising legislative staff officer II with the technical support service of the commission on appointments. On April 8, 2003, he submitted a request for vacation leave for 74 working days from August 1, 2003 to November 14, 2003. Since he already had an approved leave from June 9 to July 30, 2003, Paler left for the United States on June 8, 2003, without verifying whether his application for leave (for August 1 November 14, 2003) was approved or denied. In a letter dated September 16, 2003, the Commission Chairman informed Paler that he was being dropped from the roll of employees effective said date, due to his continuous 30-day absence without leave. Paler moved for reconsideration but this was denied on February 20, 2004, on the ground that it was filed beyond the 15-day reglementary period. On appeal, the CSC reversed and set aside the Commission Chairman's decision. The Commission filed a motion for reconsideration but this was denied by the CSC. This constrained petitioner to file with the CA a petition for review under Rule 43 of the Rules of Court. Since Paler had in the meantime already reached the compulsory age of retirement on July 28, 2005 and was no longer entitled to reinstatement, the CA affirmed with modification CSC resolution. Petitioner filed a motion for reconsideration but this was denied by the CA. In his comment, Paler, aside from arguing that the CA did not commit any error in sustaining the CSC resolutions, also assails Atty. Arturo L. Tiu's authority to file the petition and sign the verification and certification of non-forum shopping on behalf of the Commission Chairman. The CSC, represented by the Office of the Solicitor General (OSG), maintains the correctness of the CSC and CA judgments. Issues: Whether or not the Commission Secretary has the authority to file the petition and sign the verification and certification of non-forum shopping in behalf of the Commission Chairman? Ruling:

The petitioner in this case is the Commission on Appointments, a government entity created by the Constitution, and headed by its Chairman. There was no need for the Chairman himself to sign the verification. Its representative, lawyer or any person who personally knew the truth of the facts alleged in the petition could sign the verification. With regard, however, to the certification of nonforum shopping, the established rule is that it must be executed by the plaintiff or any of the principal parties and not by counsel. In this case, Atty. Tiu failed to show that he was specifically authorized by the Chairman to sign the certification of non-forum shopping, much less file the petition in his behalf. There is nothing on record to prove such authority. Atty. Tiu did not even bother to controvert Palers allegation of his lack of authority. This renders the petition dismissible.

68. EAGLE RIDGE GOLF AND COUNTRY CLUB V CA Rule 7 sections 3 and 5 Eagle Ridge is a corporation engaged in the business of maintaining golf courses. On December 6, 2005, at least 20% of its rank and file members( percentage threshold required under art 234 of the Labor Code) had a meeting where they organized themselves as a union. This was contested by eagle ridge., alleging the yet to be formed union to have committed misrepresentation, fraud, false statement in making its constitution and by-laws, numerical composition of the union, and the election of its officers. The dispute was submitted to the BLR for its resolution. BLR resolved in favor of the Eagle Ridge. Failing to gain an affirmative answer before the BLR, the union, represented by Atty Luna Piezas, filed a petition for certiorari provided in Rule 46. However, the verification and certification of non-forum shopping was subscribed to by Atty Piezas on her representation as counsel for the yet to be formed union, but sans the requisite secretarys cert or board resolution authorizing her to execute and sign the same. CA dismissed the case because Atty Piezass lack of authority to execute such. Issue: did CA committed GAD in dismissing the action. Held: no. The rules require the petitioner, not his counsel, to sign under oath the requisite certification against non-forum shopping. This certification is a peculiar personal representation on the part of the principal party, an assurance to the court that there are no other pending cases involving basically the same parties, issues and cause of action. Take note that specifically, sec 3, rule 46, require the petitioner to submit the petition with a sworn certification that he has not commenced any action involving the same issues in the SC, CA, or any tribunal or agency.

YES 69. IMELDA BIDES-ULASO V. ATTY. EDITA NOE-LACSAMA Rule 7 FACTS: Respondent attorney was the counsel of Irene Bides when the latter filed a civil action in the RTC against Imelda ULASO (plaintiff in this case), her own niece. BIDES amended the complaint to demand the declaration of nullity of the deed of sale pertaining to a parcel of land in Manila of which BIDES was the registered owner. The amended complaint of BIDES contained an amended verification and affidavit of non-forum shopping, on which was the signature of the respondent preceded by a word for above the printed name IRENE BIDES as the notary on the jurat ULASO filed a motion to dismiss on the ground that amended verification and affidavit of non-forum shopping was defective. Respondent counsel and BIDES opposed the motion to dismiss contending that the defective amended verification and affidavit had actually been a sample draft intended to instruct the respondents secretary on where BIDES should sign. RTC denied the motion to dismiss. BIDES next formally charged ULASO with falsification of public document for the execution of the nullified deed of sale resulting in the latters criminal prosecution. To counteract the moves of BIDES, ULASO filed a disbarment case against respondent attorney for her act of signing the amended verification and affidavit of non-forum shopping and notarizing the document without the signature of BIDES and despite the non-appearance of BIDES before her. Afterwards, BIDES and ULASO entered into a compromise agreement = BIDES agreed to drop to the criminal charge against ULASO in exchange for ULASOs withdrawal of the disbarment case against respondent attorney. However, the complaint for disbarment continued. IBP Committee on Bar Discipline rendered a resolution, finding respondent attorney guilty of gross negligence and violating the notarial law and recommended her 2-year suspension. ISSUE/S: Whether or not the notarization of the jurat of the amended verification and affidavit of non-forum shopping attached to the initiatory pleading even before the plaintiff-client has affixed her own signature amounts to censurable conduct on the part of the notary-counsel HELD: RULING: There was no malice on the part of respondent when she signed the amended verification and affidavit of non-forum shopping The word for was an indication that respondent did not intend to misrepresent the signature of BIDES. If indeed respondent wanted to deceive the court, she could have instead written the name Irene Bides or forged the signature of Bides. The respondent, by notarizing the document without the signature of Bides, was only anticipating that Bides would subsequently sign (because Bides had already signed the original verification and affidavit) The respondents notarizing the amended verification and affidavit of non-forum shopping in the absence of Bides as the affiant constituted a clear breach of the notarial protocol and is highly censurable. The jurat is that end part of the affidavit in which the notary CERTIFIES that the instrument is sworn to before her. As such, the notarial certification is essential. SUBSCRIBED AND SWORN TO BEFORE ME indicated the necessity for the physical presence of BIDES as the affiant and the fact that the signing was done in the presence of the respondent as the notary. The physical presence of Bides was required in order to have her as the affiant swear before the respondent that she was that person and in order to enable the respondent as the notary to ascertain whether Bides had voluntarily and freely executed the affidavit. Therefore, the respondent, by signing as notary even before Bides herself could appear before her, failed to give due observance and respect to the solemnity. Respondents duties are dictated by public policy and impressed with public interest. She could not disregard the requirements and solemnities of the Notarial Law. NOTE: However, the court opted to reprimand the respondent instead of suspending her from the practice. (Factors: Absent of bad faith, this was a first case against her, respondent was recuperating from stroke)

70. VALLACAR TRANSIT, INC. v. CATUBIG Rule 7 Sec 4 Verification Certificate of Agency FACTS:

Petitioner is engaged in the business of transportation and the franchise owner of a Ceres Bulilit bus. Quirino C. Cabanilla (Cabanilla) is employed as a regular bus driver of petitioner. Respondents husband, Quintin Catubig, Jr. (Catubig), was on his way home from Dumaguete City on a motorcycle. While approaching a curve, Catubig tried to overtake a slow moving ten-wheeler cargo truck by crossing-over to the opposite lane, which was then being traversed by the Ceres Bulilit bus driven by Cabanilla, headed for the opposite direction. When the two vehicles collided, Catubig and Emperado were thrown from the motorcycle. Catubig died on the spot where he was thrown. Cabanilla was charged with reckless imprudence resulting in double homicide before the (MCTC). It found that Cabanilla was not criminally liable for the deaths of Catubig and Emperado, because there was no negligence, not even contributory, on Cabanillas part. Respondent filed before the RTC a Complaint for Damages against petitioner, in relation to Article 2176, of the Civil Code. Respondent alleged that petitioner is civilly liable because the latters employee driver, Cabanilla, was reckless and negligent in driving the bus which collided with Catubigs motorcycle. Petitioner, in its Answer with Counterclaim, contended that the proximate cause of the vehicular collision, was the sole negligence of Catubig. As a special and affirmative defense, petitioner asked for the dismissal of respondents complaint for not being verified and/or for failure to state a cause of action, as there was no allegation that petitioner was negligent in the selection or supervision of its employee driver. RTC ruled that the husband is the reckless and negligent driver and not the driver of the [petitioner]. Respondent appealed to the Court of Appeals. It held that both Catubig and Cabanilla were equally liable for the accident in question. Petitioner asserts that respondents complaint for damages should be dismissed for the latters failure to verify the same. ISSUE: W/N the certification against forum shopping attached to the complaint, signed by respondent, is a valid substitute for respondents verification that she "has read the pleading and that the allegations therein are true and correct of her personal knowledge or based on authentic records." HELD: YES, it is a valid substitute (unless there is a law or rule specifically requiring a pleading to be verified). Respondent filed her complaint for damages against petitioner in 1995, when the 1964 Rules of Court was still in effect. Rule 7, Section 6 of the 1964 Rules of Court provided:

Sec. 6. Verification.A pleading is verified only by an affidavit stating that the person verifying has read the pleading and that the allegations thereof are true of his own knowledge. Verifications based on "information and belief," or upon "knowledge, information and belief," shall be deemed insufficient. However upon the effectivity of the 1997 Rules of Court, it provided that a pleading need not be under oath, verified or accompanied by affidavit, unless specifically required by law. The 1997 Rules of Court, even prior to its amendment by A.M. No. 00-2-10, clearly provides that a pleading lacking proper verification is to be treated as an unsigned pleading which produces no legal effect. However, it also just as clearly states that "except when otherwise specifically required by law or rule, pleadings need not be under oath, verified or accompanied by affidavit." No such law or rule specifically requires that respondents complaint for damages should have been verified. In contrast, all complaints, petitions, applications, and other initiatory pleadings must be accompanied by a certificate against forum shopping, first prescribed by Administrative Circular No. 04-94, which took effect on April 1, 1994, then later on by Rule 7, Section 5 of the 1997 Rules of Court. It is not disputed herein that respondents complaint for damages was accompanied by such a certificate. Verification like in most cases required by the rules of procedure, is a formal, not jurisdictional, requirement, and mainly intended to secure an assurance that matters which are alleged are done in good faith or are true and correct and not of mere speculation. When circumstances warrant, the court may simply order the correction of unverified pleadings or act on it and waive strict compliance with the rules in order that the ends of justice may thereby be served.

71. Michelle Lana Brown-Araneta v. Juan Ignacio Araneta Rule 7 Sec 5 Forum-shopping Facts: Juan Ignacio and Michelle were married in Las Vegas and thereafter had two children Ara and Ava. However, after 7 years of disharmonious relationship, husband and wife separated. Since the couples estrangement and de facto separation, Ara and Ava have remained in Michelles custody. Juan Ignacio filed a Petition for Custody, with prayer for visitation rights against Michelle and her motherin the RTC of Makati. Michelle filed an answer (with Affirmative Defenses and With Very

Urgent Ex-Parte Motion for Issuance of Protection Order). The Makati RTC resolved to deny admission of Michelles answer to the petition for custody and declared her in default. Michelle interposed a Motion to Withdraw Urgent Ex-Parte Motion for Protective Order, pointing out that no right of Juan Ignacio, if any, will be affected if the said urgent motion is withdrawn or expunged from her answer. Michelle then instituted a Petition for Protection Order before the RTC in Muntinlupa City. Muntinlupa RTC granted Michelles prayer for a TPO. Because of the order of Muntinlupa RTC, Juan Ignacio went to the CA on a petition for certiorari. The petition prayed that the Muntinlupa RTC be enjoined from further taking cognizance of Michelles protection order petition as the said case will infringe or intrude upon the Makati RTCs disposition of the custody case. CA found Michelle guilty of forum shopping. Michelle presently argues that there was no forum shopping when she filed her Petition for Protection Order in the Muntinlupa RTC while the custody case was pending in the Makati RTC. Her stated reason: the absence in both cases of identity of parties and rights asserted, on top of which the reliefs sought and prayed for are different and not founded on the same set of facts. Issue: W/N the petitioner is guilty of forum shopping Held: YES Ratio: It has been held that there is forum shopping (1) whenever as a result of an adverse decision in one forum, a party seeks a favorable decision (other than by appeal or certiorari) in another; or (2) if, after he has filed a petition before the Supreme Court, a party files another before the CA since in such case said party deliberately splits appeals in the hope that even as one case in which a particular remedy is sought is dismissed, another case (offering a similar remedy) would still be open; or (3) where a party attempts to obtain a preliminary injunction in another court after failing to obtain it from the original court. Michelle withdrew her Ex Parte Motion for Issuance of Protective Order in the custody case prior to her filing of her Petition for Protection Order with the Muntinlupa RTC. It should be made clear, however, that she filed said motion afterthe Makati RTC, in its Order had, for all intents and purposes, denied the said ex parte motion. As a result or in anticipation of an adverse ruling of the Makati RTC, petitioner sought the favorable opinion of the Muntinlupa RTC. The presiding judge of the Makati RTC, in the custody case, made of record that she was not inclined to issue a protection order in favor of Michelle because she did not bother to appear in Court and that the allegations against Juan Ignacio cannot, per se, prevent him from exercising visitation rights over his children.

The Petition for Custody and the Petition for Protection Order have the same parties who represent the same interests. The fact that Ava and Ara, who are parties in the Petition for Protection Order, are not impleaded in the Petition for Custody is of no moment because they are precisely the very subjects of the Petition for Custody and their respective rights are represented by their mother, Michelle. The rights asserted and reliefs prayed for in the case before the Muntinlupa RTC are practically based on the same facts and are so intertwined with that filed before the Makati RTC, such that any judgment rendered in the pending cases, regardless of which party is successful, will amount to res judicata. If the Makati RTC were to grant Juan Ignacios petition for custody, this would necessarily mean that it would be in the best interest of the children if he were allowed to visit and spend time with them and that granting Juan Ignacio visitation rights would not pose any danger or threat to the children.On the other hand, a grant by the Muntinlupa RTC of Michelles prayer for a permanent protection order would presuppose at the minimum that it would be to the childrens best interest if Juan Ignacio is directed to keep away from them, necessary implying that he is unfit even to visit Ara and Ava. Conversely, if Juan Ignacios Petition for Custody were denied, then it would mean that the Makati RTC gave weight and credence to Michelles allegations of abuse and found them to be in the best interest of the children to bar Juan Ignacio from visiting them. 72. THE PTA OF ST. MATTHEW CHRISTIAN ACADEMY VS. THE MBTC Rule 7 Facts: - Spouses Ilagan obtained a loan from Metropolitan Bank and Trust Co. (MBTC) amounting to 4.79 million with REM over parcels of land where St. Matthew Christian Academy (SMCA) is situated. - SMCA is a lessee of Spouses Ilagan on the land subject of REM, however, the former is practically owned by the latter. Terms of lease have expired 1999, but continued thereafter on a monthly basis. - Spouses Ilagan defaulted on the loan. An extrajudicial foreclosure was conducted with MBTC being the highest bidder. Certificate of Sale was issued. - During the period of redemption, MBTC filed an Ex-Parte Petition for Issuance of a Writ of Possession, posting the required bond which was subsequently approved. - SMCA filed a Petition for Injunction with Prayer for Restraining Order against MBTC and the Provincial Sheriff of Tarlac on the ground that it cannot be ejected being a third party.

- Further Parents-Teacher-Association (PTA) of SMCA and teachers filed for a Motion for Leave to file Petition in Intervention in Special Civil Action No. 9793 granted by the Court which was subsequently reversed because of the petitioners intervention would have no bearing on the issuance and implementation of the writ of possession. - PTA of SMCA, et. al. filed for Motion for Reconsideration, which was denied in a Resolution. Subsequently, they filed this Petition for Certiorari. Issues: 1. WON: CA erred when it refused to consider the petition for intervention based on the fact that petitioners are third parties in relation to the writ of possession being enforced upon in the exparte motion of MBTC. 2. WON: CA erred in not holding that considerations of Justice and Equity, wherein the court did not conduct trial for the presentation of evidence to support its conclusion that the intervention would have no bearing on the issuance and implementation of the writ of possession, thereby depriving them of due process. Ratio: 1. No. CA did not err in the setting aside the petition of PTA of SMCA because of the fact that PTA of SMCA are not third parties against whom the writ of possession cannot be issued and implemented. As a rule, it is ministerial upon the court to issue a writ of possession after the foreclosure sale and during the period of redemption. Section 7 of Act No. 3135 explicitly authorizes the purchaser in a foreclosure sale to apply for a writ of possession during the redemption period by filing an ex parte motion under oath for that purpose in the registration or cadastral proceedings if the property is registered, or in special proceedings in the case of property registered under the Mortgage Law with the Regional Trial Court of the province or place where the real property or any part thereof is situated, in the case of mortgages duly registered with the Registry of Deeds. Upon filing of such motion and the approval of the corresponding bond, the law also directs in express terms the said court to issue the order for a writ of possession. As an exception, the obligation of the trial court to issue a writ of possession ceases to be ministerial once it appears that there is a third party in possession of the property claiming a right adverse to that of the debtor/mortgagor. Where such third party exists, the trial court should conduct a hearing to determine the nature of his adverse possession. Petitioner-teachers and students did not claim ownership of the properties, but merely averred actual physical possess ion of the

subject school premises. Petitioner-teachers possession of the said premises was based on the employment contracts they have with the school. Besides, their contracts are with the school and do not attach to the school premises. Moreover, the foreclosure of the current school premises does not prevent the SMCA from continuing its operations elsewhere. Petitioners did not have superior rights to that of SMCA over the subject property because their supposed possession of the same emanated only from SMCA. Consequently, this affirms the issuance the writ of possession. 2. No. CA did not err in that consideration because as a general rule it is ministerial upon the court to issue a writ of possession after the foreclosure sale and during the period of redemption. Obiter: - The lack of authority to sign the certificate of non-forum shopping attached to the Petition for Issuance of Writ of Possession was an insignificant lapse. A certification on nonforum shopping is required only in a complaint or a petition which is an initiatory pleading. In this case, the subject petition for the issuance of a writ of possession filed by MBTC is not an initiatory pleading. Although private MBTC denominated its pleading as a petition, it is more properly a motion. What distinguishes a motion from a petition or other pleading is not its form or the title given by the party executing it, but its purpose. The purpose of a motion is not to initiate litigation, but to bring up a matter arising in the progress of the case where the motion is filed.It is not necessary to initiate an original action in order for the purchaser at an extrajudicial foreclosure of real property to acquire possession. Even if the application for the writ of possession was denominated as a petition, it was in substance merely a motion. Indeed, any insignificant lapse in the certification on non-forum shopping filed by the MBTC did not render the writ irregular. After all, no verification and certification on non-forum shopping need be attached to the motion. 73. SPOUSES DENNIS BARIAS v. BONEO Sec 5 FACTS: Heirs of BartolomeBoneo, are registered owners of a parcel of land in Albay, (covered by an OCT). They allege that the Spouses Dennis and DivinaBarias (petitioners) have been occupying a portion of the property for residential purposes on their mere tolerance, and that despite verbal demands and a written demand by letter petitioners refused to vacate the premises.

Petitioners charged respondents with forum shopping, claiming that the portion of the property subject of the complaint was also the subject of a case between petitioners and respondents predecessor-in-interest Silvestra Boneo (Silvestra) pending appeal before the Court of Appeals. The MCTC, which found respondents guilty of forum shopping, dismissed respondents complaint. On appeal to the (RTC), respondents denied that they are Silvestras successors-in-interest. They claimed that Silvestra was the second wife of Crispin Boneo and stepmother of the late Bartolome Boneo, their father and immediate predecessor-in-interest, hence, they can not be considered as the legal heirs or even successorsin-interest of Silvestra. They concluded that the Deed of Absolute Sale over the disputed portion of the property executed by Silvestra in favor of the herein petitioners has no binding effect upon them. While the RTC did not find respondents guilty of forum shopping, it nevertheless dismissed their appeal, holding that petitioners have a superior right to possess the property.On appeal, the Court of Appeals reversed the RTC decisionruling that it was error for both the RTC and MTC to have sustained Bariasclaim which was based on a deed of sale, as against the claim of Heirs of Boneo which was based on a free patent. ISSUE: W/N it was error for the RTC in not finding Boneos guilty of "forum shopping" warranting outright dismissal of their petition. HELD: NO. RATIO: The test in determining the presence of forum shopping is whether in two or more cases pending, there is identity of (1) parties, (2) rights or causes of action, and (3) reliefs sought. The case filed by Silvestra, which was pending when respondents filed the complaint for unlawful detainer, was for annulment of the deed of sale that she executed in favor of petitioner DivinaBarias mother. Thus, the causes of action of that case and respondents complaint for unlawful detainer subject of the present petition are different: the cause of action of the first is the alleged fraud in inducing Silvestra to execute the deed of sale, while the cause of action of the second is the alleged unlawful possession of petitioners of that portion of the property which was allegedly sold by Silvestra. The reliefs sought in both cases are likewise different. In an unlawful detainer case, the sole issue for resolution is physical or material possession of the property involved, independent of any claim of ownership by any of the parties. Where the issue of ownership is raised

by any of the parties, the courts may pass upon the same in order to determine who has the right to possess the property.The adjudication is, however, merely provisional and would not bar or prejudice an action between the same parties involving title to the property. As both parties raise the issue of ownership in the unlawful detainer case, its resolution boils down to which of their respective documentary evidence deserves more weight. Respondents have a Torrens title over the property which was issued in 1991. The age-old rule is that the person who has a Torrens title over a land is entitled to possession thereof. The deed of sale which was executed by Silvestra in 1994 and was the subject of a case for annulment could not affect the herein respondents-registered owners superior right to possess the property. It bears emphasis that this determination of ownership in an ejectment case is only initial and only for the sole purpose of settling the issue of possession.It does not prejudice the case for annulment of the deed of sale.

74. MID-PASIG LAND DEVT CORP VS. TABLANTE Rule 7 Facts: Mid-Pasig Land Development (petitioner) is the registered owner of a piece of land in Pasig City. Petitioner and ECRM Enterprises, represented by its proprietor, Mario Tablante, a lease agreement where Mid-Pasig leased a portion of land to ECRM. Tablante assigned his rights to Laurie Litam and/or Rockland Construction Company. Petitioner learned that Tablante had executed a Contract of Lease with respondent MC Home Depot, Inc. over the same parcel of land. MC Home Depot, Inc. constructed improvements on the land and leased portions to various entities. Petitioner demanded that respondents vacate the land. Rockland filed a case for Specific Performance with the RTC of Pasay City to compel petitioner to execute a new lease contract for another three (3) years. Petitioner filed a case for unlawful detainer against respondents. The MTC rendered judgment in the unlawful detainer (ejectment) case and held that the issue was whether or not ECRM had the right to exercise an option to renew its lease contract. This issue was incapable of pecuniary estimation and jurisdiction over the case was vested in the RTC. The RTC of Pasig City affirmed. A petition for certiorari was consequently filed with the CA.

CA dismissed the case on the ground, among others, that the verification and certification against non-forum shopping was signed by a certain Antonio A. Merelos as General Manager of the petitionercorporation without attaching therewith a Corporate Secretarys certificate or board resolution that he is authorized to sign for and on behalf of the petitioner.

Issue: W/N CA dismissal of the case on the ground of lack of authority to sign the certification and verification against non-forum shopping was proper. Held: No. Ratio: Sec. 23, in relation to Sec. 25 of the Corporation Code, clearly enunciates that all corporate powers are exercised by the board of directors. However, as provided in one case decided by the SC, the following officials or employees of the company can sign the verification and certification without need of a board resolution: (1) the Chairperson of the Board of Directors, (2) the President of a corporation, (3) the General Manager or Acting General Manager, (4) Personnel Officer, and (5) an Employment Specialist in a labor case.

The listing of authorized signatories to the verification and certification is not exclusive. Determination of the sufficiency of the authority was done on a case to case basis. The failure to attach the Secretarys Certificate, attesting to General Manager Antonio Mereloss authority to sign the Verification and Certification of Non-Forum Shopping, should not be considered fatal to the filing of the petition. The requisite board resolution was subsequently submitted to the CA, together with the pertinent documents. Petitioner substantially complied with the rules. The rules of procedure ought not to be applied in a very rigid, technical sense. 75. SAN MIGUEL BUKID HOMEOWNERS ASSOCIATION, INC. VS. CITY OF MANDALUYONG Doctrine: Certiorari may be resorted to when there is no appeal, nor any plain, speedy, and adequate remedy in the ordinary course of law.An order of dismissal is a final order which is a proper subject of an appeal, not certiorari. In cases where an appeal was available, certiorari will not prosper, even if the ground therefor is grave abuse of discretion. The

existence and availability of the right of appeal are antithetical to the availability of the special civil action for certiorari, although where it is shown that the appeal would be inadequate, slow, insufficient, and will not promptly relieve a party from the injurious effects of the order complained of, or where appeal is inadequate and ineffectual, the extraordinary writ of certiorari may be granted. Facts: San Miguel Bukid Homeowners Association, Inc., an association of urban poor dwellers of San Miguel Bukid Compound, Plainview, Mandaluyong City, filed with the RTC of MandaluyongCity a Complaint for specific performance and damages against respondents City of Mandaluyong and A.F. Calma General Construction. It is alleged therein that pursuant to the Citys Land for the Landless Program, San Miguel and the City entered into a MOA, whereby the City purchased lots and then transferred the same to San Miguel with a first real estate mortgage in favor of the City. Subsequently, the City and Calma entered into a Contract Agreement for Calma to construct row houses and medium-rise buildings on the aforementioned lots within 540 calendar days for the benefit of San Miguels members. Calma began construction, but in June 1996, work on the project was stopped. The period of 540 days elapsed sometime in November 1996, but the houses and buildings were not yet completed. San Miguels letters sent to the Mayor of the City requesting an update on the project remained unanswered. Hence, San Miguelfiled the complaint praying that the City and Calma be ordered to perform their respective undertakings and obligations under the Contract Agreement and to pay petitioner attorneys fees, exemplary damages and litigation expenses. The City filed an answer contending that the MOA had already been abrogated due to San Miguels failure to secure a loan from the Home Mortgage and Finance Corporation, and that petitioner had no standing or personality to institute the action, as it was not a party to the Contract Agreement. Calma did not file an Answer. San Miguel filed a Motion to Declare City in Default contending that the answer filed by the City was filed by a private counsel and not the office of the city legal officer which was the only office authorize under the local government code to represent local government unit in all civil cases. RTC denied this motion. San Miguel elevated the matter to CA via petition for certiorari. Issue: W/N petition for certiorari under Rule 65 is the proper remedy Held: No. Petition is doomed to fail petitioner resorted to an improper remedy, the present petition is already dismissible and undeserving of the Courts attention. Certiorari may be resorted to when there is no appeal, nor any plain, speedy, and adequate remedy in the ordinary course of law.An order of dismissal is a final order which is a proper subject of an appeal, not certiorari. In cases where an appeal was available, certiorari will not

prosper, even if the ground therefor is grave abuse of discretion. The existence and availability of the right of appeal are antithetical to the availability of the special civil action for certiorari, although where it is shown that the appeal would be inadequate, slow, insufficient, and will not promptly relieve a party from the injurious effects of the order complained of, or where appeal is inadequate and ineffectual, the extraordinary writ of certiorari may be granted. 76. HEIRS OF ARCILLA V. TEODORO G.R. NO. 162886 Facts: Teodoro initially filed with the RTC an application for land registration of 2 parcels of land located at Barangay San Pedro, Virac, Catanduanes. She alleged that, with the exception of the commercial building constructed thereon, she purchased the subject lots from her father, as shown by a Deed of Sale, who acquired the said lots by virtue of the partition of the estate of his father, Pacifico, evidenced by a document entitled Extrajudicial Settlement of Estate. Teodoro also presented as evidence an Affidavit of Quit-Claim in favor of Pacifico, executed by herein petitioners as Heirs of Vicente Arcilla (Vicente), brother of Pacifico. The case was transferred to the MTC of Virac, Catanduanes in view of the expanded jurisdiction of said court as provided under Republic Act No. 7691. In their Opposition, petitioners contended that they are the owners pro-indiviso of the subject lots including the building and other improvements constructed thereon by virtue of inheritance from their deceased parents, spouses Vicente and Josefa Arcilla. Subsequently, trial of the case ensued. Teodoro filed a Motion for Admission contending that through oversight and inadvertence she failed to include in her application, the verification and certificate against forum shopping required by SC Revised Circular No. 28-91 in relation to SC Administrative Circular No. 04-94. Petitioners filed a Motion to Dismiss Application on the ground that Teodoro should have filed the certificate against forum shopping simultaneously with the petition for land registration which is a mandatory requirement and that any violation shall be a cause for the dismissal of the application upon motion and after hearing. Opposing the motion to dismiss, Teodoro asserted that the petitioners' Motion to Dismiss Application was filed out of time; respondent's failure to comply with SC Administrative Circular No. 04-94 was not willful, deliberate or intentional; and the Motion to Dismiss was deemed waived for failure of petitioners to

file the same during the earlier stages of the proceedings. MTC denied petitioners' Motion to Dismiss Application and granted a decision in favor of Teodoro. Petitioners appealed in the RTC, which dismissed the appeal for lack of merit and affirmed in toto the Decision of the MTC. Petitioners filed a Motion for Reconsideration but was denied. Aggrieved by the RTC Decision, petitioners filed a Petition for Review with the CA. The CA dismissed the petition. Petitioners filed a Motion for Reconsideration but the same was denied.

Issues: 1. W/N the belated filing, after more than 2 years and 3 months from the initial application for land registration, of a sworn certification against forum shopping in Teodoros application for land registration, constitu ted substantial compliance with SC Admin. Circular No. 04-94. 2. W/N the certification of non-forum shopping subsequently submitted by respondent does not require a certification from an officer of the foreign service of the Philippines as provided under Section 24, Rule 132 of the Rules of Court. Held: 1. Yes. 2. Yes. Ratio: 1. The CA ruled correctly when it held that the belated filing of a sworn certification of non-forum shopping was substantial compliance with SC Administrative Circular No. 04-94. The Court is fully aware that procedural rules are not to be belittled or simply disregarded, for these prescribed procedures insure an orderly and speedy administration of justice.21 However, it is equally settled that litigation is not merely a game of technicalities. Rules of procedure should be viewed as mere tools designed to facilitate the attainment of justice. Their strict and rigid application, which would result in technicalities that tend to frustrate rather than promote substantial justice, must always be eschewed. Even the Rules of Court reflect this principle. In Uy v. Land Bank of the Philippines,No the Court ruled, thus: The admission of the petition after the belated filing of the certification, therefore, is not unprecedented. In those cases where the Court excused non-compliance with the requirements, there were special circumstances or compelling reasons making the strict application of the rule clearly unjustified. In the case at bar, the apparent merits of the substantive aspects of the case should be deemed as a "special circumstance" or "compelling reason" for the reinstatement of the petition. The Court may still proceed to decide the case on the merits pursuant to its inherent power to suspend its own rules on grounds of substantial justice and apparent merit of the case.

In the instant case, the Court finds that the lower courts did not commit any error in proceeding to decide the case on the merits, as Teodoro was able to submit a certification of non-forum shopping. More importantly, she showed that she had no intention to violate the Rules with impunity, as she was the one who invited the attention of the court to the inadvertence committed by her counsel, should be deemed as special circumstances or compelling reasons to decide the case on the merits. In addition, considering that a dismissal contemplated under Rule 7, Sec 5 is, as a rule, a dismissal without prejudice, and since there is no showing that respondent is guilty of forum shopping, to dismiss respondent's petition for registration would entail a tedious process of refiling the petition, etc.

of Court, is devoid of any merit. What is important is the fact that the respondent-applicant certified before a commissioned officer clothed with powers to administer oath that [s]he has not and will not commit forum shopping. 77. SPOUSES MELO V. CA FACTS Respondent Arsenia Coronel mortgaged to the Rural Bank of Mabalacat, Inc. a parcel of land in Angeles City. Because of her failure to pay the loan, the bank caused the extra-judicial foreclosure of the mortgage, and the land was sold to petitioners as the highest bidders. Petitioners Melo then filed a Petition for the Issuance of a Writ of Possession. To counter the petition, respondent filed a complaint for injuction against petitioners. Petitioners moved to dismiss respondent Coronels action on the ground of forum shopping and the failure to attach a certification of non-forum shopping to her complaint. Respondent later amended her complaint by including the certification of non-forum shopping. ISSUES 1. W/N respondent Coronel is guilty of forum shopping. NO 2. W/N the failure of respondent to attach a certification of nonforum shopping shall cause the dismissal of the complaint. YES HELD 1. There is no forum shopping. The petition for the issuance of a writ of possession was filed by the petitioners against respondent. On the other hand, the complaint with preliminary injunction was filed by the respondent against herein petitioners. It is not a case, therefore, of the private respondent instituting 2 remedies in two different fora. Her case entailed only one forum. To begin with, the essence of forum-shopping is the filing of multiple suits involving the same parties for the same cause of action, either simultaneously or successively, for the purpose of obtaining a favorable judgment. Petitioners sought possession of the subject property, whereas private respondent sought to enjoin them from consolidating title over the same. Respondent could very well oppose petitioners' action to obtain possession of the property while trying to prevent them from consolidating title in a separate case. The decision in one is not conclusive of the other.

2. The certification of non-forum shopping executed in a foreign country is not covered by Section 24, Rule 132 of the Rules of Court. There is no merit to petitioners contentions that the verification and certification subsequently submitted by respondent did not state the country or city where the notary public exercised her notarial functions; and that the MTC simply concluded, without any basis, that said notary public was from Maryland, USA; that even granting that the verification and certification of non-forum shopping were notarized in the USA, the same may not be deemed admissible for any purpose in the Philippines for failure to comply with the requirement of Section 24, Rule 132 of the Rules of Court that the notarized document must be accompanied by a certificate issued by an officer in the foreign service of the Philippines who is stationed in the country in which a record of the subject document is kept, proving or authenticating that the person who notarized the document is indeed authorized to do so and has custody of the same. The Court agrees with the disquisition of the CA, to wit: From the foregoing provision [referring to Section 24, Rule 132, Rules of Court], it can be gathered that it does not include documents acknowledged before [a] notary public abroad. For foreign public documents to be admissible for any purpose here in our courts, the same must be certified by any officer of the Philippine legation stationed in the country where the documents could be found or had been executed. However, after judicious studies of the rule, Sec. 24, Rule 132 of the 1997 Rules of Court basically pertains to written official acts, or records of the official of the sovereign authority, official bodies and tribunals, and public officers, whether of the Philippines, or of a foreign country. This is so, as Sec. 24, Rule 132 explicitly refers only to paragraph (a) of Sec. 19. If the rule comprehends to cover notarial documents, the rule could have included the same. Thus, petitioners-oppositors' contention that the certificate of forum shopping that was submitted was defective, as it did not bear the certification provided under Sec. 24, Rule 132 of the Rules

2. Respondent failed to comply with Administrative Circular No. 0994 on non-forum shopping and, therefore, her complaint should have been dismissed. Any clearly willful and deliberate forum shopping by any party and his counsel through the filing of multiple complaints shall be a ground for summary dismissal shall constitute direct contempt of court. The submission of a false certification shall constitute indirect contempt of court. The requirement to file a certificate of non-forum shopping is mandatory. Failure to comply with this requirement cannot be excused by the fact that plaintiff is not guilty of forum shopping. The compliance with the certification against forum shopping is separate from, and independent of, the avoidance of forum shopping itself. The former is merely a cause for the dismissal, without prejudice, of the complaint or initiatory pleading, while the latter is a ground for summary dismissal thereof and constitutes direct contempt.

During the scheduled ex parte hearing on August 18, 2000, De Konings counsel appeared and manifested that he filed a motion to dismiss on the ground that Metrobanks petition violated Section 5, Rule 7 of the Rules of Court (Rules)[4] which requires the attachment of a certification against forum shopping to a complaint or other initiatory pleading. According to De Koning, Metrobanks petition for the issuance of a writ of possession involved the same parties, the same issues and the same subject matter as the case he had filed on October 30, 1998 with the RTC of Makati,[5] to question Metrobanks right to foreclose the mortgage. De Koning also had a pending petition for certiorari with the CA,[6] which arose from the RTC case he filed. When Metrobank failed to disclose the existence of these two pending cases in the certification attached to its petition, it failed to comply with the mandatory requirements of the Rules so that its petition should be dismissed. The RTC agreed with De Koning and dismissed Metrobanks petition in its September 18, 2000 order on the ground De Koning cited, i.e.,for having a false certification of non-forum shopping. The lower court denied Metrobanks motion for reconsideration. Metrobank thus elevated the matter to the CA on a petition for certiorari on January 5, 2001. The CA affirmed the dismissal of Metrobanks petition. It explained that Section 5, Rule 7 of the Rules is not limited to actions, but covers any initiatory pleading that asserts a claim for relief. Since Metrobanks petition for writ of possession is an initiatory pleading, it must perforce be covered by this rule. Thus, Metrobanks failure to disclose in the verification and certification the existence of the two cases filed by De Koning, involving the issue of Metrobanks right to foreclose on the property, rendered the petition dismissible.

78. MBTC v. Rule 7, Rule 15!

Hon.

Salvador

Santos

Manfred De Koning obtained a loan from Metrobank in the amount of Php 2,019,000.00 as evidenced by a promissory note. To secure payment, De Koning executed a real estate mortgage in favor of Metrobank over a condominium unit in Makati along with all its improvements. When De Koning failed to pay his loan despite demand, Metrobank instituted extrajudicial foreclosure proceedings against the REM. Metrobank was the highest bidder at the public auction of the condominium unit held on November 24, 1998 and a Certificate of Sale was issued in the bank's favor. Metrobank duly registered this Certificate of Sale with the Registry of Deeds for Makati City on January 18, 2000. The redemption period lapsed without De Koning redeeming the property. Thus, Metrobank demanded that he turn over possession of the condominium unit. When De Koning refused, Metrobank filed on July 28, 2000 with the RTC Makati, Branch 65, an ex parte petition for a writ of possession over the foreclosed property, pursuant to Act No. 3135, as amended. On August 1, 2000, the lower court issued an order setting the ex parte hearing of Metrobanks petition and directing that a copy of the order be given to De Koning to inform him of the existence of the proceedings.

79. HERNANDO BORRA v. CA Rule 7 Sec 5 Forum Shopping; Rule 16 Res judicata! FACTS: Petitioners filed with the NLRC Regional Arbitration Branch No. VI in Bacolod two separate complaints docketed as RAB Case No. 06-0910698-97 (Case 1) and RAB Case No. 06-09-10699-97 (Case 2). In Case 1, petitioners asked that they be recognized and confirmed as regular employees of Hawaiian and further prayed that they be awarded various benefits received by regular employees for three (3) years prior to the filing of the complaint. In Case 2, petitioners sought for payment of unpaid wages, holiday pay, allowances, 13th month pay, service incentive leave pay, moral

and exemplary damages also during the three (3) years preceding the filing of the complaint. Hawaiian filed a Motion to Consolidate the cases, but the Labor Arbiter in charge of the case denied it. It also filed a Motion to Dismiss Case 1 on the ground of res judicata. Hawaiian contended that a previous case (Perez case), already final and executory, bars the litigation of Case 1, because it was ruled therein that petitioners are not its employees but of Castillon. CA affirmed the NLRCs decision and denied the subsequent motion for reconsideration. SC likewise affirmed the Decision of the CA. CA found that Perez case pertains to Borra et al. employment from 1987 to 1995, while the instant case covers a different (subsequent) period. Moreover, in Perez, the finding that no employer-employee relationship existed between petitioner and private respondents was premised on absence of privity between Castillon and petitioner. Consequently, Perez and the instant case involve different subject matters and causes of action. Resolution of the case at bench would hinge on the nature of the relationship between petitioner and Fela Contractor. In other words, the action for declaration as regular employees of petitioner will not succeed unless it is established that Fela Contractor is merely a "labor-only" contractor and that Hawaiian is their real employer. In the meantime, the Labor Arbiter rendered a Decision in Case 2, holding that there is no employer-employee relations between Hawaiian and petitioners. No appeal was taken thus, the same became final and executory. Hawaiian again filed a Motion to Dismiss Case 1 on the ground, among others, of res judicata. It contended that the final and executory Decision of the Labor Arbiter in Case 2 serves as a bar to the further litigation of Case 1. The MD was denied. Hawaiian filed a petition for certiorari and prohibition with the CA assailing the Order of the Labor Arbiter. CA granted the petition. Borra et al. filed an MR but it was denied.

Issue 1 (Res Judicata) In Case 1 and 2, petitioners real employer has to be determined. The Courts cannot proceed to decide w/n they are considered as regular employees and thus entitled to benefits, if they are not employees of Hawaiian. Petitioners prayer for regularization in Case 1 and benefits in Case 2 is dependent on the existence of employeremployee relationship. As earlier mentioned, this issue has already been settled in Case 2 because the petitioners real employer is not Hawaiian. The final judgment in Case 2 has the effect and authority of res judicata by conclusiveness of judgment (discussion of res judicata omitted, please check orig case). Hence, there is no point in determining the main issue raised in Case 2. Therefore, the Labor Arbiter committed grave abuse of discretion in denying Hawaiians motion to dismiss Case 1. Issue 2 (Forum shopping) There can be no forum shopping, because the grounds cited by private respondent in its motions to dismiss filed in 1998 and in the present case are different. In 1998, the motion to dismiss is based on the argument that the final and executory decision in the Perez case serves as res judicata. In the instant case, private respondent again cites res judicata as a ground for its motion to dismiss but this time in Case 2. Thus, the relief prayed for in private respondent's motion to dismiss subject of the instant case is founded on totally different facts and issues.

80. PANGANIBAN v. PILIPINAS SHELL RULE 7; RULE 16 (E) Litis Pendencia! G.R. No. 131471 January 22, 2003 FACTS: Carmelita Panganiban (petitioner) entered into a Sublease and Dealer Agreement(SLDA) with Pilipinas Shell Petroleum Corporation (private respondent). Through theSLDA, private respondent subleased to petitioner a gasoline station located at 427 Samson Road, EDSA,Caloocan City. The period of sublease is stipulated as follows: This Agreement maybe terminated by SHELL at any time during the at any first six (6) months from the dateof approval by ERB of the application of the DEALER to operate this station, x xxIf thisagreement is not terminated during the first six (6) months, it shall continue to be in effect for another periodof 4 years, unless otherwise terminated xxx.The parties agree that

ISSUES: 1. Whether Case 2 bars the litigation of Case 1 on the ground of res judicata. 2. Whether Hawaiian is guilty of forum shopping considering that it already filed a motion to dismiss Case 1 in 1998. HELD: 1. YES 2. NO. RATIO:

thisAgreement is, however, co-terminus with SHELL's lease on the site referred to under paragraph 1 of thisAgreement notwithstanding the total 5-year period aforementioned. The Lease Agreementwas effective from January 1, 1987 to December 31, 2002 or for a period of 15 years. Before the SLDAs expiration, private respondent notified petitioner that the SLDA was expiring. Private respondent then advised petitioner to wind up her business on or before July 31, 1995. Believing that the SLDA had not yet expired and was still effective until December 31, 2002, petitioner continued topay rentals for the gasoline station. Private respondent refused to accept the payments. On July 10, 1995, petitioner filed a petition for declaratory relief with Branch 137 of the Regional Trial Court of Makati City. On August 30, 1995, private respondent filed its Answer. On April 26, 1996, private respondent filed an unlawful detainer case against petitioner with the Metropolitan TrialCourt of Caloocan City. Private respondent filed a Manifestation with Motion to Dismiss, claiming thatthe issue of the renewal of the lease should be raised in the unlawful detainer case pending before the MTC. MTC issued its Decision in the unlawful detainer case in favor of private respondent. This was appealed to the RTC. On the other hand, RTC ordered the dismissal of the petition for declaratory relief. Petitioner filed motion for reconsideration of the Order, which was denied due to petitioners failure to appear at the hearing. Petitioner then filed a petition for review under Rule 45 of the Rules of Court with the Supreme Court, assailing the validity of the RTCs order to dismiss the declaratory relief. This petition was referred to the CA. CA denied the petition for certiorari.

ISSUE: Whether CA erred in affirming RTCs dismissal of the declaratory relief on motion of Shell on the ground of litispendentia which was filed long after Shell had filed its answer. HELD: No, the dismissal was proper. RATIO: Petitioner points out that private respondent filed the motionto dismiss some eight months after it had already filed an answer in the action for declaratoryrelief. This, according to petitioner, is a violation of Section 1, Rule 16 of the Rules of Court mandating that themotion to dismiss must be filed within the time for but before the filing of the answer to the complaint.

The requirement that a motion to dismiss should be filed within the time for filing the answer is not absolute. Evenafter an answer has been filed, a defendant can still file a motion to dismiss on the following grounds: (1) lack ofjurisdiction, (2) litispendentia (3) lack of cause of action, and (4) discovery during trial of evidence that wouldconstitute a ground for dismissal. Litispendentia is also one of the grounds that authorize a court to dismiss acase motuproprio. In this case,the bona fide existence of litispendentia is beyond dispute. The following requisites of litispendentia are presentin this case: (a) the identity of parties, or at least such as representing the same interests in both actions; (b) theidentity of rights asserted and relief prayed for, the relief being founded on the same facts; and (c) the identity ofthe two cases such that judgment in one, regardless of which party is successful, would amount to res judicata inthe other. Litispendentia as a ground for the dismissal of a civil action refers to that situation wherein another action ispending between the same parties for the same cause of actions and that the second action becomes unnecessary and vexatious. We have set the relevant factors that a court must consider when it has todetermine which case should be dismissed given the pendency of two actions. These are: "(1) the date of filing, with preference generally given to the first action filed to be retained; (2) whether the action sought to be dismissed was filed merely to preempt the latter action or to anticipateits filing and lay the basis for its dismissal; and (3) whether the action is the appropriate vehicle for litigating the issues between the parties." The mere fact that the action for declaratory relief was filed earlier than the case for unlawful detainer does notnecessarily mean that the first case will be given preference. Clearly, the interpretation of a provision in the SLDA as to when the SLDA would expire isthe key issue that would determine petitioner's right to possess the gasoline service station. When the primaryissue to be resolved is physical possession, the issue should be threshed out in the ejectment suit, and not in anyother case such as an action for declaratory relief to avoid multiplicity of suits. 81. RALLOS v. CITY OF CEBU GR No. 202651August 28, 2013 Rule 7 Sec 5 Forum Shopping; Rule 16 (e) Litis Pendencia! FACTS Lots 485-D and 485-E of the Baniad Estate, Cebu City, registered under the predecessors-in-interest of petitioner Rallos, were expropriated to be used as a public road in 1963. The heirs of Fr. Rallos sought for just compensation. The City of Cebu contended that the subject parcels of land are road lots and are not residential in character and that they have been withdrawn from the commerce of men and were occupied by Cebu without

expropriation proceedings pursuant to Ordinance No. 416, which was enacted 35 years before the Heirs of Rallos instituted their complaint. Many various motions and writs were filed and issued but the Courts ultimately ruled in favor of the heirs of Rallos, making the City of Cebu liable to pay the FMV of the subject properties. However, in response to the Mayors query, COA Regional Director Aguilar wrote the former a letter, opining that under AC 10-2000 issued by the SC, it was clearly stated that the prosecution, enforcement or satisfaction of state liability must be pursued in accordance with the rules and procedures laid down in PD 1445 (Government Auditing Code of the Philippines), providing that all money claims against the government must first be filed with the COA. The City of Cebu filed before the CA a Petition for Annulment of Final Decision/s and Order/s with prayer for the issuance of injunctive reliefs, contending that that the act of the Heirs of suppressing the existence of the Convenio amounted to extrinsic fraud which would justify the annulment of the RTCs decisio ns and stressed that it had already paid the heirs Php56M. The procedures prescribed were not yet complied with hence, public funds cannot be released notwithstanding the rendition of the decisions and issuance of orders by the RTC. CA granted the petition. Lucena Rallos, petitioner, filed 6 various petitions for indirect contempt before 4 different branches of the RTC of Cebu City and the SC, all of which in relation to the civil case at bar.

two cases, such that any judgment that may be rendered in the pending case, regardless of which party is successful, would amount to res judicata in the other case. In the Verification and Non-Forum Shopping Certification attached to the instant petition and executed by Lucena, she admitted that there are 5 other pending actions for indirect contempt which she filed relative to the civil case, but claimed that the issues are different from one another. Her claim cannot be sustained. There is a confluence of the requisites in the case at bar. Litis pendentia does not require the exact identity of parties in the actions. The officials of the City of Cebu do not in any way represent any interest distinct or separate from that of the latter. The reliefs sought in the petitions are founded on the same set of facts. Citation for indirect contempt in either the instant petition or the other petitions filed would amount to res judicata in the other considering the identities of parties and issues involved. Once there is a finding of forum shopping, the penalty is summary dismissal not only of the petition pending before this Court, but also of the other case that is pending in a lower court. This is so because twin dismissal is a punitive measure to those who trifle with the orderly administration of justice. Even if in the higher interest of justice, SC were to be exceptionally liberal and gloss over Lucenas act of forum shopping, the instant petition would still be susceptible for dismissal. The exercise and enforcement of rights as regards the expropriation by the government are subject to compliance with the requirements provided for by law to protect public funds. CASE DISMISSED. 82. PILAR DEVELOPMENT CORP. V. THE HON. CA Rule 7 Sec 5 Forum Shopping in relation to Res Judicata FACTS: Sps Ng and Lopez acquired land in Las Pinas from Dumbrique on 1977 and was given TCTs Fusilero filed an adverse claim against the Ngs and Lopezs, but was denied, both the SC and the CA affirmed the CFIs rulling that the land was owned by the Sps Ng and Lopez Lopez sold their land to Martel However, while the Fusilero case was pending, the Factors sold to Petitioner the same land in 1975 claiming that they got it from their parents and has been living OCEN for 30yrs and filed for an application for confirmation of title. Eventually, in 1994, the RTC awarded the land to the Lopez and Ngs, This decision was not appealed.

ISSUES Whether the rules on litis pendentia and forum shopping are a bar to the granting of the petition HELD YES RATIO: Forum shopping is the act of litigants who repetitively avail themselves of multiple judicial remedies in difficult for a, simultaneously or successively, all substantially founded on the same transactions and the same essential facts and circumstances; and raising substantially similar issues either pending in or already resolved adversely by some other court; or for the purpose of increasing their chances of obtaining a favorable decision, if not in one court, then in another. It exists when the elements of litis pendentia are present or where a final judgment in one case will (1) identity of parties, or at least such parties as those representing the same interests in both actions; (2) identity of rights asserted and reliefs prayed for, the reliefs being founded on the same facts; and (3) identity with respect to the two preceding particulars in the

Instead of appealing the RTC judgement, the Factors filled for a new Complaint for annulment of judgement alleging that the TCT of Lopez was spurious However this was denied by the courts because the dismissal of their Application (non-appeal) made this complaint moot and academic and barred by Res judicata

because the promissory notes on which its complaint was based were subject to a condition that did not occur. Respondent was the counsel of Ciudad Real Development Inc. (CRDI). In 1994, Pentacapital Realty Corporation (Pentacapital Realty) offered to buy parcels of land known as the Molino Properties, owned by CRDI. As the Molino Properties were the subject of a pending case, Pentacapital Realty paid only the down payment amounting to P12,000,000.00. However, the Molino Properties continued to be haunted by the seemingly interminable court actions initiated by different parties which thus prevented respondent from collecting his commission. On motion of respondent, the Regional Trial Court (RTC) allowed him to file a Third Party Complaint against CRDI. Admittedly, respondent earlier instituted an action for Specific Performance (later characterized by respondent as Preliminary Mandatory Injunction) against Pentacapital Realty for the payment of his commission on the sale of the Molino Properties. Acting on Pentacapital Realtys Motion to Dismiss, the RTC dismissed the case for lack of cause of action. The dismissal became final and executory. With the dismissal of the aforesaid case, respondent filed a Motion to Permit Supplemental Compulsory Counterclaim. The RTC allowed the filing of the supplemental counterclaim. On appeal, the CA sustained the allowance of the supplemental compulsory counterclaim based on the allegations in respondents pleading. The CA further concluded that there was a logical relationship between the claims of petitioner in its complaint and those of respondent in his supplemental compulsory counterclaim. Petitioner contends that respondents counterclaim is barred by res judicata. Respondent, on the other hand, avers that petitioner is guilty of forum-shopping by filing the petition for review (G.R. No. 181482) despite the pendency of G.R. No. 171736 assailing the CA Decision. ISSUES:

ISSUE: W/o The case should be dismissed because of res judicata HELD: YES RATIO: The fundamental principle behind the doctrine of res judicata is that parties ought not to be permitted to litigate the same issue more than once. That is, when a right or a fact has been judicially tried and determined by a court of competent jurisdiction, or an opportunity for such trial has been given, the judgment of the court so long as it remains unreversed should be conclusive upon the parties and those in privity with them in law or estate In order that there may be res judicata, it is requisite (a) that the former judgment is final; (b) that it has been rendered by a court of competent jurisdiction; (c) that it is a judgment on the merits; and (d) that, between the first and the second actions, there is identity of parties, subject-matter, and cause of action The lower courts have already passed judgement on their rightful ownership of the lands and were left unappealed by the parties. Lastly, it must be stressed that petitioners act of filing multiple suits involving the same parties and the same cause of action for the purpose of obtaining a favorable judgment amounts to forum-shopping, which by itself is already a valid ground to deny the instant Petition. 83. PENTACAPITAL INVESTMENT CORPORATION v. MAHINAY G.R. No. 171736, July 5, 2010 Rule 7 Sec 5 Forum Shopping in relation to Rule 16 Sec 1 (e) Res Judicata! FACTS: Petitioner filed a complaint for a sum of money against respondent Makilito Mahinay based on two separate loans obtained by the latter, amounting to P1,520,000.00 and P416,800.00, or a total amount of P1,936,800.00. In his Answer with Compulsory Counterclaim, respondent claimed that petitioner had no cause of action

1) Whether respondents counterclaim is barred by res judicata 2) Whether petitioner is guilty of forum-shopping HELD: 1. Yes. The requisites of res judicata are: (1) The former judgment or order must be final; (2) It must be a judgment on the merits; (3) It must have been rendered by a court having jurisdiction over the subject matter and the parties; and (4) There must be between the first and second actions, identity of parties, subject matter, and

cause of action. These requisites are present in the instant case. The RTC already ruled that respondent had no cause of action against Pentacapital Realty, there being no privity of contract between them. Instead of assailing the said Order, respondent filed his supplemental compulsory counterclaim, demanding again the payment of his commission, this time, against petitioner in the instant case. The Order, therefore, became final and executory. 2. No. Forum-shopping is the act of a litigant who repetitively availed of several judicial remedies in different courts, simultaneously or successively, all substantially founded on the same transactions and the same essential facts and circumstances, and all raising substantially the same issues, either pending in or already resolved adversely by some other court, to increase his chances of obtaining a favorable decision if not in one court, then in another. More particularly, the elements of forum-shopping are: (a) identity of parties or at least such parties that represent the same interests in both actions; (b) identity of rights asserted and reliefs prayed for, the relief being founded on the same facts; (c) identity of the two preceding particulars, such that any judgment rendered in the other action will, regardless of which party is successful, amount to res judicata in the action under consideration. These elements are not present in this case. In G.R. No. 171736, petitioner assails the propriety of the admission of respondents supplemental compulsory counterclaim; while in G.R. No. 181482, petitioner assails the grant of respondents supplemental compulsory counterclaim. In other words, the first case originated from an interlocutory order of the RTC, while the second case is an appeal from the decision of the court on the merits of the case. There is, therefore, no forum-shopping for the simple reason that the petition and the appeal involve two different and distinct issues.

RULE 7 Sec 5 FORUM SHOPPING, RULE 16 Sec 1 (E) a LITIS PENDENTIA, OR RULE 16 Sec 1 (G )RES JUDICATA! Facts: FR Cement Corporation (FRCC), owner/operator of a cement manufacturing plant, issued several withdrawal authorities for the account of cement dealers and traders, Fil-Cement Center and Tigerbilt. These withdrawal authorities state the number of bags that the dealer/trader paid for and can withdraw from the plant. Fil-Cement Center and Tigerbilt, through their administrative manager, Gail Borja (Borja), sold the withdrawal authorities covering 50,000 bags of cement to Co for the amount of P3.15 million Subsequently, Co sold these withdrawal authorities to Lim allegedly at the price of P3.2 million. Using the withdrawal authorities, Lim withdrew the cement bags from FRCC on a staggered basis. She successfully withdrew 2,800 bags of cement, and sold back some of the withdrawal authorities, covering 10,000 bags, to Co. At some time, FRCC did not allow Lim to withdraw the remaining 37,200 bags covered by the withdrawal authorities. Co and Borja explained that the plant implemented a price increase and would only release the goods once Lim pays for the price difference or agrees to receive a lesser quantity of cement. Lim objected and maintained that the withdrawal authorities she bought were not subject to price fluctuations. Lim sought legal recourse after her demands for Co to resolve the problem and to return the money which she has paid. The criminal case An Information for Estafa through Misappropriation or Conversion was filed against Co. The RTC of Pasig City, acquitted Co of the estafa charge for insufficiency of evidence. After the trial on the civil aspect of the criminal case, the Pasig City RTC also relieved Co of civil liability to Lim. Lim sought a reconsideration of the above Order. The trial court denied the motion. Lim filed her notice of appeal on the civil aspect of the criminal case to the CA.

84. LILY LIM vs.KOU CO PING a.k.a. CHARLIE CO G.R. No. 175256 | August 23, 2012

The civil action for specific performance Lim filed a complaint for specific performance and damages RTC of Manila. The defendants in the civil case were Co and all other parties to

the withdrawal authorities, Tigerbilt, Fil-Cement Center, FRCC, Southeast Asia Cement, and La Farge Corporation. The complaint asserted two causes of action: breach of contract and abuse of rights. Motions to dismiss both actions In reaction to the filing of the civil complaint for specific performance and damages, Co filed motions to dismiss the said civil case and Lims appeal in the civil aspect of the estafa case. He maintained that the two actions raise the same issue, which is Cos liability to Lim for her inability to withdraw the bags of cement, and should be dismissed on the ground of lispendens and forum shopping. Issue: Did Lim (private complainant) commit forum shopping in filing the civil case for specific performance and damages during the pendency of her appeal on the civil aspect of the criminal case for estafa? Held: NO. Ratio: A single act or omission that causes damage to an offended party may give rise to two separate civil liabilities on the part of the offender (1) civil liability ex delicto, that is, civil liability arising from the criminal offense under Article 100 of the Revised Penal Code, and (2) independent civil liability, that is, civil liability that may be pursued independently of the criminal proceedings. The independent civil liability may be based on "an obligation not arising from the act or omission complained of as a felony," as provided in Article 31 of the Civil Code (such as for breach of contract or for tort). It may also be based on an act or omission that may constitute felony but, nevertheless, treated independently from the criminal action by specific provision of Article 33 of the Civil Code ("in cases of defamation, fraud and physical injuries"). Because of the distinct and independent nature of the two kinds of civil liabilities, jurisprudence holds that the offended party may pursue the two types of civil liabilities simultaneously or cumulatively, without offending the rules on forum shopping, litis pendentia, or res judicata. One of the elements of res judicata is identity of causes of action. In the instant case, it must be stressed that the action filed by petitioner is an independent civil action, which remains separate and distinct from any criminal prosecution based on the same act. Not being deemed instituted in the criminal action based on culpa criminal, a ruling on the

culpability of the offender will have no bearing on said independent civil action based on an entirely different cause of action, i.e., culpa contractual. In the same vein, the filing of the collection case after the dismissal of the estafa cases against the offender did not amount to forumshopping. The essence of forum shopping is the filing of multiple suits involving the same parties for the same cause of action, either simultaneously or successively, to secure a favorable judgment. Although the cases filed by [the offended party] arose from the same act or omission of [the offender], they are, however, based on different causes of action. The criminal cases for estafa are based on culpa criminal while the civil action for collection is anchored on culpa contractual. Moreover, there can be no forum-shopping in the instant case because the law expressly allows the filing of a separate civil action which can proceed independently of the criminal action. Thus, Civil Case involves only the obligations arising from contract and from tort, whereas the appeal in the estafa case involves only the civil obligations of Co arising from the offense charged (civil action ex delicto). They present different causes of action, which under the law, are considered "separate, distinct, and independent" from each other. Both cases can proceed to their final adjudication, subject to the prohibition on double recovery under Article 2177 of the Civil Code. 85. SALUDAGA v. COMELEC AND BALAG 191120 April 7, 2010 G.R. Nos. 189431 and

RULE 7; RULE 39! FACTS: Quintin B. Saludaga and respondent ArtemioBalag were candidates for Mayor of Lavezares, Northern Samar in the May 2007 elections. Saludaga was declared the winner by a margin of 635 votes over Balag Balag filed an election protest against Saludaga before the RTC of Allen, Northern Samar, Branch 23. Saludaga, too, filed a counterprotest. RTC declared Balag as the winning candidate. Pending appeal to the COMELEC, Balag moved for execution of RTC Decision but was denied. Second Division of COMELEC also found Balag as the winning candidate. Balag again filed a Motion for Execution Pending Motion for Reconsideration. Saludaga on his part filed a Verified Motion for Reconsideration. On September 4, 2009, COMELEC subsequently granted Banags Motion for Execution Pending Motion for Reconsideration. The

order directed Saludaga to cease and desist from discharging the powers and duties of Mayor of Lavezares. Saludaga filed an Extremely Urgent Motion for Reconsideration with the COMELEC en banc on September 8, 2009. Later, Saludaga also filed a Petition for Certiorari with the SC challenging the September 4, 2009 Order. Thus, Balag filed a Manifestation and Motion to Dismiss with the COMELEC en banccontending that Saludaga engaged in forum shopping. Balag in the meantime had taken his oath and assumedthe post of Mayor of Lavezares, Northern Samar. COMELEC en banc granted the Motion to Dismiss, basing its resolution solely on a finding that Saludaga committed forum shopping when he filed a motion for reconsideration with the Second Division and a petition for certiorari in the SC, both to question the Order deted September 4, 2009.

Futhermore, in determining whether a party violated the rule against forum shopping, the most important factor to ask is whether the elements of litispendencia are present, or whether a final judgment in one case will amount to res judicata in another. For the principle of res judicata to apply, the following elements must be present: (1) the judgment sought to bar the new action must be final; (2) the decision must have been rendered by a court having jurisdiction over the subject matter and the parties; (3) the disposition of the case must be a judgment on the merits; and (4) there must be as between the first and second actions, identity of parties, subject matter, and cause of action. In the present case, the second element is wanting. Pursuant to Section 5, Rule 3 of the Comelec Rules of Procedure, the order, being interlocutory, shall be resolved by the division which issued it, not by COMELEC en banc. Hence, COMELEC en banc does not have jurisdiction over the petition. 2. COMELEC Rules of Procedures does not contain an express provision on execution pending appeal or resolution for reconsideration. Thus, Rules of Civil Procedures shall apply. Pertinently, Section 2 (a), Rule 39 of the 1997 Rules of Civil Procedure, as amended, provides: SEC. 2.Discretionary execution.- (a) Execution of a judgment or final order pending appeal. On motion of the prevailing party with notice to the adverse party filed in the trial court while it has jurisdiction over the case and is in possession of either the original record or the record on appeal, as the case may be, at the time of the filing of such motion, said court may, in its discretion, order execution of a judgment or final order even before the expiration of the period to appeal. After the trial court has lost jurisdiction, the motion for execution pending appeal may be filed in the appellate court. Discretionary execution may only issue upon good reasons to be stated in a special order after due hearing. Pursuant to Item 6 of COMELEC Resolution No. 8654, after the lapse of the 10-day period, the only power (and duty) that a division has is to certify and elevate the case, together with all the records, to the Commission en banc, for appropriate action. Hence, upon the lapse of the 10-day period or after August 23,

ISSUE: 1. Whether Saludaga violated the rule on forum shopping 2. Whether the order dated September 4, 2009, granting execution pending resolution of the motion for reconsideration is void. HELD: 1. No, there was no violation of the rule on forum shopping 2. Yes, the September 4, 2009 order is VOID. RATIO: 1. Forum shopping is the institution of two (2) or more suits in different courts, either simultaneously or successively, in order to ask the courts to rule on the same or related causes and/or to grant the same or substantially the same reliefs. There is forum shopping when as a result of an adverse decision in one (1) forum, or in anticipation thereof, a party seeks favorable opinion in another forum through means other than appeal or certiorari. Under paragraph 2, Section 5, Rule 7 of the 1997 Rules of Civil Procedure, as amended, if the acts of the party or his counsel clearly constitute willful and deliberate forum shopping, the same shall be ground for summary dismissal with prejudice and shall constitute direct contempt, as well as a cause for administrative sanctions. Based on the above definition, the mere filing of a separate case, as in the original action for certiorari and prohibition filed by petitioner, after filing a responsive pleading in the other case, does not necessarily constitute forum shopping.

2009, the Second Division no longer had jurisdiction to rule on respondents motion for execution. Having done so, the September 4, 2009 Order is void for having been issued by the COMELEC, Second Division without jurisdiction. Furthermore, even if said Order was promulgated within 10 days from the filing of the motion for execution, it would still be void because Presiding Commissioner Ferrer alone signed it.

The Court of Appeals issued a Resolution granting Landheights a new period of ten (10) days within which to correct and rectify the deficiencies in the petition. Landheights filed its Manifestation of Compliance. Mediserv filed a motion for reconsideration but was denied by the appellate court, hence this petition. Petitioner cites Section 5, Rule 7 of the 1997 Rules of Civil Procedure, which provides that failure to comply with the requirements on certification against forum shopping shall not be curable by mere amendment of the complaint or other initiatory pleading but shall be cause for dismissal of the case. Petitioner asserts that the appellate court acted with grave abuse of discretion amounting to lack or in excess of jurisdiction in reinstating the petition for review filed by respondent corporation. ISSUE: 3. W/N the CA gravely abused its discretion and acted without and/or in excess of jurisdiction in reinstating the petition despite the clear mandate of the rules as well as the jurisprudence for the dismissal of the said petition. 4. W/N failure to comply with the requirements on certification against forum shopping is not be curable by mere amendment of the complaint or other initiatory pleading but shall be cause for dismissal of the case. HELD: 1. NO 2. YES unless there are compelling reasons RATIO: 1. The instant petition was filed under Rule 65 of the 1997 Rules of Civil Procedure, which requires the existence of grave abuse of discretion. Grave abuse of discretion exists where an act of a court or tribunal is performed with a capricious or whimsical exercise of judgment equivalent to lack of jurisdiction. The abuse of discretion must be so patent and gross as to amount to an evasion of a positive duty or to a virtual refusal to perform a duty enjoined by law, or to act at all in contemplation of law, as where the power is exercised in an arbitrary and despotic manner by reason of passion or personal hostility. No such grave abuse of discretion exists in this case to warrant issuance of the extraordinary writ of certiorari. 2. Under Rule 46, Section 3, paragraph 3 of the 1997 Rules of Civil Procedure, petitions for certiorari must be verified and accompanied by a sworn certification of non-forum shopping. A pleading by an affidavit that the affiant has read the pleading and that the allegations therein are true and correct of his personal knowledge or based on authentic records. The party need not sign the verification. A party's

86. MEDISERV INC. V. CA Rule 7; Rule 45/46! FACTS: Petitioner Mediserv, Inc. executed a real estate mortgage in favor of China Banking Corporation as security for a loan. The mortgage was constituted on a 500-square meter lot with improvements located in Sampaloc, Manila and covered by Transfer Certificate of Title of the Registry of Deeds for the City of Manila. Mediserv defaulted on its obligation with Chinabank and the real estate mortgage was foreclosed. At the public auction sale, private respondent Landheights Development Corporation emerged as the highest bidder with a bid price of P17,617,960.00 for the subject property. Landheights filed with the RTC of Manila an "Application for Possession of Real Estate Property Purchased at an Auction Sale under Act No. 3135." The title of the property was consolidated in favor of Landheights and the Register of Deeds for the City of Manila issued a TCT in its favor. Landheights, seeking to recover possession of the subject property, filed a verified complaint for ejectment against Mediserv before the MeTC of Manila. The MeTC of Manila, rendered a decision in favor of Landheights. Mediserv appealed the decision to the RTC of Manila. The decision of the MeTC was reversed and the Complaint for Ejectment was dismissed. Landheights seasonably filed a motion for reconsideration and subsequently submitted a Secretarys Certificate by its Corporate Secretary, Polly S. Tiu, stating that the Board of Directors affirms the authority of Mr. Dickson Tan to file the Petition for Review.

representative, lawyer or any person who personally knows the truth of the facts alleged in the pleading may sign the verification. A certification of non-forum shopping is a certification under oath by the plaintiff or principal party in the complaint or other initiatory pleading asserting a claim for relief or in a sworn certification annexed thereto and simultaneously filed therewith, (a) that he has not commenced any action or filed any claim involving the same issues in any court, tribunal or quasi-judicial agency and, to the best of his knowledge, no such other action or claim is pending therein; (b) if there is such other pending action or claim, a complete statement of the present status thereof; and (c) if he should thereafter learn that the same or similar action or claim has been filed or is pending, he shall report that fact within five (5) days therefrom to the court wherein his aforesaid complaint or initiatory pleading has been filed. The Court has consistently held that the requirement regarding verification of a pleading is formal, not jurisdictional. Such requirement is simply a condition affecting the form of the pleading, non-compliance with which does not necessarily render the pleading fatally defective. Verification is intended to secure an assurance that the allegations in the pleading are true and correct and not the product of the imagination or a matter of speculation, and that the pleading is filed in good faith. The court may order the correction of the pleading if verification is lacking or act on the pleading although it is not verified, if the attending circumstances are such that strict compliance with the rules may be dispensed with to serve justice. On the other hand, the lack of certification against forum shopping is generally not curable by the submission after the filing of the petition. Section 5, Rule 45 of the 1997 Rules of Civil Procedure provides that the failure of the petitioner to submit the required documents, including the certification against forum shopping, shall be sufficient ground for dismissal. The same rule applies to certifications against forum shopping signed by a person on behalf of a corporation which are unaccompanied by proof that said person is authorized to file a petition on behalf of the corporation. In certain exceptional circumstances, however, the Court has allowed the belated filing of the certification. In the instant case, the merits of petitioner's case should be considered special circumstances or compelling reasons that justify tempering the requirement in regard to the certificate of non-forum shopping. A distinction must be made between non-compliance with the requirements for certificate of non-forum shopping and verification and substantial compliance with the requirements as provided in the Rules of Court. There is sufficient jurisprudential basis to hold that

Landheights has substantially complied with the verification and certification requirements. Jurisprudence shows that there is substantial compliance with the Rules of Court when there is a belated submission or filing of the secretary's certificate through a motion for reconsideration of the Court of Appeals' decision dismissing the petition for certiorari. In the present case, Landheights rectified its failure to submit proof of Mr. Dickson Tan's authority to sign the verification/certification on non-forum shopping on its behalf when the required document was subsequently submitted to the Court of Appeals. It is settled that liberal construction of the rules may be invoked in situations where there may be some excusable formal deficiency or error in a pleading, provided that the same does not subvert the essence of the proceeding and connotes at least a reasonable attempt at compliance with the rules. After all, rules of procedure are not to be applied in a very rigid, technical sense; they are used only to help secure substantial justice.

RULE 8 MANNER

OF

MAKING ALLEGATIONS

IN

PLEADINGS

RECALL: Addition Hills Mandaluyong Civic & Social Organization vs Megaworld Properties & Holdings Inc UNDER DOCTRINE OF EXHAUSTION OF ADMINISTRATIVE REMEDIES AND; DOCTRINE OF PRIMARY JURISDICTION; FOR: RULE 8

ANGELINA SORIENTE, ET. AL VS. THE ESTATE OF THE LATE ARSENIO E. CONCEPCION ET. AL., GR. NO. 160239

RECALL

UNDER: RULE 5;

FOR: RULE 8; AND FOR: RULES ON SUMMARY PROCEDURE


87. ASIAN CONSTRUCTION AND DEVELOPMENT CORP V. LOURDES MENDOZA Rule 8, Sec. 7 Facts: Lourdes K. Mendoza, owner ofHighett Steel Fabricators (Highett), filed Complaint for a sum of money, against petitioner Asian Construction and Development Corporation. Attaching a sales invoice to prove her claim. Mendoza alleged that Asian Construction purchased from Highett steel materials and supplies amounting to P1M+but Asian refused to pay. Asian Construction moved for a bill of particulars on the ground that no copies of the purchase orders and invoices were attached to the complaint to enable petitioner to prepare a responsive pleading to the complaint with the defense of lack of cause of action. Issue: W/N a charge or sales invoice is an actionable document? Held: The charge invoices are not actionable documents Section 7 of Rule 8 of the Rules of Court states: SEC. 7. Action or defense based on document. Whenever an action or defense is based upon a written instrument or document, thesubstance of such instrument or document shall be set forth in the pleading, and the original or a copy thereof shall be attached to the pleading as an exhibit, which shall be deemed to be a part of the pleading, or said copy may with like effect be set forth in the pleading. (Emphasis supplied.) A document is actionable when an action or defense is grounded upon such written instrument or document. In the instant case, the Charge Invoices are not actionable documents per se as these only provide details on the alleged transactions. These documents need not be attached to or stated in the complaint as these are evidentiary in nature. In fact, respondents cause of action is not based on these documents but on the contract of sale between the parties. BUT Delivery of the supplies and materials was duly proved. Although the Charge Invoices are not actionable documents, we find that these, along with the Purchase Orders, are sufficient to prove that petitioner indeed ordered supplies and

materials from Highett and that these were delivered to petitioner . 88. AQUILINO Q. PIMENTEL, Jr., et. al., vs. SENATE COMMITTEE OF THE WHOLE REPRESENTED BY SENATE PRESIDENT JUAN PONCE ENRILE RULE 8, SEC. 10; SUMMARY JUDGMENT! FACTS: This case is a petition for prohibitionwith prayer for issuance of a writ of preliminary injunction and/or temporary restraining order filed by Senators Pimentel, Villar, Arroyo, Pangilinan, PiaCayetano, and Alan PeterCayetano (petitioners). Petitioners seek to enjoin the Senate Committee of the Whole (respondent) from conducting further hearings on the complaint filed by Senator Madrigal against Senator Villar pursuant to Senate Resolution No. 706 (P.S. Resolution 706) on the alleged double insertion of P200 million for the C-5 Road Extension Project in the 2008 General Appropriations Act. Such issue began when Sen. Lacson mentioned in his privileged speech the alleged double insertion of P200M in the Gen. Appropriations Act and his investigations revealed the connection of Sen. Villar to such anomaly. Sen. Villar was the then Senate President when the issue was brought up. Then Sen. Madrigal introduced a resolution which she referred to the Ethics Committee regarding the anomaly. Sen. Enrile was then elected as the Senate President and the Ethics Committee was reorganized with the election of Sen. Lacson as its Chairperson. Sen.Lacson inquired whether the Minority was ready to name their representatives to the Ethics Committee. After consultation with the members of the Minority, Sen. Pimentel informed the body that there would be no member from the Minority in the Ethics Committee. Then Sen. Lacson reiterated his appeal to the Minority to nominate their representatives to the Ethics Committee. Sen. Pimentel stated that it is the stand of the Minority not to nominate any of their members to the Ethics Committee, but he promised to convene a caucus to determine if the Minoritys decision on the matter is final. Thereafter, the Senate adopted the Rules of the Senate Committee on Ethics and Privileges (Committee Rules) which was published in the Official Gazette on 23 March 2009. On 2009. Sen.Villar delivered a privilege speechwhere he stated that he would answer the accusations against him on the floor and not before the Ethics Committee. However, due to the accusation that the Ethics Committee could not act with fairness on Senator Villars case, Senator Lacson moved that the responsibility of the Ethics Committee be undertaken by the Senate, acting as a Committee of the Whole. The motion was

approved with ten members voting in favor, none against, and five abstentions. Senate Committee of the whole adopted several rules of the Ethics Committee and Sen. Pimentel raised an issue regarding the need to publish such rules but the Respondent Committee proceeded with the Preliminary inquiry on the resolution filed by Sen. Madrigal and alleged that publication is no longer necessary since the Rules on Ethics Committee was already published in the OG.

ISSUES: 1. Whether Sen. Madrigal, who filed the complaint against Senator Villar, is an indispensable party in this petition; 2. Whether the petition is premature for failure to observe the doctrine of primary jurisdiction or prior resort; 3. Whether publication of the Rules of the Senate Committee of the Whole is required for their effectivity. HELD: 1. NO, she is not an indispensable party. 2. NO, the case is within the jurisdiction of the court. 3. YES, publication is required as expressly stated in the internal rules formulated by the Senate. RATIO: 1. In this case, Senator Madrigal is not an indispensable party to the petition before the Court. While it may be true that she has an interest in the outcome of this case as the author of P.S. Resolution 706, the issues in this case are matters of jurisdiction and procedure on the part of the Senate Committee of the Whole which can be resolved without affecting Senator Madrigals interest. The nature of Senator Madrigals interest in this case is not of the nature that this case could not be resolved without her participation. 2. The doctrine of primary jurisdiction does not apply to this case. The Court has ruled: The issues presented here do not require the expertise, specialized skills and knowledge of respondent for their resolution. On the contrary, the issues here are purely legal questions which are within the competence and jurisdiction of the Court, and not an administrative agency or the Senate to resolve. Also, the Court is not precluded from resolving the legal issues raised by the mere invocation by respondent of the doctrine of separation of powers. On the contrary, the resolution of the legal issues falls within the exclusive jurisdiction of this Court. NOTE: No discussion regarding RULE 8, SEC. 10 Specific Denial or about Summary Judgment So I guess, a part of the case is a sample of Specific Denial. See next page. Copy paste from actual case.

Petitioners came to this Court for relief, raising the following grounds: 1. The transfer of the complaint against Senator Villar from the Ethics Committee to the Senate Committee of the Whole is violative of Senator Villars constitutional right to equal protection; 2. The Rules adopted by the Senate Committee of the Whole for the investigation of the complaint filed by Senator Madrigal against Senator Villar is violative of Senator Villars right to due process and of the majority quorum requirement under Art. VI, Sec. 16(2) of the Constitution; and 3. The Senate Committee of the Whole likewise violated the due process clause of the Constitution when it refused to publish the Rules of the Senate Committee of the Whole in spite of its own provision [which] require[s] its effectivity upon publication. In its Comment, respondent argues that: 1. The instant petition should be dismissed for failure to join or implead an indispensable party. In the alternative, the instant petition should be archived until such time that the said indispensable party has been joined or impleaded and afforded the opportunity to be heard; 2. There was no grave abuse of discretion on the part of respondent Committee; 3. Petitioners are not entitled to a writ of prohibition for failure to prove grave abuse of discretion on the part of respondent Committee of the Whole; 4. The principle of separation of powers must be upheld; 5. The instant petition must be dismissed for being premature. Petitioners failed to observe the doctrine or primary jurisdiction or prior resort; 6. It is within the power of Congress to discipline its members for disorderly behavior; 7. The determination of what constitutes disorderly behavior is a political question which exclusively pertains to Congress; 8. The Internal Rules of the Senate are not subject to judicial review in the absence of grave abuse of discretion; [and] 9. The Rules of the Ethics Committee, which have been duly published and adopted[,] allow the adoption of supplementary rules to govern adjudicatory hearings.

89. PBCOM V. SPOUSES GO GR No. 175514 February 14, 2011

Rule 8, Section 10; Summary Judgment! FACTS Go obtained 2 loans from PBCom, evidenced by 2 PNs for 17.8M and 80M payable within a 10-year period. To secure the loans, Go pledged shares of stock in Ever Gotesco Resources and Holdings, Inc. 2 years later, however, the market value of the said shares of stock plunged, thus, PBCom, as pledge, notified Go that it was renouncing the pledge agreements. Later, PBCom filed before the RTC a complaint for sum of money with preliminary attachment against the Spouses Go, alleging that the respondents defaulted in their obligation, having paid only 3 installments on interest payments, thus making the whole amount due and demandable. Spouses Go counterclaimed that they were not aware of a demand from the petitioner, that substantial payments were made, and according to the PNs, payment was due only after 10 years. PBCom filed a verified motion for summary judgment on the grounds that (1) the material averments of the complaint admitted by the spouses in their Answer obviate the necessity of trial; (2) no real defenses and no genuine issues as to any material fact were tendered in the Answer; (3) plaintiffs causes of actions are supported by voluntary admissions and authentic documents which may not be contradicted. PBCom granted the motion. CA reversed the decision of the RTC and remanded the records to the court for trial on the merits. The CA believed that the spouses Go raised genuine issues on the material facts in the action. ISSUE Whether summary judgment was proper HELD NO RATIO The CA correctly ruled that there exist genuine issues as to 3 material facts, which have to be addressed during trial: (1) the fact of default; (2) the amount of the outstanding obligation; and (3) the existence of prior demand. Under Rule 35 of the 1997 Rules of Procedure, except as to the amount of damages, there is no genuine issue as to any material fact, and the moving party is entitled to a judgment as a matter of law, summary judgment may be rendered. Summary or accelerated judgment is a procedural technique aimed at weeding out sham claims or defenses at an early stage of litigation thereby avoiding the expense and loss of time involved in a trial. Even if on their face the pleadings appear to raise issues, when the affidavits, depositions and admissions show that such

issues are not genuine, then summary judgment must ensue as a matter of law. A genuine issue is an issue of fact which requires the presentation of evidence as distinguished from a fictitious or false claim. When the facts as pleaded appear uncontested or undisputed, then there is no real or genuine issue or questions as to the facts, and summary judgment is called for. The party who moves for judgment has the burden of demonstrating clearly the absence of any genuine issue of fact. In the case at bar, the Answer discloses that the material facts are not undisputed so as to call for the rendition of summary judgment. The pleadings show that they did in fact raise material issues that have to be addressed and threshed out in a full-blown trial. Under the Rules, every pleading must contain, in a methodical and logical form, a plain, concise and direct statement of the ultimate facts on which the party pleading relies for his claim or defense, as the case may be, omitting the statement of mere evidentiary facts. Rule 8, Section 10 of the Rules contemplates 3 modes of specific denial, namely: (1) by specifying each material allegation of the fact in the complaint, the truth of which the defendant does not admit, and whenever practicable, setting forth the substance of the matters which he will rely upon to support his denial; (2) by specifying so much of an averment in the complaint as is true and material and denying only the remainder; (3) by stating that the defendant is without knowledge or information sufficient to form a belief as to the truth of a material averment in the complaint, which has the effect of a denial. The purpose of requiring the defendant to make a specific denial is to make him disclose the matters alleged in the complaint which he succinctly intends to disprove at the trial, together with the matter which he is relied upon to support the denial. The parties are compelled to lay their cards on the table. Spouses Go are not disclaiming knowledge of the transaction or the execution of the PNs or the pledge agreements sued upon. The matters in contention are, as the CA stated, whether or not respondents were in default, whether there was prior demand, and the amount of the outstanding loan. These are matters that the parties disagree on and by which reason they set forth vastly different allegations in their pleadings which each will have to prove by presenting relevant and admissible evidence during trial. PBCom cited 4 cases to bolster their claim, but the Court found that in stark contrast to such cases, where one of the parties disclaimed knowledge of something so patently within his knowledge, in the instant case, respondents categorically stated in the Answer that there was no prior demand, that they were not in default, and that the amount of the outstanding loan would have to be ascertained based on official records. PETITION DENIED.

90. GSIS AND WINSTON GARCIA V. VILLAVIZA FACTS: -Winston was President and General Manager (PGM) of GSIS filed several suits against certain public -officials for grave misconduct -That on 2005, said employees marched to the office of GSIS in a mass demonstration rally which was prohibited by CSC Resolution 21316 -Defendants submitted that it was NOT PLANNED (Planned kasi yung bawal) however, The respondents never filed their answers to the formal charges. They only submitted letters denying their planned march -CSC lowered the penalty -This was being questioned by GSIS ISSUE: AN ADMINISTRATIVE TRIBUNAL MAY APPLY SUPPLETORILY THE PROVISIONS OF THE RULES OF COURT ON THE EFFECT OF FAILURE TO DENY THE ALLEGATIONS IN THE COMPLAINT AND FAILURE TO FILE ANSWER, WHERE THE RESPONDENTS IN THE ADMINISTRATIVE PROCEEDINGS DID NOT FILE ANY RESPONSIVE PLEADING TO THE FORMAL CHARGES AGAINST THEM. HELD: Yes RATIO: Must remember that RULES apply only suppletorily, in this case the Rules of GSIS will govern. The failure of a respondent to file an answer merely translates to a waiver of his right to file an answer. There is no such deficiency as the rules of the GSIS are explicit in case of failure to file the required answer. Even granting that Rule 8, Section 11 of the Rules of Court finds application in the case, petitioners must remember that there remain averments that are not deemed admitted by the failure to deny the same. Among them are immaterial allegations and incorrect conclusions drawn from facts set out in the complaint, thus even if respondents failed to file their answer, it does not mean that all averments found in the complaint will be considered as true and correct in their entirety, and that the forthcoming decision will be rendered in favor of the petitioners. We must not forget that even in administrative proceedings, it is still the complainant, or in this case the petitioners, who have the burden of proving, with substantial evidence, the allegations in the complaint or in the formal charges RULE 9 E FFECT
OF

Rule 9 Sec 1 Facts: The Coronels executed a document entitled receipt of down payment, stating that they received from Ramona Alcaraz, through Ramonas mother, Concepcion, the sum of P50,000 as downpayment on the total purchase price of P1,240,000.00 for their inherited house and lot The Coronels then sold the property to petitioner Catalina BalaisMabanag for a higher price (P1.5M). For this reason, the Coronels rescinded their contract with Ramona by depositing her downpayment. Ramonas mother, Conception, later filed a complaint for specific performance in her own name in the RTC. Concepcion sought leave of court to amend the complaint for the purpose of impleading Ramona as a co-plaintiff However, petitioner Catalina had a notice of adverse claim annotated in the title and later on a TCT was issued in her name. RTC ruled in favor of Ramona and Conception, so the Coronels and petitioner Catalina interposed an appeal to the CA, however the CA only affirmed RTCs decision. Thereafter, the decision of the RTC became final and executory. CA affirmed RTCs decision and sustained the registration by the Registrar of Deeds of the deed of absolute sale despite the lack of indication of the citizenship of the buyer (Conception) of the subject property. It also sustained the order of the RTC directing the Branch Clerk of Court to execute the deed of absolute sale without first requiring the defendants Conception and Ramona to execute the deed of absolute sale as required by the decision. Issue: W/N CA erred in sustaining the registration by the Registrar of Deeds of the deed of absolute sale despite the lack of indication of the citizenship of the buyer of the subject property; and in sustaining the order of the RTC directing the Branch Clerk of Court to execute the deed of absolute sale without first requiring the defendants to execute the deed of absolute sale as required by the decision. Held: NO. Ratio: In her complaint filed in 1985, Concepcion, as plaintiff, categorically averred that she was a Filipino citizen.The petitioner did not also advert to the issue of citizenship after the complaint was amended in order to implead Ramona as a co-plaintiff, despite the petitioners opportunity to do so. Yet, now, when the final decision of the RTC is already being implemented, the petitioner would thwart the execution byassailing the directive of

FAILURE

TO

PLEAD

91. CATALINA BALAIS-MABANAG V. RD OF QC

the RTCon the ground that Ramona was disqualified from owning land in the Philippines. This move is unwarranted. Petitioner Catalina did not raise any issue against Ramonas qualifications to own land in the Philippines during the trial or, at the latest, before the finality of the RTC judgment. The petitioner was thereby deemed to have waived the objection, pursuant to Section 1, Rule 9 of the Rules of Court. In every action, indeed, the parties and their counsel are enjoined to present all available defenses and objections in order that the matter in issue can finally be laid to rest in an appropriate contest before the court. The rule is a wise and tested one, borne by necessity. Without the rule, there will be no end to a litigation, because the dissatisfied litigant may simply raise new or additional issues in order to prevent, defeat, or delay the implementation of an already final and executory judgment.

Dismiss or any responsive pleading for that matter within the reglementary period, which is 15 days from receipt of Summons or on October 2, 2000 but no pleading was filed on the aforesaid date, not even a Motion for Extension of Time. Unsatisfied with the RTC orders, the petitioners filed with the CA a petition for certiorari. CA affirmed the RTC orders.

ISSUE: Whether or not the order of default is proper? HELD: Yes. RULING: The Order of the trial court declaring the defendants-appellants in default for their failure to file their responsive pleading to the complaint within the period prescribed under Section 3 of Rule 9 of the Revised Rules of Court had been declared final and beyond review already by the Supreme Court through its Resolution dated March 5, 2008 and June 25, 2008. Judicial decisions of the Supreme Court, as the final arbiter of any justiciable controversy, assume the same authority as the law itself. Thus, the issue raised by the defendants-appellants questioning the wisdom of the trial courts decision in declaring them in default is now rendered moot and academic by the aforecited Supreme Court resolutions. The petitioners default by their failure to file their answer led to certain consequences. Where defendants before a trial court are declared in default, they thereby lose their right to object to the reception of the plaintiffs evidence establishing his cause of action. This is akin to a failure to, despite due notice, attend in court hearings for the presentation of the complainants evidence, which absence would amount to the waiver of such defendants right to object to the evidence presented during such hearing, and to cross-examine the witnesses presented therein. Taking into consideration the banks allegations in its complaint and the totality of the evidence presented in support thereof, coupled with the said circumstance that the petitioners, by their own inaction, failed to make their timely objection or opposition to the evidence, both documentary and testimonial, presented by TMBC to support its case, there is no cogent reason to reverse the trial and appellate courts findings. In civil cases, the party having the burden of proof must establish his case only by a preponderance of evidence. Preponderance of evidence is a phrase which means probability to truth. It is evidence which is more convincing to the court as worthier of belief than that which is offered in opposition thereto.

92. MAGDIWANG REALTY CORPORATION v. MANILA BANKING CORPORATION E Rule 9 Sec 3 Default FACTS: The Manila Banking Corporation (TMBC) filed on April 18, 2000 a complaint for sum of money before the RTC, Makati City against Magdiwang Realty Corporation, Renato Dragon and Esperanza Tolentino, after said petitioners allegedly defaulted in the payment of their debts under the 5 promissory notes they executed in favor of TMBC, which amounted to P2.5M. Instead of filing a responsive pleading with the trial court, the petitioners filed on October 12, 2000, which was notably beyond the 15-day period allowed for the filing of a responsive pleading, a Motion for Leave to Admit Attached Motion to Dismiss and a Motion to Dismiss, raising therein the issues of novation, lack of cause of action against individuals Dragon and Tolentino, and the impossibility of the novated contract due to a subsequent act of the Congress. The motions were opposed by the respondent TMBC, which likewise asked that the petitioners be declared in default for their failure to file their responsive pleading within the period allowed under the law. RTC issued an Order declaring the petitioners in default. The record shows that summons were served on 19 September 2000 by way of substituted service. Summons were received by a certain LINDA G. MANLIMOS, a person of sufficient age and discretion then working/residing at the address indicated in the Complaint. Consequently, defendants should have filed an Answer or Motion to

93. ROBERTO OTERO v. ROGER TAN (Rule 9 Sec 3) FACTS: Roger Tan filed for a collection of sum of money and damages w/ the MTCC against Roberto Otero. Otero purchased several petroleum products from Tans Petron outlet amounting to P270,818.01 but he failed to pay. Despite the complaint against Otero, he failed to file an answer so Tan filed a motion to declare Otero in default. Otero alleged that he did not receive a copy of summons and the copy of Tans complaint. A rehearing was set but Otero failed to appear, thus MTCC declared Otero in default. Tan was then allowed to present his evidence ex-parte w/c includes testimonies from his 2 employees and various statements of accounts. These statements of accounts were prepared by one Betache but he was not presented as a witness. Tan wonn Otero appealed w/ RTC claiming he was deprived of due process, but the RTC affirmed the decision of the MTCC. RTC pointed out that Oteros wife actually received the summons and the complaint. Otero was further furnished a copy of the motion to declare him in default w/c Otero only received but didnt file an answer to the Motion or any pleading against him he only commented to the Motion to declare him in Default. Otero then filed a petition for review w/ the CA claiming that Tan failed to prove the material allegations of his complaint since the statements of account which he presented are inadmissible in evidence because they are private documents w/c are not authenticated. The CA affirmed both of the decision of RTC and the MTCC ISSUE: W/N Otero waived whatever defenses he has against Tans claim by failing to file an answer HELD: A defendant who was declared in default may nevertheless appeal from the judgment by default, albeit on limited grounds. RATIO: Rule 9 of the ROC: Sec. 3. Default; declaration of. If the defending party fails to answer within the time allowed therefor, the court shall, upon motion of the claiming party with notice to the defending party, and proof of such failure, declare the defending party in default. xxx Loss of standing in court, the forfeiture of ones right as a party litigant, contestant or legal adversary, is the consequence of an order of default. Nonetheless, a defendant declared in default still has remaining remedies: 1. The defendant in default may, at any time after discovery thereof and before judgment, file a motion, under oath, to set aside the

order of default on the ground that his failure to answer was due to fraud, accident, mistake or excusable neglect, and that he has meritorious defenses; (Sec 3, Rule 18) 2. If the judgment has already been rendered when the defendant discovered the default, but before the same has become final and executory, he may file a motion for new trial under Section 1(a) of Rule 37; 3. If the defendant discovered the default after the judgment has become final and executory, he may file a petition for relief under Section 2 of Rule 38 4. He may also appeal from the judgment rendered against him as contrary to the evidence or to the law, even if no petition to set aside the order of default has been presented by him. (Sec. 2, Rule 41) A defending party declared in default retains the right to appeal from the judgment by default. However, the grounds that may be raised in such an appeal are restricted to any of the following: first, the failure of the plaintiff to prove the material allegations of the complaint; second, the decision is contrary to law; and third, the amount of judgment is excessive or different in kind from that prayed for. However, he is proscribed in the appellate tribunal from adducing any evidence to bolster his defense against the plaintiffs claim. Although the defendant would not be in a position to object, elementary justice requires that only legal evidence should be considered against him. (For recit) The SC opines that the statements of account presented by Tan were merely hearsay. A private document is any other writing, deed, or instrument executed by a private person without the intervention of a notary or other person legally authorized by which some disposition or agreement is proved or set forth. A private document requires authentication in the manner allowed by law or the ROC before its acceptance as evidence in court. The requirement of authentication of a private document is excused only in 4 instances, specifically: (a) when the document is an ancient one within the context of Section 21, Rule 132 of the ROC; (b) when the genuineness and authenticity of an actionable document have not been specifically denied under oath by the adverse party; (c) when the genuineness and authenticity of the document have been admitted; or (d) when the document is not being offered as genuine. Here, Tan did not present anyone who testified that the said statements of account were genuine and duly executed. Notwithstanding the inadmissibility of the said statements of account, the SC finds that Tan was still able to prove by a preponderance of evidence the material allegations of his complaint 94. RAPID CITY REALTY AND DEVELOPMENT CORPORATION V. ORLANDO VILLA AND LOURDES PAEZ-VILLA

Rule 9; Rule 14! FACTS: Rapid City Realty and Development Corporation (petitioner) filed a complaint for declaration of nullity of subdivision plans . . . mandamus and damages against several defendants including Spouses Orlando and Lourdes Villa (respondents). After one failed attempt at personal service of summons, Zapanta, court process server, resorted to substituted service by serving summons upon respondents househelp who did not acknowledge receipt thereof and refused to divulge their names. Zapanta stated this in the Return of Summons. Despite substituted service, respondents failed to file their Answer, prompting petitioner to file a Motion to Declare Defendants[ -herein respondents] in Default which the trial court granted by Order. 8 months thereafter, respondents filed a Motion to Lift Order of Default, claiming that on January 27, 2006 they officially received all pertinent papers such as Complaint and Annexes. And Motion to Dismiss of the Solicitor General and the ORDER granting the Motion to Declare [them] in Default. Respondents denied the existence of two women helpers who allegedly refused to sign and acknowledge receipt of the summons. In any event, they contended that assuming that the allegation were true, the helpers had no authority to receive the documents. By Order of July 2006, the trial court set aside the Order of Default and gave herein respondents five days to file their Answer. Respondents just the same did not file an Answer, drawing petitioner to again file a Motion to declare them in default, which the trial court again granted by Order of February 2007. On April 2007, respondents filed an Omnibus Motion for reconsideration of the second order declaring them in default and to vacate proceedings, this time claiming that the trial court did not acquire jurisdiction over their persons due to invalid service of summons. The trial court denied respondents Omnibus Motion by Order of May 2007 and proceeded to receive ex-parte evidence for petitioner. Respondents, via certiorari, challenged the trial courts February 2007 and April 2007 Orders before the Court of Appeals. In the meantime, the trial court, by Decision of September 2007, rendered judgment in favor of petitioner ISSUE: Did the trial court acquire jurisdiction? HELD: YES

RATIO: It is settled that if there is no valid service of summons, the court can still acquire jurisdiction over the person of the defendant by virtue of the latters voluntary appearance. Thus Section 20 of Rule 14 of the Rules of Court provides: Sec. 20. Voluntary appearance. The defendants voluntary appearance in the action shall be equivalent to service of summons. The inclusion in a motion to dismiss of other grounds aside from lack of jurisdiction over the person shall not be deemed a voluntary appearance. And Philippine Commercial International Bank v. Spouses Wilson Dy Hong Pi and Lolita Dy, et al. enlightens: Preliminarily, jurisdiction over the defendant in a civil case is acquired either by the coercive power of legal processes exerted over his person, or his voluntary appearance in court. As a general proposition, one who seeks an affirmative relief is deemed to have submitted to the jurisdiction of the court. It is by reason of this rule that we have had occasion to declare that the filing of motions to admit answer, for additional time to file answer, for reconsideration of a default judgment, and to lift order of default with motion for reconsideration, is considered voluntary submission to the courts jurisdiction. This, however, is tempered by the concept of conditional appearance, such that a party who makes a special appearance to challenge, among others, the courts jurisdiction over his person cannot be considered to have submitted to its authority. Prescinding from the foregoing, it is thus clear that: (1) Special appearance operates as an exception general rule on voluntary appearance; to the

(2) Accordingly, objections to the jurisdiction of the court over the person of the defendant must be explicitly made, i.e., set forth in an unequivocal manner; and (3) Failure to do so constitutes voluntary submission to the jurisdiction of the court, especially in instances where a pleading or motion seeking affirmative relief is filed and submitted to the court for resolution. In their first Motion to Lift the Order of Default dated January 2006, Respondents did not allege that their filing thereof was a special appearance for the purpose only to question the jurisdiction over their persons.

Clearly, they had acquiesced to the jurisdiction of the court.

ISSUE: WHETHER CA erred in admitting the Kasunduan and the TCT of Concepcion dela Paz-Lesaca. NO RATIO: CA justified its admitting of the evidence using RULE 10, Sec. 5, that issues not raised in the pleadings may be tried by express or implied consent of the parties, as if they had been raised in the pleadings and the court can validly resolve them. The GENERAL RULE is that a judgment must conform to the pleading and the theory of the action which the case is tried. BUTthe court may also rule based on the evidence before it even though the relevant pleadings has not been amended. As long as no prejudice to the adverse party is thereby caused and there is express/implied consent to the presentation of evidence. In this case, since there was no objection by the Agrifina to the presentation of evidence, the same should be admitted by the court a quo in accordance with RULE 10, Sec 5 and Liberal Construction of the Rules of Court. SC have ruled that amendment need not be applied rigidly so long as it does not cause surprise or prejudice to the other party. 96. SPOUSES VILLUGA vs. CONSTRUCTION SUPPLY INC. Rule 10; Rule 26; Rule 35! FACTS: The spouses Villuga made purchases of various construction materials from Kelly Hardware in the sum of P259,809.50, which has not been paid, both principal and stipulated interests. Kelly Hardware made several demands for the spouses to pay all their obligations, but the latter failed and refused to comply. Eventually, Kelly Hardware filed with the RTC of Bacoor, Cavite a Complaint for a Sum of Money and Damages. Besides the principal obligation, the hardware store also sought the spouses to pay for the suit. In their Answer to Complaint, the spouses admitted having made purchases from respondent. Petitioners claimed that they have made payments to the respondent in the amounts of P110,301.80 and P20,000.00, respectively, and they are willing to pay the balance of their indebtedness after deducting the payments made and after verification of their account. Respondent filed an Amended Complaint adding that only P20,000.00 had been paid from the principal obligation leaving a balance of P259,809.50. Respondent then filed a Request for Admission asking petitioners to admit that the latter's principal obligation is P279,809.50 and that only P20,000.00 was paid. KELLY HARDWARE AND

RULE 10 AMENDED

AND

S UPPLEMENTAL P LEADINGS

95. AGRIFINA PANGANIBAN V. ROLDAN DOCTRINE: Where there is a variance in defendants pleadings and the evidence adduced at the trial, the COURT may treat the pleading as if it had been amended to conform to the evidence FACTS: Panganiban filed a complaint against Roldan for the recovery of possession and damages in the MTC SUBIC, Zambales because allegedly, respondents entered the land and built a small hut on a portion thereof without Panganibans consent at first and then subsequently she gave them permission with the condition that they would vacate upon demand. According to Agrifina, upon demand sometime on 1997, repondents refused to vacate thus Agrifina required them to pay rental for P500/month from December that year. Roldandenied that allegation saying that they were occupying the land as caretakers of the heirs of Concepcion dela Paz-Lesaca isnce 1973, as evidenced by a Kasunduan and that the said lot was part of the TCT of Concepcion dela Paz-Lesaca. MTC did not admit Roldans evidence presented during trial consisting of (1) TCT of Concepcion dela Paz-Lesaca; and (2) the Kasunduan BECAUSE these matters were not raised in their ANSWER or in their PRE-TRIAL Brief. MTCsaid that there was a variance of the allegation and proof, thus it considered the evidence as no proof at all and said that the proper remedy was to AMEND the ANSWER to conform to the evidence which the Roldans failed to do. CA reversed and ADMITTED as evidence the Kasunduanand the TCT of Concepcion dela Paz-Lesaca

The spouses failed to reply. After some time, the respondent again filed a Second Amended Complaint that confirmed petitioners' partial payment in the sum of P110,301.80, but alleged that this payment was applied to other obligations which petitioners owe respondent. Respondent reiterated its allegation that, despite petitioners' partial payment, the principal amount which petitioners owe remains P259,809.50. Upon the failure to reply to the Request for Admission by the spouses, the respondent filed a Motion to Expunge with Motion for Summary Judgment claiming that petitioners were deemed to impliedly admit the matters subject of the said request. Respondent also contended that it is already entitled to the issuance of a summary judgment in its favor as petitioners not only failed to tender a genuine issue as to any material fact but also did not raise any special defenses, which could possibly relate to any factual issue. RTC granted the respondents Motion. The CA affirmed.

On Rule 35 (Sections 1 and 3):The Court finds that the CA was correct in sustaining the summary judgment rendered by the RTC in accordance with Sections 1 and 3, Rule 35 of the Rules of Court. When respondent subsequently filed its Second Amended Complaint, it admitted therein that petitioners made partial payments of P110,301.80 and P20,000.00. Nonetheless, respondent accounted for such payments by alleging that these were applied to petitioners' obligations which are separate and distinct from the sum of P259,809.50 being sought in the complaint. This allegation was not refuted by petitioners in their Answer to Second Amended Complaint. This lead the court to no other conclusion than that payments made by the spouses were indeed applied to their other debts to respondent leaving an outstanding obligation of P259,809.50.Hence, the summary judgment of the RTC in favor of respondent is proper.

ISSUE: Whether there should not have been a summary judgment against the spouses Villuga HELD: No.The instant petition is DENIED. The Decision and Resolution of the CA are AFFIRMED. RATIO: On Rule 10 (Section 8): The Court agrees with the CA in holding that respondent's Second Amended Complaint supersedes only its Amended Complaint and nothing more.From the foregoing, it is clear that respondent's Request for Admission is not deemed abandoned or withdrawn by the filing of the Second Amended Complaint. The Court holds that by reason of the belated filing of petitioners' Comments on the Request for Admission, they are deemed to have impliedly admitted that they are indebted to respondent in the amount of P259,809.50. On Rule 26 (Section 2):This Court has ruled that if the factual allegations in the complaint are the very same allegations set forth in the request for admission and have already been specifically denied, the required party cannot be compelled to deny them anew. A request for admission that merely reiterates the allegations in an earlier pleading is inappropriate under Rule 26 of the Rules of Court. The party being requested should file an objection to the effect that the request for admission is improper and that there is no longer any need to deny anew the allegations contained therein considering that these matters have already been previously denied.

LILIA B. ADA, LUZ B. ADANZA, FLORA C. BAYLON, REMO BAYLON, JOSE BAYLON, ERIC BAYLON, FLORENTINO BAYLON, and MA. RUBY BAYLONvs.FLORANTE BAYLON SEC 6 MISJOINDER OF CAUSE OF ACTIONS; RULE 10!

RULE 11 WHEN TO FILE R ESPONSIVE PLEADINGS


97. JAIME GOSIACO VS. LETICIA CHING AND EDWIN CASTA GR No. 173807 April 16, 2009 Facts Petitioner Gosiaco loaned money to ASB Holdings, Inc. in the amount of P8M for a period of 48 days with interest at 10.5%. In exchange, ASB, through its Business Development Operation Group Manager, Respondent Leticia Ching, signed and issued 2 checks in favor of Gosiaco. Upon maturity of the ASB checks, petitioner went to deposit the two checks. However, upon presentment, the checks were dishonored and payments were refused because of a stop payment order and for insufficiency of funds. Petitioner informed respondents about the dishonor of the checks and demanded replacement checks or the return of the money placement but to no avail. Petitioner then filed a criminal complaint for violation of BP22 before the MTC of San Juan against the private respondents. Ching denies liability and claims that she was a mere employee of ASB. She asserted that she did not have knowledge as to how much money ASB had in the banks.

Citing Sec. 1(b), Rule 111 Gosiaco then moved that ASB and its President, Roxas, be impleaded as party defendants but it was denied by the MTC. MTC acquitted Ching of criminal liability but it did not absolve her from civil liability. The MTC ruled that Ching, as a corporate officer of ASB, was civilly liable since she was a signatory to the checks. RTC exonerated Ching from civil liability and ruled that the subject obligation fell squarely on ASB. The RTC likewise affirmed the MTCs ruling on denying the motion to implead ASB and Roxas. CA affirmed the decision of the RTC and stated that the amount Gosiaco sought to recover was a loan made to ASB and not to Ching. It further ruled that ASB cannot be impleaded in a BP22 case since it is not a natural person and in the case of Roxas, he was not the subject of a preliminary investigation. CA also held that there was no need to pierce the corporate veil of ASB. Issues 1. Whether a corporate officer who signed a bouncing check is civilly liable under BP22. 2. Whether a corporation can be impleaded in a BP22 case. 3. Whether there is basis to pierce the corp veil Held 1. Yes, a corporate officer who signed a bouncing check may be civilly liable under BP22. 2. No, a corporation cannot be impleaded in a BP22 case. 3. No, there is no basis to pierce. Ratio 1. Section 1 of B.P. Blg. 22 or the Bouncing Checks Law provides for checks drawn by a corporation, company or entity, the person or persons, who actually signed the check in behalf of such drawer shall be liable. The statute imposes criminal penalties on anyone who with intent to defraud another of money or property, draws or issues a check on any bank with knowledge that he has no sufficient funds in such bank to meet the check on presentment. Moreover, the personal liability of the corporate officer is predicated on the principle that he cannot shield himself from liability from his acts on the ground that it was a corporate act. 2. SC ruled that ASB, or any other corporation, cannot be impleaded in the BP 22 case even if Sec. 1(b), Rule 11 require the joint trial of both the criminal and civil liability. Nowhere in BP 22 is it provided that a corporation may be impleaded as an accused or defendant in the prosecution for violations of that law, even in the litigation of the civil aspect thereof.

However, this does not mean that an aggrieved party has no recourse against the corporation. The substantive right of a creditor to recover due and demandable obligations against a debtor cannot be denied or diminished by a rule of procedure. Nothing in Sec. 1(b), Rule 11 prohibits the filing of a separate civil action against the corporation on whose behalf the check was issued. What the rules prohibit is the reservation of a separate civil action against the natural person charged with violating BP 22, including such corporate officer who had signed the bounced check. The implied institution of the civil case into the criminal case for BP 22 should not affect the civil liability of the corporation for the same check, since such implied institution concerns the civil liability of the signatory, and not of the corporation. The civil liability attaching to the signatory arises from the wrongful act of signing the check despite the insufficiency of funds in the account, while the civil liability attaching to the corporation is itself the very obligation covered. There is, however, the possibility that the aggrieved party might be able to collect twice on the basis of the same checks if he wins in both case. This should not be permitted as it would amount to unjust enrichment. Consequently, the matter was already referred to the Committee on Rules for the formulation of proper guidelines to prevent that possibility. 3. BP22 criminal liability of the person who issued the bouncing check in behalf of a corporation stands independent of the civil liability of the corporation itself The actions involving these liabilities should be adjudged according to their respective standards and merits. In the BP22 case, what the trial court should determine whether or not the signatory had signed the check with knowledge of the insufficiency of funds, while in the civil case the trial court should ascertain whether or not the obligation itself is valid and demandable. The litigation of both questions could, in theory, proceed independently and simultaneously without being ultimately conclusive on one or the other. RULE 12 BILL NO CASE RULE 13 F ILING
AND OF

PARTICULARS

SERVICE

OF

P LEADINGS , J UDGMENTS PAPERS

AND

OTHER

98. MINDANAO TERMINAL AND BROKERAGE SERVICE, INC., V. COURT OF APPEALS

Doctrine: Service by registered mail is complete upon actual receipt by the addressee, or after five (5) days from the date he received the first notice of the postmaster, whichever date is earlier. Facts: RTC Manila rendered a decision in Philippine Ports Authority v. Mindanao Terminal and Brokerage Service, Inc., ordering MINTERBRO to pay PPA the sum of P36,585,901.18, as governments ten percent (10%) share in MINTERBROs gross income from its port-related services. MINTERBRO appealed to the CA. CA in a decision dated 21 November 2002, affirmed the decision of the RTC. On same date, copies of the said Decision were sent via registered mail to the parties respective counsels along with the Notice of the Decision. PPA filed Compliance manifesting its receipt of the decision. MINTERBRO failed to do the same. CA inquired with the post master on the status of the sent mail containing its decision. The Postmaster of Pasig City - Central Post Office advised the Court of Appeals that registered letter No. 6270-B was received by Virgie Cabrera (Cabrera) at the stated address on 4 December 2002. Counted from that date, 4 December 2002, the Court of Appeals Decision became final and executory on 20 December 2002 or 15 days after Cabreras receipt of the decision. The decision was, thus, recorded in the Book of Entries of Judgments. Copies of the Entry of Judgment were sent on 1 August 2003 to the parties counsels, with MINTERBROs copy having been addressed to Atty. Rafael Dizon (Atty. Dizon), 6/F Padilla Building, Emerald Avenue, Ortigas Commercial Center, Pasig City.Atty. Dizon argued that he did not receive the 21 November 2002 Court of Appeals Decision, and, hence, the Decision rendered by CA has not been served on MINTERBRO, it is without doubt that the reglementary period to appeal has not commenced and therefore, the aforesaid decision has not become final and since the Court of Appeals decision has not yet become final, the issuance by the Division Clerk of Court of the Entry of Judgment was premature. Meanwhile, the PPA, by virtue of the Entry of Judgment, filed a Motion for the Issuance of a Writ of Execution. The RTC later held in abeyance the execution of judgment because the case is still pending in the CA. Issue: Whether the Court of Appeals Decision dated 21 November 2002 had become final and executory; Whether the decision was properly served on MINTERBROs counsel. Whether RTC committed grave abuse of discretion when it held in abeyance the execution of the CA decision

Held: The service of judgment serves as the reckoning point to determine whether a decision had been appealed within the reglementary period or has already become final. Atty. Dizon contends that the receptionist is not connected with her office therefore he was not properly served with the decision. Service by registered mail is complete upon actual receipt by the addressee, or after five (5) days from the date he received the first notice of the postmaster, whichever date is earlier. Wherefore, the CA decision is final upon the lapse of the period to appeal. Once a judgment becomes final, the prevailing party is entitled as a matter of right to a writ of execution. RTC abused its discretion when it held in abeyance the issuance of the writ of execution of the judgment notwithstanding the fact that the same had already become final and executory this notwithstanding that MINTERBRO filed before this Court a petition for certiorari under Rule 65 of the Rules of Court .It is a basic rule that a petition for certiorari under Rule 65 does not by itself interrupt the course of the proceedings. It is necessary to avail of either a temporary restraining order or a writ of preliminary injunction to be issued by a higher court against a public respondent so that it may, during the pendency of the petition, refrain from further proceedings. 99. MARCELINO DOMINGO v. CA, ET AL. FACTS This is a petition for certiorari under Rule 65 of the Rules of Court. The petition challenges the 5 April and 10 June 2005 Resolutions of the Court of Appeals in CA-G.R. SP No. 89023. The Court of Appeals dismissed the petition for certiorari, with prayer for issuance of a temporary restraining order, filed by Marcelino Domingo (Marcelino) for failure to serve the pleadings personally and for failure to provide a written explanation why the service was not done personally. Before he died, Julio Domingo (Julio) allegedly executed a Deed of Absolute Sale over a 4.1358-hectare parcel of land in favor of Marcelinos wife, Carmelita Mananghaya (Mananghaya); the lot was located in Nueva Ecija. Agapita and Ana before the RTC Mananghaya for Domingos alleged Domingo, and the other heirs of the Domingos filed of Nueva Ecija a complaint against Marcelino and the annulment of the Deed of Absolute Sale. The that Julios signature in the deed was forged.

In its Decision, the RTC held that Julios signature in the Deed of Absolute Sale was forged; thus, the deed was void. The RTC ordered

Marcelino and Mananghaya to deliver possession of the property to the Domingos. Marcelino reentered and retook possession of the property. The Domingos filed before the RTC a motion to cite Marcelino in contempt. In its Order, the RTC found Marcelino in contempt, fined him P25,000 and ordered his arrest and imprisonment. Marcelino filed an MR. MR denied. Marcelino filed before the Court of Appeals a petition for certiorari under Rule 65 of the Rules of Court, with prayer for the issuance of a temporary restraining order. Marcelino alleged that the RTC had no jurisdiction to order him to deliver possession of the property to the Domingos and that the RTC gravely abused its discretion in finding him in contempt. CA dismissed petition. Marcelino filed an MR. The Court of Appeals denied the motion. The Court of Appeals held that Marcelinos failure to file a written explanation to justify service by mail in lieu of the preferred mode of personal service is an absolutely insurmountable obstacle to the success of this motion for reconsideration. Marcelino filed another motion for reconsideration. The Court of Appeals noted the motion. The Court of Appeals held that, We cannot accept the motion for reconsideration for the reason that a second motion for reconsideration of a final order is a prohibited pleading. ISSUE W/N the CA committed GAD when it dismissed Marcelinos petition on the ground that the latter failed to incorporate a written explanation to justify service by mail of his pleading in lieu of the preferred mode of personal service (Rule 13, Sec. 11) W/N Rule 13, Sec. 11 is merely directory HELD NO Sec. 11, Rule 13 is mandatory, CA therefore did not commit GAD RATIO Personal service and filing are preferred for obvious reasons. Plainly, such should expedite action or resolution on a pleading, motion or other paper; and conversely, minimize, if not eliminate, delays likely to be incurred if service or filing is done by mail, considering the inefficiency of postal service. Likewise, personal service will do away with the practice of some lawyers who, wanting to appear clever, resort to the following less than ethical practices: (1) serving or filing pleadings by mail to catch opposing counsel off-guard, thus leaving the latter with little or no time to prepare, for instance, responsive pleadings or an opposition; or (2) upon receiving notice from the post office that the registered parcel containing the pleading of or other paper from the adverse party may be

claimed, unduly procrastinating before claiming the parcel, or, worse, not claiming it at all, thereby causing undue delay in the disposition of such pleading or other papers. We thus take this opportunity to clarify that under Section 11, Rule 13 of the 1997 Rules of Civil Procedure, personal service and filing is the general rule, and resort to other modes of service and filing, the exception. Henceforth, whenever personal service or filing is practicable, in light of the circumstances of time, place and person, personal service or filing is mandatory. Only when personal service or filing is not practicable may resort to other modes be had, which must then be accompanied by a written explanation as to why personal service or filing was not practicable to begin with. (See also Rule 41 Voltaire Rovira vs. Heirs of Deleste) RULE 14 SUMMONS 100. Planters Development Bank v. Julie Chandumal A contract to sell was executed between BF Homes and Julie Chandumal. Later on, BF Homes sold all its rights and interests to PDB. Chandumal paid for her monthly amortizations however she began to default on her payments. PDB then gave her 30 days to settle her accounts, otherwise her rights under the contract shall be deemed extinguished. Chandumal still failed to settle her obligation. Summons were served by sheriff Roberto Galing but Chandumal was always out of the house when summons were delivered. Hence, the sheriff caused substituted service of summons through Chandumals mother who acknowledged the receipt thereof. Chandumal still failed to answer within the prescribed period so PDB filed to declare her in default. RTC issued an order granting PDBs motion. Chandumal filed an Urgent Motion to Set Aside Order of Default and to Admit Attached Answer. She said she did not receive the summons nor was she notified of the same. She alleged that PDB actually did not demand for her to pay amortization. RTC denied Chandumals motion to set aside order of default and her motion for reconsideration was also denied. RTC ruled in favor of PDB. However, CA reversed due to invalid and substituted service of summons.

PDBs contentions

RTC properly acquired jurisdiction over the person of Chandumal and that there was proper service of summons since the sheriff complied with the proper procedure as laid down in Section 7, Rule 14 of the Rules of Court.

Chandumals contentions She never received a copy of the summons or was ever notified of it, hence the RTC never acquired jurisdiction over her. Issues: 1. W/N there was valid substituted service of summons. 2. W/N Chandumal voluntarily submitted to the jurisdiction of the RTC. Held: 1. NO. There was no valid substituted service of summons. 2. YES. Chandumal voluntarily submitted to the jurisdiction of the court. Ratio: 1. There was no valid substituted service of summons. The requisites for a valid substituted service of summons are: (1) impossibility of prompt personal service (2) specific details in the return the sheriff must describe in the return of summons the facts and circumstances surrounding attempted personal service (3) a person of suitable age and discretion the sheriff must determine if the person in the alleged dwelling is of legal age, what the recipients relationship with the defendant is and whether said person comprehends the significance of the receipt of the summons; and (4) a competent person in charge. In this case, the sheriffs return failed to justify a resort to substituted service of summons. The return of summons does not specifically show or indicate in detail the actual exertion of efforts or any positive steps taken in the service of summons. There was no indication if he even asked Chandumals mother of her whereabouts. 2. Chandumal voluntarily submitted to the jurisdiction of the court. When Chandumal filed an Urgent Motion to Set Aside Order of Default, she admittedly submitted to the jurisdiction of the trial court because the filing of the pleading is equivalent to service of summons and vests the trial court with jurisdiction over the defendants person. ARNEL SAGANA v. RICHARD FRANCISCO (Rule 14) FACTS:

Arnel Sagana filed a Complaint before w/ the RTC, to recover damages alleging that Richard A. Francisco shot him with a gun hitting him on the right thigh. Process Server Manuel Panlasigue attempted to personally serve summons at Franciscos address but was unsuccessful. RTC also attempted to serve summons to Franciscos office through registered mail, however, Francisco failed to pick up summons. The case was dismissed by the RTC on account of Saganas lack of interest to prosecute since he did not take action since the filing of the Servers Return. Sagana filed an MR, contended that he exerted efforts to locate the Francisco, it was confirmed that Francisco indeed lived at his given address in Q.C. RTC granted the Motion with a condition upon the service of summon on the Francisco within 10 days from the receipt of the Order. Process Server Jarvis Iconar tried to serve summons at Franciscos address but no avail. Franciscos brother, Michael Francisco, told him that Francisco no longer lived at the said address, however, Iconar left a copy of the summons to Francisco. Sagana filed a Motion to Declare Francisco in Default for failure to file Answer. RTC declared that the summons was validly served to Francisco, declared that Francisco in default and allowed Sagana to present his evidence ex parte. Michael Francisco, through his lawyer filed a Manifestation and Motion, he denied that he received the summons and he was authorized to receive on behalf of his brother. He asserted that he was 19 y/o, and Francisco had left the house since 1993 and Francisco would only write or call them without informing his whereabouts. RTC denied the Manifestation and Motion for lack of merit. Sagana had already sent numerous pleadings to R. Francisco at his last known address. Michael has not adduced evidence, except his affidavit of merit, to impugn the service of summons thru him. Michael herein also admits that his brother communicates with him through telephone. Michael, therefore, would be able to inform R. Francisco of the summons was sent to him. Having failed to file an answer or any responsive pleading, R. Francisco was declared in default and Sagana was allowed to present evidence ex parte. RTC ruled in favor of Sagana. R. Francisco appealed to the CA and prayed that the RTC erred in assuming jurisdiction over the person, despite the irregularity of the substituted service of summons by the court Process Server and in awarding of damages to Sagana. CA favored Francisco hence Sagana filed this petition for review.

ISSUE: W/N the substituted service of summons was validly made upon Francisco through his brother? HELD: The substituted service of summons was validly made RATIO: Section 8 of Rule 14 of the old Revised ROC, the rules of procedure then in force at the time summons was served: Section 8. Substituted service. If the defendant cannot be served within a reasonable time as provided in the preceding section [personal service on defendant], service may be effected (a) by leaving copies of the summons at the defendants residence with some person of suitable age and discretion then residing therein, or (b) by leaving the copies at defendants office or regular place of business with some competent person in charge thereof. (NEW RULE) Rule 14, Section 7. Substituted service. If, for justifiable causes, the defendant cannot be served within a reasonable time as provided in the preceding section, service may be effected (a) by leaving copies of the summons at the defendant's residence with some person of suitable age and discretion then residing therein, or (b) by leaving the copies at defendant's office or regular place of business with some competent person in charge thereof. For substituted service of summons to be valid, the following must be demonstrated: (a) that personal service of summons within a reasonable time was impossible; (b) that efforts were exerted to locate the party; and (c) that the summons was served upon a person of sufficient age and discretion residing at the party's residence or upon a competent person in charge of the party's office or regular place of business. It is likewise required that the pertinent facts proving these circumstances be stated in the proof of service or in the officer's return. The Franciscos actively attempt to frustrate the proper service of summons by refusing to give their identity, rebuffing requests to sign for or receive documents or eluding the officers of court. Undeniably, no Sheriffs Return was prepared by process server Jarvis Iconar; the only record of the second service of summons was Mr. Iconars handwritten notation in the summons itself. However, the information required by law and prevailing jurisprudence, that is, that personal service was impossible because of the claim that respondent no longer lived at the stated address, that efforts were exerted to locate the respondent through the multiple attempts to serve summons, and that summons was served upon a person of sufficient age and discretion, were already in the records of the trial court. The purpose of summons is two-fold: to acquire jurisdiction over the person of the defendant and to notify the defendant that an action has been commenced so that he may be given an opportunity to be heard on

the claim against him. Under the circumstances of this case, the SC finds that Francisco was duly apprised of the action against him and had every opportunity to answer the charges made by Sagana.

101.

LEAH PALMA VS. HON. DANILO GALVEZ

Facts: Petitioner Leah Palma filed with the RTC an action for damages against the Philippine Heart Center and its doctors and nurses professional fault, negligence and omission for having removed her right ovary against her will, and losing the same and the tissues extracted from her during the surgery. One of the defendants is private respondent Psyche Elena Agudo, one of the nurses during the surgery. The RTCs process server submitted his return of summons stating that the alias summons, together with a copy of the amended complaint and its annexes, were served upon private respondent thru her husband Alfredo Agudo, who received and signed the same as private respondent was out of the country. Agudo filed a Motion to Dismiss on the ground that the RTC had not acquired jurisdiction over her as she was not properly served with summons, since she was temporarily out of the country; that service of summons on her should conform to Section 16, Rule 14 of the Rules of Court. Palma filed her Opposition to the motion to dismiss, arguing that a substituted service of summons on private respondents husband was valid and binding on her; that service of summons under Section 16, Rule 14 was not exclusive and may be effected by other modes of service, i.e., by personal or substituted service. Issue:W/N there was a valid service of summons on private respondent. Held: Yes. Considering that private respondent was temporarily out of the country, the summons and complaint may be validly served on her through substituted service under Section 7, Rule 14 of the Rules of Court. Ratio: Section 16 of Rule 14 uses the words may and also, it is not mandatory. Other methods of service of summons allowed under the Rules may also be availed of by the serving officer on a defendantresident who is temporarily out of the Philippines. Thus, if a resident defendant is temporarily out of the country, any of the following modes of service may be resorted to: (1) substituted service set forth in

section 7, Rule 14; (2) personal service outside the country, with leave of court; (3) service by publication, also with leave of court; or (4) in any other manner the court may deem sufficient. Under Section 7 of Rule 14, substituted service may be made by leaving copies of the summons at the defendants residence with some person of suitable age and discretion then residing therein. The service was made at the defendants residence and her husband acknowledged receipt thereof. The husband was presumably of suitable age and discretion, who was residing in that place and, therefore, was competent to receive the summons on private respondents behalf. 102. MA. IMELDA MANOTOC v. CA (included on 16 December 2013, previously filed under Rule 16) FACTS: Petitioner is the defendant in Civil Case for Filing, Recognition and/or Enforcement of Foreign Judgment. Respondent Trajano seeks the enforcement of a foreign courts judgment in a case for wrongful death of deceased Archimedes Trajano committed by military intelligence officials of the Philippines allegedly under the command of defendant Manotoc, pursuant to the provisions of Rule 39 of the then Revised Rules of Court. The trial court issued a Summons addressed to petitioner. The Summons and a copy of the Complaint were allegedly served upon Macky de la Cruz, an alleged caretaker of petitioner. When petitioner failed to file her Answer, the trial court declared her in default. Petitioner, filed a Motion to Dismiss on the ground of lack of jurisdiction of the trial court over her person due to an invalid substituted service of summons. Agapita Trajano, presented Robert Swift, lead counsel for plaintiffs who testified that he participated in the deposition taking of Ferdinand R. Marcos, Jr.; and he confirmed that Mr. Marcos, Jr. testified that petitioners residence was at the Alexandra Apartment, Greenhills. In addition, the entries in the logbook of Alexandra Homes listing the name of petitioner Manotoc and the Sheriffs Return, were adduced in evidence. The trial court rejected Manotocs Motion to Dismiss on the strength of its findings that her residence, was Alexandra Homes, Unit E-2104, No. 29 Meralco Avenue, Pasig, Metro Manila, based on the documentary evidence of respondent Trajano. The trial court relied on the presumption that the sheriffs substituted service was made in

the regular performance of official duty, and such presumption stood in the absence of proof to the contrary. Manotoc filed a Petition for Certiorari and Prohibition before the Court of Appeals. The CA rendered the assailed Decision, dismissing the Petition for Certiorari and Prohibition. The CA also rejected petitioners Philippine passport as proof of her residency in Singapore as it merely showed the dates of her departure from and arrival in the Philippines without presenting the boilerplates last two inside pages where petitioners residence was indicated. Thus, according to the CA, the trial court had acquired jurisdiction over petitioner as there was a valid substituted service.

ISSUES: 1. Whether or not validity of the substituted service of summons for the trial court to acquire jurisdiction over petitioner was vaild HELD: 1. No, the service of summons was not valid. RATIO: 1. Jurisdiction over the defendant is acquired either upon a valid service of summons or the defendants voluntary appearance in court. When the defendant does not voluntarily submit to the courts jurisdiction or when there is no valid service of summons, any judgment of the court which has no jurisdiction over the person of the defendant is null and void. If defendant, for excusable reasons, cannot be served with the summons within a reasonable period, then substituted service can be resorted to. The party relying on substituted service or the sheriff must show that defendant cannot be served promptly or there is impossibility of prompt service. Under the Rules, the service of summons has no set period. However, when the court, clerk of court, or the plaintiff asks the sheriff to make the return of the summons and the latter submits the return of summons, then the validity of the summons lapses. The plaintiff may then ask for an alias summons if the service of summons has failed. The Sheriffs Return provides data to the Clerk of Court, which the clerk uses in the Monthly Report of Cases to be submitted to the Office of the Court Administrator within the first ten (10) days of the succeeding month. For substituted service of summons to be available, there must be several attempts by the sheriff to personally serve the summons within a reasonable period [of one month] which eventually resulted in failure to prove impossibility of prompt service. Several attempts means at least three tries,

preferably on at least two different dates. In addition, the sheriff must cite why such efforts were unsuccessful. It is only then that impossibility of service can be confirmed or accepted.

The sheriff must describe in the Return of Summons the facts and circumstances surrounding the attempted personal service. The efforts made to find the defendant and the reasons behind the failure must be clearly narrated in detail in the Return. The form on Sheriffs Return of Summons on Substituted Service prescribed in the Handbook for Sheriffs requires a narration of the efforts made to find the defendant personally and the fact of failure. If the substituted service will be effected at defendants house or residence, it should be left with a person of suitable age and discretion then residing therein. Thus, the person must have the relation of confidence to the defendant, ensuring that the latter would receive or at least be notified of the receipt of the summons. The sheriff must therefore determine if the person found in the alleged dwelling or residence of defendant is of legal age, what the recipients relationship with the defendant is, and whether said person comprehends the significance of the receipt of the summons and his duty to immediately deliver it to the defendant or at least notify the defendant of said receipt of summons. A scrutiny of the aforementioned Return readily reveals the absence of material data on the serious efforts to serve the Summons on petitioner Manotoc in person. There is no clear valid reason cited in the Return why those efforts proved inadequate, to reach the conclusion that personal service has become impossible or unattainable. It has not been shown that respondent Trajano or Sheriff Caelas, who served such summons, exerted extraordinary efforts to locate petitioner. Before resorting to substituted service, a plaintiff must demonstrate an effort in good faith to locate the defendant through more direct means. More so, in the case in hand, when the alleged petitioners residence or house is doubtful or has not been clearly ascertained, it would have been better for personal service to have been pursued persistently. Granting that such a general description be considered adequate, there is still a serious nonconformity from the requirement that the summons must be left with a person of suitable age and discretion residing in defendants house or residence. The Sheriffs Return lacks information as to residence, age, and discretion of Mr. Macky de la Cruz. It is doubtful if Mr. de la Cruz

is residing with petitioner Manotoc in the condominium unit considering that a married woman of her stature in society would unlikely hire a male caretaker to reside in her dwelling. With the petitioners allegation that Macky de la Cruz is not her employee, servant, or representative, it is necessary to have additional information in the Return of Summons. The court a quo heavily relied on the presumption of regularity in the performance of official duty. The Court acknowledges that this ruling is still a valid doctrine. However, for the presumption to apply, the Sheriffs Return must show that serious efforts or attempts were exerted to personally serve the summons and that said efforts failed. These facts must be specifically narrated in the Return. Otherwise, the Return is flawed and the presumption cannot be availed of.

RECALL RAPID CITY REALTY V. ORLANDO VILLA UNDER RULE 9


RULE 15 MOTIONS

RECALL: MBTC v. UNDER: Rule 7 FOR Rule 15!

Hon.

Salvador

Santos

103. MANUEL N. MAMBA, et. al. vs. EDGAR R. LARA, et. al. FACTS: Several resolutions were passed by the SangguniangPanlalawigan of Cagayan regarding the authorization for Gov. Lara to engage the services of and appoint Preferred Ventures Corp. (PVC) as financial advisor for the issuance and floatation of bonds to fund the priority projects of the governor and ratification of several agreements entered into by Gov. Lara in connection with the bond floatation.
(payment of one-time fee of 3% to PVC, construction and improvement of priority projects, construction and development of the new Cagayan Town Center and those Cagayan Provincial Bond Agreements with RCBC, MICO and LBP)

Then, Gov. Lara awarded the project of construction of the Town Center to Asset Builders Corp. (ABC) for P213,795,732.39 Manuel Mamba (Cong. Of 3rd district of Cagayan), Guzman and Fausto (members of Sanggunian) filed a Petition for Annulment of Contracts and Injunction with Prayer for Temporary Restraining Order/Writ of Preliminary Injunction against Lara as Governor, other members of the Sanggunian, PVC, ABC, RCBC, MICO and LBP.

Respondents filed their Answers with Motion to Dismiss alleging that Petitioners have no legal standing to file the suit and the issues raised were purely political. Petitioners then filed a Motion to Admit Amended Petition attaching thereto the amended petition. But Respondents opposed the motion for the following reasons: 1. Motion was belatedly filed 2. Amended Petition is not sufficient in form and in substance 3. Motion is patently dilatory Petitioners also filed a Consolidated Opposition to the Motion to Dismiss but the RTC denied the Motion to Admit Amended Petition and dismiss the Petition for lack of cause of action So the Petitioners filed a Motion for Reconsideration but the RTC denied it the court found the motion to be a mere scrap of paper as the notice to hearing was addressed only to the Clerk of Court in violation of Sec. 5 Rule 15 of the Rules of Court.

3.A perusal of the Motion for Reconsideration filed by petitioners would show that the notice of hearing was addressed only to the Clerk of Court in violation of Sec. 5, Rule 15, which requires the notice of hearing to be addressed to all parties concerned. This defect, however, did not make the motion a mere scrap of paper. The rule is not a ritual to be followed blindly. The purpose of a notice of hearing is simply to afford the adverse parties a chance to be heard before a motion is resolved by the court. In this case, respondents were furnished copies of the motion, and consequently, notified of the scheduled hearing. Counsel for public respondents in fact moved for the postponement of the hearing, which the court granted. Moreover, respondents were afforded procedural due process as they were given sufficient time to file their respective comments or oppositions to the motion. From the foregoing, it is clear that the rule requiring notice to all parties was substantially complied with. In effect, the defect in the Motion for Reconsideration was cured. We cannot overemphasize that procedural rules are mere tools to aid the courts in the speedy, just and inexpensive resolution of cases. Procedural defects or lapses, if negligible, should be excused in the higher interest of justice as technicalities should not override the merits of the case. Dismissal of cases due to technicalities should also be avoided to afford the parties the opportunity to present their case. Courts must be reminded that the swift unclogging of the dockets although a laudable objective must not be done at the expense of substantial justice. 104. Facts: Petitioner Fajardos filed a complaint against respondent Alberto Comandante and the Register of Deeds of Iba, Zambales, before the RTC of Olongapo City.The Fajardos asked the court to annul the deed of sale that they supposedly executed in Albertos favor over a parcel of land in Subic, Zambales, their signatures on the document having been forged. In his answer, Alberto denied the Fajardos allegations and claimed that the deed of sale had been regularly executed and the subject property validly transferred to him. The Fajardos did not ask the RTC to set it for pre-trial hearing. This omission prompted Alberto, after 6 months, to file a motion to dismiss the Fajardos action for failure to prosecute. And when the court required the Fajardos to oppose the motion or file a comment on it, they did not. Still, on February 7, 2006 the court denied Albertos motion to dismiss. When Alberto filed his February 9, 2006 motion for reconsideration of the denial of his motion, the court again directed the Fajardos to oppose the motion or file a comment on it but, like before, they did not, eventually persuading the court on February 27, 2006 to dismiss their action "without" prejudice. RUFINA FAJARDO, et al. v. ALBERTO COMANDANTE, et al.

ISSUES: 1. Whether the petitioners have locus standi to file the case? 2. Do the issues raised involved political questions? 3. (MAIN TOPIC) Whether the RTC erred in finding that the Motion for Reconsideration filed by the petitioners was in violation of Sec.5, Rule 15? HELD: 1. YES, as taxpayers since the Bond Agreements involved disbursements of public funds. 2. NO, the court has jurisdiction if it involves grave abuse of discretion amount to lack or excess of jurisdiction. 3. YES the RTC erred, Sec.5, Rule 15 of the Rules of Court was substantially complied with. RATIO:
1. The decision to entertain a taxpayers suit is discretionary upon the Court. It can choose to strictly apply the rule or take a liberal stance depending on the controversy involved. Advocates for a strict application of the rule believe that leniency would open floodgates to numerous suits, which could hamper the government from performing its job. Such possibility, however, is not only remote but also negligible compared to what is at stake - "the lifeblood of the State". For this reason, when the issue hinges on the illegal disbursement of public funds, a liberal approach should be preferred as it is more in keeping with truth and justice. 2.Obviously, the issues raised in the petition do not refer to the wisdom but to the legality of the acts complained of. Thus, we find the instant controversy within the ambit of judicial review. Besides, even if the issues were political in nature, it would still come within our powers of review under the expanded jurisdiction conferred upon us by Sec. 1, Art. VIII of the Constitution, which includes the authority to determine whether grave abuse of discretion amounting to excess or lack of jurisdiction has been committed by any branch or instrumentality of the government.

Unsatisfied, Alberto filed a manifestation/motion, substantially asking the RTC to reconsider its February 27, 2006 Order and dismiss the complaint with prejudice. For their part, the Fajardos filed a motion for reconsideration of the same Order and asked that the case be set for pre-trial. On April 4, 2006, the trial court denied both motions. It explained that it could not dismiss the complaint "with prejudice" since it was more of counsels fault rather than of the Fajardos that the case had not moved. The trial court could not, on the other hand, grant the Fajardos motion because their counsel did not bother to inform it that he could not appear at the hearing due to some illness and because the motion did not conform to the notice requirements of Rule 15 of the Rules of Civil Procedure. On April 24, 2006 Alberto again moved for reconsideration, insisting that the trial courts dismissal of the complaint should be with prejudice. On May 11, 2006, the trial court, now finding that neither the Fajardos nor their counsel had shown real interest in pressing their action even up to that time, reconsidered its earlier order and dismissed the complaint "with"prejudice. On September 11, 2006 the Fajardos filed a motion for the trial court to treat Albertos April 24, 2006 motion for reconsideration, which had already been granted and resulted in the dismissal of the case with prejudice, as a mere scrap of paper for its non-compliance with the three-day notice rule. Albertos motion, according to them, did not produce any legal effect such as the courts Order of May 11, 2006 that dismissed their complaint with prejudice. On October 4, 2006 the trial court issued an order, denying the Fajardos motion. It chided them for not even bothering to file any motion for reconsideration of its Orders of February 27 and May 11, 2006 although they were in receipt of these. The spouses moved for reconsideration. Subsequently, they submitted in support of it a certification that the post office delivered Albertos April 24, 2006 motion for reconsideration to the Fajardos counsel only on April 28, 2006, the day after the hearing of such motion. But, on finding that the Fajardos did not bother to appeal from its order dismissing the case with prejudice, rendering the same final and executory, the RTC issued an Order on May 28, 2007, denying their motion for reconsideration. Undeterred, the Fajardos filed a petition for certiorari with the Court of Appeals. Court of Appeals rendered a decision, dismissing the petition for failure of the Fajardos to clearly demonstrate the RTCs grave abuse of discretion and jurisdictional errors.

in not setting aside, as a consequence, the order that dismissed their case "with" prejudice. Held:NO. Ratio: Since Alberto failed to give them proper notice with respect to his April 24, 2006 motion for reconsideration that sought dismissal of the case "with" prejudice, the RTC should have treated that motion as a mere scrap of paper and not act on it. Alberto set his motion for hearing on April 27, 2006 but served a copy of it on the Fajardos counsel by registered mail, which the latter received only on April 28, 2006, one day after the scheduled hearing. Mistaken in its belief that proper notice had been given to the Fajardos, however, the RTC considered Albertos motion submitted for resolution. However, the Fajardos did nothing with Albertos motion that they received on April 28, 2006, which motion asked the RTC to change its order of dismissal of their case from "without" prejudice to one "with" prejudice. It apparently did not alarm them or their counsel at all. They did not oppose it or bother to check with the court the outcome of the motion that had been set for hearing on April 27, 2006. Yet they had ample time to do this since the RTC incurred delay in acting on Albertos motion. Only on May 11, 2006 or about two weeks later did the court grant the same and dismissed the case, now "with" prejudice. Since the RTCs May 11, 2006 Order of dismissal "with" prejudice was a final order, the Fajardos remedy was to file either a motion for its reconsideration or a notice of appeal to have the RTCs error in entertaining Albertos defective motion rectified. But the Fajardos allowed the period for filing such motion or notice of appeal to lapse. Only on September 11, 2006, four months later, did they file their motion to treat Albertos motion of April 24, 2006 as a mere scrap of paper for failing to comply with the three-day notice requirement for motions. Actually, this was in the nature of a motion for reconsideration of the May 11, 2006 Order, crudely masked to hide the fact that they filed it out of time. And, when the RTC denied this belated motion, the Fajardos resorted to the remedy of a special civil action of certiorari under Rule 65 filed with the Court of Appeals to get that May 11, 2006 Order reviewed and set aside. That remedy is not available to them. Since the Fajardos did not appeal from the May 11, 2006 Order of the RTC, the same became final and executory as a matter of course. It can no longer be modified in any respect, even if the modification is meant to

Issue: Whether or not the RTC gravely abused its discretion a) in not treating Albertos April 24, 2006 motion for reconsideration as a mere scrap of paper for non-compliance with the three-day notice rule and b)

correct erroneous conclusions of fact and law, and whether it is made by the court that rendered it or by an appellate court. As a final and valid order, it could not be collaterally attacked through the Fajardos artful motion to treat Albertos April 24, 2006 motion as a scrap of paper, where the sole object, in truth, is the nullification of the May 11, 2006 Order. RULE 16 MOTION
TO

Yes. RTC did not dismiss the case on the ground that the complaint did not state a cause of action, which is an entirely different matter. The case must be dismissed on the ground of abandonment of claim/demand. RATIO: Section 1, Rule 16: The trial court may dismiss a complaint on the ground that the claim or demand set forth in the plaintiffs complaint has been paid, waived, abandoned, or otherwise extinguished. This ground essentially admits the obligation set out in the complaint but points out that such obligation has been extinguished, in this case apparently by abandonment after respondent Molave Development received partial reimbursement from Medina as a consequence of the cancellation of contract to sell between them. After it filed its complaint with the RTC, Molave Development acknowledged having received P1.3 million as a consideration for the cancellation of its contract to sell with Medina. Molave Development has no valid claim against petitioner Doa Rosana Realty and its President. 106. Subic Telecom Authority Company vs. Subic Bay Metropolitan

D ISMISS

105. DONA ROSANA REALTY VS. MOLAVE DEVT CORP Rule 16: Motion to Dismiss FACTS: Carmelita Austria Medina executed a contract to sell her land to Molave Development Corporation, for P14M. Molave paid P1M to Medina upon the signing of the contract and P1.3M more as first installment. But it refused to pay the rest of the installments on being informed by the DAR of the existence of alleged tenants on the land.Two years later, Medina wrote Molave a letter, rescinding the contract to sell. Molave later learned that a month earlier, Medina sold the land to Doa Rosana Realty.Molave sued Medina, Doa Rosana Realty, and its President. The RTC declared Medina in default. Doa Rosana Realty, on the other hand, filed an answer and a motion to set the case for preliminary hearing on its special and affirmative defenses claiming that it acted in good faith in purchasing the property and that Molave was estopped from questioning the sale because it agreed to cancel the contract to sell and, after the complaint was filed, its presidentreceived from Medinas counsel a P1.3M partial reimbursement.RTC denied Doa Rosana Realtys motion to dismiss but, on petition, the CA directed the RTC to conduct a preliminary hearing on Doa Rosana Realtys special affirmative defense of good faith.The RTC did so and it dismissed the complaint insofar as Doa Rosana Realty and Sy Ka Kieng were concerned. But the CA reversed finding there is cause of action against them and remanded the case to the RTC for further proceedings. ISSUE: Whether or not there is ground for dismissing Molave Developments complaint against Doa Rosana Realty given that such complaint stated a cause of action. HELD:

Rule 16, sec 1 SBMA wanted to upgrade the antiquated telephones system left in the US base. After holding an international competitive bidding, PLDT and AT&T entered into a JVA. The JVA gave birth to Subic Telecom. SBMA then granted Subic Telecom its franchise. JVA also made the subic telecom the exclusive telecom provider for 10 yrs through sec 11 of the covenant attesting the JVA. Before the end of the 10 yr period, Subic telecom wanted to exercise its exclusivity contract for an extended period of 5 years. Subic telecom notified SBMA about it.SBMA did not answer. SBMA started accepting applications for the franchise like those coming from Innove communications. In lieu of its new policy of liberalizing the telecom sector in the SBFZ, SBMA ratified its denial on the exclusivity contract asserted by Subic telecom. It also granted innove communications provisional authority to operate inside the zone. Aggrieved by the action of SBMA, Subic telecom filed two cases: One case was filed before the SBMA telecom department, for a CPCN to operate international and leased lines services as well as toll services.

The other case was filed before the RTC for specific performance and TPO with preliminary injunction.

the allegations and affirmative defenses raised by AB. The CA, however, denied the motion for reconsideration. ISSUE: Whether or not it is proper to file a motion to dismiss after an answer has already been filed; HELD: Yes. Respondents failed to oppose the motion to dismiss despite having been given the opportunity to do so by the RTC. Therefore, any right to contest the same was already waived by them.Petition is DENIED for lack of merit. The Regional Trial Court is ordered to proceed with the trial of the case with dispatch. RATIO: Section 6, Rule 16 of the Rules of Court: If no motion to dismiss has been filed, any of the grounds for dismissal provided for in this Rule may be pleaded as an affirmative defense in the answer and, in the discretion of the court, a preliminary hearing may be had thereon as if a motion to dismiss had been filed.The dismissal of the complaint under this section shall be without prejudice to the prosecution in the same or separate action of a counterclaim pleaded in the answer. The rule is based on practicality. Both the parties and the court can conveniently save time and expenses necessarily involved in a case preparation and in a trial at large, when the issues involved in a particular case can otherwise be disposed of in a preliminary hearing. It isinconsequential that petitioner had already filed an answer to the complaint prior to its filing of a motion to dismiss. The option of whether to set the case for preliminary hearing after the filing of an answer which raises affirmative defenses, or to file a motion to dismiss raising any of the grounds set forth in Section 1, Rule 16 of the Rules are procedural options which are not mutually exclusive of each other.

RTC dismissed the action on the ground of litis pendencia. CA affirmed. Issue: did RTC commit GAD in dismissing the complaint under Rule 16, sec 1? Held: Yes. There can be no litis pendencia since the first action was for an application to operate international and leased lines services, while the other was for specific performance. Subic cannot be made to enforce specific performance in the admin case since SBMA is also a party. The resolution of the adm case would not necessarily settle the rights being asserted by subic telecom under the contract signed with SBMA. 107. ASSOCIATED BANK VS.SPS. MONTANO Rule 16: Motion to Dismiss FACTS: Spouses Justiniano and Ligaya Montano owned parcels of land in Tanza, Cavite utilized as an integrated farm and as a stud farm used for raising horses.Justiniano was then serving as congressman for the lone district of Cavite and as minority floor leader. When then Pres. Marcos placed the country under martial law, Justiniano went on self-exile to the US to avoid the harassment and threats made against him by Marcos. While still in the USA, the Montanos transferred the properties to Tres Cruces Agro-Industrial Corporation (TCAIC) in exchange for shares of stock.Later, TCAIC sold the properties to International Country Club, Inc. (ICCI) which mortgaged the land to Citizens Bank and Trust Co. (later renamed as Associated Bank). The loan matured but remained unpaid, prompting Associated Bank (AB) to foreclose the mortgage.Later, ownership over the said properties was consolidated by AB. Following the ouster of Marcos, the Montanos returned to the country. They filed an action for reconveyance of title against AB.The trial court dismissed the complaint as barred by the statute of limitations.CA reversed the RTCs ruling and reinstated the case for further proceedings. AB moved for reconsideration,arguing that the cause of action of the Montanos, if there had been any, had already prescribed pointing out that the failure of the Montanos to file a comment on or an objection to the motion to dismiss despite opportunity to do so should be construed as a waiver in contesting

108. MUNICIPALITY OF HAGONOY, BULACAN, V. HON. SIMEON P. DUMDUM, JR. G.R. No. 168289 | March 22, 2010 Facts: The case stems from a Complaint by private respondent Emily Rose Go Ko Lim Chao against herein petitioners, the Municipality

of Hagonoy, Bulacan and its chief executive, Felix V. Ople (Ople) for collection of a sum of money and damages. Respondent, doing business as KD Surplus and as such engaged in buying and selling surplus trucks, heavy equipment, machinery, spare parts and related supplies, was contacted by petitioner Ople. Respondent had entered into an agreement with petitioner municipality through Ople for the delivery of motor vehicles, which supposedly were needed to carry out certain developmental undertakings in the municipality. Respondent claimed that because of Oples earnest representation that funds had already been allocated for the project, she agreed to deliver twenty-one motor vehicles whose value totaled P5,820,000.00. She attached to the complaint copies of the bills of lading showing that the items were consigned, delivered to and received by petitioner municipality. However, despite having made several deliveries, Ople allegedly did not heed respondents claim for payment. Respondent prayed for full payment of the said amount, with interest and damages. Instead of addressing private respondents allegations, petitioners filed a Motion to Dismisson the ground that the claim on which the action had been brought was unenforceable under the statute of frauds, pointing out that there was no written contract or document that would evince the supposed agreement they entered into with respondent. In other words, no contract was ever entered into by the local government with respondent. Petitioners advanced that the bills of lading attached to the complaint were hardly probative, inasmuch as these documents had been accomplished and handled exclusively by respondent herself as well as by her employees and agents. Petitioners also filed a Motion to Dissolve and/or Discharge the Writ of Preliminary Attachment Already Issued, invoking immunity of the state from suit, unenforceability of the contract, and failure to substantiate the allegation of fraud.

may be proved. But it does not declare them invalid because they are not reduced to writing inasmuch as, by law, contracts are obligatory in whatever form they may have been entered into, provided all the essential requisites for their validity are present. The object is to prevent fraud and perjury in the enforcement of obligations depending, for evidence thereof, on the unassisted memory of witnesses by requiring certain enumerated contracts and transactions to be evidenced by a writing signed by the party to be charged. The effect of noncompliance with this requirement is simply that no action can be enforced under the given contracts. If an action is nevertheless filed in court, it shall warrant a dismissal under Section 1(i), Rule 16 of the Rules of Court, unless there has been, among others, total or partial performance of the obligation on the part of either party. As far as the private respondent is concerned, she has already performed her part of the obligation under the agreement by undertaking the delivery of the 21 motor vehicles contracted for by Ople in the name of petitioner municipality. This claim is well substantiated by copies of the bills of lading attached to the complaint, naming petitioner municipality as consignee of the shipment. Since there exists an indication by way of allegation that there has been performance of the obligation on the part of respondent, the case is excluded from the coverage of the rule on dismissals based on unenforceability under the statute of frauds, and either party may then enforce its claims against the other. No other principle in remedial law is more settled than that when a motion to dismiss is filed, the material allegations of the complaint are deemed to be hypothetically admitted. This hypothetical admission extends not only to the relevant and material facts well pleaded in the complaint, but also to inferences that may be fairly deduced from them. Thus, where it appears that the allegations in the complaint furnish sufficient basis on which the complaint can be maintained, the same should not be dismissed regardless of the defenses that may be raised by the defendants. Stated differently, where the motion to dismiss is predicated on grounds that are not indubitable, the better policy is to deny the motion without prejudice to taking such measures as may be proper to assure that the ends of justice may be served. It is interesting to note at this point that in their bid to have the case dismissed, petitioners theorize that there could not have been a contract by which the municipality agreed to be bound, because it was not shown that there had been compliance with the required bidding or that the municipal scouncil had approved the contract. The argument is flawed. By invoking unenforceability under the Statute of Frauds, petitioners are in effect acknowledging the existence of a contract between them and private respondent only, the said contract cannot be enforced by action for being non-compliant with the legal requisite that it be reduced into writing. Suffice it to say that while this assertion

Issue: WON there was grave abuse of discretion has been committed by the trial court in denying petitioners motion to dismiss Held: NO Ratio: To begin with, the Statute of Frauds found in paragraph (2), Article 1403 of the Civil Code, requires for enforceability certain contracts enumerated therein to be evidenced by some note or memorandum and require certain classes of contracts to be in writing. However, the Statute of Frauds only lays down the method by which the enumerated contracts

might be a viable defense against respondents claim, it is principally a matter of evidence that may be properly ventilated at the trial of the case on the merits. Verily, no grave abuse of discretion has been committed by the trial court in denying petitioners motion to dismiss this case. The Court of Appeals is thus correct in affirming the same.

upon the same. The determination of whether a specific rule or set of rules issued by an administrative agency contravenes the law or the constitution is within the jurisdiction of the regular courts. Indeed, the Constitution vests the power of judicial review or the power to declare a law, treaty, international or executive agreement, presidential decree, order, instruction, ordinance, or regulation in the courts, including the regional trial courts In the case at bar, the issuance by the NTC of the memorandum ciruclars was pursuant to its quasi-legislative or rule-making power. As such, petitioners were justified in invoking the judicial power of the Regional Trial Court to assail the constitutionality and validity of the said issuances. In Drilon v. Lim,it was held: Specifically, B.P. 129 vests in the regional trial courts jurisdiction over all civil cases in which the subject of the litigation is incapable of pecuniary estimation, even as the accused in a criminal action has the right to question in his defense the constitutionality of a law he is charged with violating and of the proceedings taken against him, particularly as they contravene the Bill of Rights. Moreover, Article X, Section 5(2), of the Constitution vests in the Supreme Court appellate jurisdiction over final judgments and orders of lower courts in all cases in which the constitutionality or validity of any treaty, international or executive agreement, law, presidential decree, proclamation, order, instruction, ordinance, or regulation is in question. In their complaint before the Regional Trial Court, petitioners averred that the Circular contravened Civil Code provisions on sales and violated the constitutional prohibition against the deprivation of property without due process of law. These are within the competence of the trial judge. Contrary to the finding of the Court of Appeals, the issues raised in the complaint do not entail highly technical matters. Rather, what is required of the judge who will resolve this issue is a basic familiarity with the workings of the cellular telephone service, including prepaid SIM and call cards and this is judicially known to be within the knowledge of a good percentage of our population and expertise in fundamental principles of civil law and the Constitution. 110. JC Lopez v. COA

109.

SMART V. NTC

Res Judicata Facts: National Telecommunications Commission (NTC) issued various memorandum circulars promulgating rules and regulations on the billing of telecommunications services, restrictions on the use of prepaid sim cards requiring the registration of which, in order to minimize if not totally eliminate the incidence of stealing of cellular phone units. Isla Communications Co., Inc. and Pilipino Telephone Corporation filed against NTC, a declaration of nullity of the memorandum circulars with prayer for the issuance of TRO. They alleged that the NTC has no jurisdiction to regulate the sale of consumer goods such as the prepaid call cards since such jurisdiction belongs to the Department of Trade and Industry under the Consumer Act of the Philippines; that the Billing Circular is oppressive, confiscatory and violative of the constitutional prohibition against deprivation of property without due process of law. Soon thereafter, Globe Telecom and Smart Communicationsfiled a joint Motion for Leave to Intervene and to Admit Complaint-inIntervention. RTC issued a TRO. In the mean time, NTC and its co-defendants filed a motion to dismiss the case on the ground of petitioners failure to exhaust administrative remedies.

Issue: W/N the RTC can take cognizance of the case Held: YES Ratio: Where what is assailed is the validity or constitutionality of a rule or regulation issued by the administrative agency in the performance of its quasi-legislative function, the regular courts have jurisdiction to pass

Res Judicata Facts: JC lopez Inc. entered into a contract with NAPOCOR for the dredging of

the vicinity of the Intake Tower at the Hydroelectric plant at Benguet. JC started the actual dredging works. Due to the delays incurred by JC resulting in substantial slippages. NAPOCOR terminated their contract. JCs letter seeking reconsideration of the termination was denied by NAPOCOR. NAPOCOR entered into a negotiated contract with a consortium led by Meralco Industrial Engineering Services Corporation (MIESCOR) to rehabilitate, the Ambuklao Hydroelectric Plant, including the dredging of silt within the vicinity of the intake tower. JC filed a complaint for injunction assailing NAPOCORs termination of its contract. Issue: W/N the dredging of the Ambuklao water reservoir is to be considered an infrastructure work, and therefore, within the coverage of P.D. 1818, and as such, may not be enjoined(stopped) or whether the same is merely a service undertaking and therefore, outside the ambit of [the] said decree? Held:Within PD 1818, due to res judicata Ratio: 1. since the dredging improves the efficiency of the power plant, such undertaking constitutes an infrastructure project as defined in EO 380: Infrastructure projects shall mean construction, improvement or rehabilitation of roads and bridges, railways, airports, seaports, communication facilities, irrigation, flood control and drainage, water supply and sewerage systems, shore protection, power facilities, national buildings, school buildings, hospital buildings, and other related construction projects that form part of the government capital investment. 2. Decision of the CA in Meralco Industrial Engineering Services Corporation vs. Hon. Romeo F. Zamora and J.C. Lopez, Inc.,: In Executive Order No. 380, issued by the former President of the Philippines, Corazon C. Aquino, dated November 27, 1989, the term infrastructure projects is defined as construction, improvement of [should read or] rehabilitation of roads and bridges, railways, airports, seaports, communication facilities, irrigation, flood control and drainage, water supply and sewerage systems, shore protection, power facilities, national buildings, school buildings, hospital buildings, and other related construction projects that form part of the government capital

investment. Material facts or questions which were in issue in a former action and were there admitted or judicially determined are conclusively settled by a judgment rendered therein and that such facts or questions become res judicata and may not again be litigated in a subsequent action between the same parties or their privies, regardless of the form the issue may take in the subsequent action, whether the subsequent action involves the same or a different form of proceeding, or whether the second action is upon the same or a different cause of action, subject matter, claim or demand, as the earlier action. In such cases, it is also immaterial that the two actions based on different grounds, or tried on different theories, or instituted for different purposes, and seek different reliefs. By the same token, whatever is once irrevocably established as the controlling legal principle or decision continues to be the law of the case between the same parties in the same case, whether correct on general principles or not, so long as the facts on which such decision was predicated continue to be the facts of the case before the court. Upon attaining finality, the said decision became the law of the case and constituted a bar to any re-litigation of the same issue in any other proceeding under the principle of res judicata.

111.

SPS. FERNANDO TORRES v. MEDINA

(Rule 16 sec 1 (e) FACTS: Sps Torres mortgaged their property to Amparo Medina. Failing to pay, Medina extra judicially foreclosed the REM. The property was sold to Medina where he is the highest bidder in May 1997. In 1999 Sps. Torres filed for a declaration of nullity of the extrajudicial foreclosure of mortgage w/ the RTC alleging that the REM itself was invalid and the foreclosure is premature and improper. In turn, Medina filed a Motion to Dismiss raising the grounds of res judicata and forum shopping. Apprently, Sps. Torres already filed a complaint for the annulment of REM in March 1997 involving the same property before the RTC. The 1997 Decision: Sps. Torres tried to annul the REM w/ the RTC in 1993. RTC upheld the REMs validity so Sps. Appealed to the CA, but they filed their brief out of time so the appeal was dimissmed. Sps filed for an MR but was denied so they elevated the case to the SC. In 1999 SC denied the petition of the Sps.

RTC ruled in favor of Medina and said that res judicata was present and that the Sps. Torres were guilty of forum shopping. RTC said that while the Sps. Torres alleged separate causes of action in the instant complaint, they are actually using the very same grounds they have brought before the 1997 decision to support their claim to annul the foreclosure proceedings. The validity of the REM is again being assailed to ask for the annulment of the foreclosure proceedings conducted over the mortgaged property. The REM was deemed valid in the 1997 decision, w/c has already attained finality. The test of identity of causes of action lies not in the form of an action but on whether the same evidence would support and establish the former and present causes of action. Sps. Torres cannot avoid the application of res judicata by simply varying the form of their action or by adopting a different method in presenting it. Sps. Torres appealed to the CA but the CA ruled in the same fashion that there is res judicata. Sps. Torres filed an MR but was denied hence this decision.

the prior judgment shall not constitute a bar to subsequent actions. The SC opined that any affirmative relief that this Court may grant on the current case would affect the validity of the real estate mortgage; an issue which could no longer be revived, as the same has been settled. The doctrine of res judicata actually embraces two different concepts: (1) bar by former judgment and (b) conclusiveness of judgment. The second concept conclusiveness of judgment only requires identity of issues not identity of cause of action. Res judicata, as a ground for dismissal, is based on two grounds (1) public policy and necessity, which makes it to the interest of the State that there should be an end to litigation --- republicae ut sit litium; and (2) the hardship on the individual of being vexed twice for the same cause --- nemo debet bis vexari et eadem causa. Finally Sps. Torres only filed the current complaint after 2 years from the sale of the property. The Sps. were not completely left without any remedy as they still had the right of redemption. The SC must assume that no attempt to redeem the property was undertaken by the Sps Torres and that they simply allowed their right and remedy to lapse by their inaction. (For recit purposes) Among the allegations of Sps. Torres, one involves the double compensation that Medina will receive since Medina will be able to already collect from them in a criminal suit for BP 22 (Sps. Torres issued RCBC checks to pay, but they were void for insufficiency of funds). The court said that BP 22 is not a collection case which bars a mortgagee from later on electing to foreclose the mortgaged property. The fine provided for in BP 22 was intended as an additional penalty for the act of issuing a worthless check. 112. NATIONAL HOUSING AUTHORITY VS. CORAZON BAELLO

ISSUE: W/N res judicata bars the filing of the 1999 RTC case. HELD: Res judicata is a ground for a Motion to Dismiss a subsequent case. RULING: Res judicata lays the rule that an existing final judgment or decree rendered on the merits, and without fraud or collusion, by a court of competent jurisdiction, upon any matter within its jurisdiction, is conclusive of the rights of the parties or their privies, in all other actions or suits in the same or any other judicial tribunal of concurrent jurisdiction on the points and matters in issue in the first suit. The elements of res judicata are: (1) the judgment sought to bar the new action must be final; (2) the decision must have been rendered by a court having jurisdiction over the subject matter and the parties; (3) the disposition of the case must be a judgment on the merits; and (4) there must be as between the first and second action identity of parties, subject matter, and causes of action. Sps. Torres argued that the 1st 3 elements might be present but said that the 4th was lacking since the evidence necessary to establish the cause of action in 1999 RTC case is different from the 1997 decision (See BP 22 issue). The SC has previously employed various tests in determining whether or not there is identity of causes of action as to warrant the application of the principle of res judicata. One test of identity is the "absence of inconsistency test" where it is determined whether the judgment sought will be inconsistent with the prior judgment. If no inconsistency is shown,

Res Judicata FACTS: Pedro and Nicanora Baello tiled an application for registration of a parcel of land in Caloocan with the CFI of Rizal, of the land they inherited from their mother, Esperanza Baello. The CFI of Rizal confirmed the title of the applicants and awarded the land to Pedro and Nicanora, pro indiviso. Pedro: 2/3, Nicanora: 1/3. The Republic of the Philippines, through the Director of the Bureau of Lands, did not appeal. The decision became final and executory. October 1954- The Land Registration Commission issued a decree in favor of Pedro and Nicanora. The Register of Deeds issued an OCT.

Pedro died intestate, with 32 surviving heirs including respondents Corazon, Wilhelmina, and Ernesto Baello /// Nicanora also died intestate, her husband died a few days after. President Marcos issued PD No. 569 creating a committee to expropriate the Dagat-Dagatan Lagoon and its adjacent areas, and Imelda Marcos launched the Project (program for the homeless) which covered the subject properties. Military personnel entered the Baello property and ejected the family caretaker at gunpoint. They demolished the two-storey residence and destroyed the fishpond improvements on the Baello property. The Baello and Rodriguez heirs, out of fear, did not complain. The NHA took possession and executed separate conditional contracts to sell subdivision lots in favor of chosen beneficiaries. A proclamation was issued declaring the Metropolitan Manila and the Dagat-Dagatan area, as area for priority development and Urban Land Reform Zones. Again, the subject properties were included. Minister of Natural Resources Rodolfo Del Rosario issued a BFD A.O. declaring and certifying forestlands in Caloocan City, Malabon, and Navotas as alienable or disposable for cropland and other purposes. After the People Power Revolution, the heirs of Baello executed an extrajudicial partition of Pedros estate, including the Baello property. Respondents were issued a TCT over an undivided portion of the property. Corazon and Wilhelmina later sold their shares to Ernesto. NHA filed an action for eminent domain and secured a writ of possession against the heirs of Baello and Rodriguez before the RTC of Caloocan. RTC dismissed the complaint on the ground of res judicata and lack of cause of action. The NHA appealed to the Court of Appeals. The Court of Appeals affirmed the Order of the RTC. The NHA filed a petition for review in SC. SC denied due course to the petition since the Court of Appeals did not commit any reversible error in affirming the order of the RTC. The NHA filed a motion for reconsideration but it was denied in a Resolution. The Clerk of Court later made an Entry of Judgment. The NHA filed a complaint for nullity of OCT issued in the names of Pedro and Nicanora. The RTC dismissed the complaint on grounds of estoppel and res judicata and because the issue on the legal nature and ownership of the property covered by OCT was already barred by a final judgment. The NHA appealed to the Court of Appeals which affirmed the decision of the RTC. Again, the NHA went to the SC to assail the decision of the Court of Appeals. SC denied the NHAs petition for lack of merit. SC ruled that NHAs action was barred by the decision of the CFI of Riza l regarding the validity of the OCT based on judicial estoppel. Respondents filed an action for Recovery of Possession and Damages against the NHA. NHA alleged that respondents title, was obtained fraudulently because the land was declared alienable and disposable only on January 1986.

The case was sent into archives and resumed upon the denial by this Court of the NHAs petition. The Decision of the Trial Court In favor of respondents. The trial court ruled that the dismissal of NHAs complaint for expropriation and for declaration of nullity of OCT in the names of Pedro and Nicanora left NHA with no right to hold possession of respondents property which was admittedly a part of Pedros land. The trial court ruled that this Court already declared respondents as the bona fide owners of the land and as such, their right to possession and enjoyment of the property becomes indisputable. NHA appealed. The Decision of the Court of Appeals The Court of Appeals denied the NHAs appeal. The Court of Appeals ruled that the main issue raised by the NHA was already resolved by the SC. The SC already declared that the NHA was judicially estopped from assailing the OCT. The Court of Appeals further ruled that this Court already declared that the NHA acted in bad faith when it took possession of respondents property in 1976 despite knowledge of the ownership of the Baello and Rodriguez heirs. The Court of Appeals also sustained the findings of the trial court that respondents were entitled to moral and exemplary damages as well as attorneys fees. The NHA filed a motion for reconsideration but was denied. ISSUES: Whether the Court of Appeals committed a reversible error in adopting the facts in the previous case when the case was not tried on the merits. HELD: NO RATIO: The petition has no merit. The doctrine of res judicata has been explained as follows: The rule is that when material facts or questions, which were in issue in a former action and were admitted or judicially determined are conclusively settled by a judgment rendered therein, such facts or questions become res judicata and may not again be litigated in a subsequent action between the same parties or their privies regardless of the form of the latter. Jurisprudence expounds that the concept of res judicata embraces two aspects. The first, known as bar by prior judgment, or estoppel by verdict, is the effect of a judgment as a bar to the prosecution of a second action upon the same claim, demand or cause of action.

The second, known as conclusiveness of judgment, otherwise known

as the rule of auter action pendent, ordains that issues actually and directly resolved in a former suit cannot again be raised in any future case between the same parties involving a different cause of action. x x x

on the title did not amount to a judgment on merits thus, precluding the applicability of res judicata.

The SC explained further that conclusiveness of judgment does not require identity of the causes of action for it to work. If a particular point or question is in issue in the second action, and the judgment will depend on the determination of that particular point or question, a former judgment between the same parties will be final and conclusive in the second if that same point or question was in issue and adjudicated in the first suit; but the adjudication of an issue in the first case is not conclusive of an entirely different and distinct issue arising in the second. Hence, facts and issues actually and directly resolved in a former suit cannot again be raised in any future case between the same parties, even if the latter suit may involve a different claim or cause of action. In this case, the NHAs petition is barred by conclusiveness of judgment which states that x x x any right, fact, or matter in issue directly adjudicated or necessarily involved in the determination of an action before a competent court in which judgment is rendered on the merits is conclusively settled by the judgment therein and cannot again be litigated between the parties and their privies whether or not the claim, demand, purpose, or subject matter of the two actions is the same. The SC sustained the Court of Appeals in ruling that the main issue raised by the NHA is the validity of OCT which had long been settled by this Court in a previous case. The SC noted that the Republic did not oppose Pedro and Nicanoras application for registration and neither did it appeal the decision. The OCT was issued by the Register of Deeds in 1959 and the Republic did not file any action to nullify the CFIs decision until the NHA filed a complaint for nullity on November 1993. The NHA was already barred from assailing the OCT and its derivative titles.

Issues: 1. w/n the dismissal of the complaint on the ground that is in the nature of collateral attack on the Certificate of title constitutes a bar to a subsequent petition Held: Petition denied. While the 2 titles are technically not barred by prior judgment as they involve different causes of action, the dismissal of petition is proper. The remedy of petitioner is to institute intestate proceedings for the settlement of the estate of the deceased spouses Lifted from original Petitioner claims that the determination of the RTC in Civil Case No. 0442 that she is an adopted child and that the signature of her adoptive mother Eligia in the deed of absolute sale transferring the subject land to Rogelio and Orlando was forged amounts to a new interest that should be reflected on the certificates of title of said land, or provides a reasonable ground for the amendment thereof. The Court disagrees for two reasons:

First. While the RTC may have made a definitive ruling on petitioner's adoption, as well as the forgery of Eligia's signature on the questioned deed, no partition was decreed, as the action was, in fact, dismissed. Consequently, the declaration that petitioner is the legally adopted child of Maximino and Eligia did not amount to a declaration of heirship and co-ownership upon which petitioner may institute an action for the amendment of the certificates of title covering the subject land. More importantly, the Court has consistently ruled that the trial court cannot make a declaration of heirship in an ordinary civil action, for matters relating to the rights of filiation and heirship must be ventilated in a special proceeding instituted precisely for the purpose of determining such rights.49cralaw virtualaw library Second. Petitioner cannot avail of the summary proceedings under Section 108 of PD 1529 because the present controversy involves not the amendment of the certificates of title issued in favor of Rogelio and Orlando but the partition of the estate of Maximino and Eligia who are both deceased. As held in Philippine Veterans Bank v. Valenzuela, 50 the prevailing rule is that proceedings under Section 108 of PD 1529 are summary in nature, contemplating corrections or insertions of mistakes which are only clerical but certainly not controversial issues.51 Relief under said legal provision can only be granted if there is unanimity among the parties, or that there is no adverse claim or serious objection on the part of any party in interest. This is now the controlling precedent, and the Court should no longer digress from such ruling.52 Therefore, petitioner may not avail of the remedy provided under Section 108 of PD

113. HILARIA BAGAYAS V. ROGELIO BAGAYAS Res Judicata; Rule 69! Facts: Petition for annulment of sale and partition Petitioner contends that he is an adopted child and therefore entitled to an aliquot share of the land in question Petitioner contends that the dismissal of his case (for annulment of sale and partition) on the ground that it was a collateral attack

1529. In fine, while LRC Nos. 08-34 and 08-35 are technically not barred by the prior judgment in Civil Case No. 04-42 as they involve different causes of action, the dismissal of said petitions for the amendment of TCT Nos. 375657 and 375658 is nonetheless proper for reasons discussed above. The remedy then of petitioner is to institute intestate proceedings for the settlement of the estate of the deceased spouses Maximino and Eligia. WHEREFORE, the petition is DENIED.chanroblesvirtuala RECALL: POSITOS VS. CHUA RULE 2; RULE 16!

UNDER BARANGGAY CONCILIATION FOR RULE 16 Sec 1 (j) Condition Precedent

RULE 17 DISMISSAL OF ACTIONS


114. PCI Leasing and Finance, Inc. vs. Antonio Milan, GR. No. 151215 (PENDING SEE FOLDER 6) 115. SUSIE CHAN-TAN v. JESSE C. TAN Rule 17 Sec 1 FACTS: Petitioner and respondent were married and were blessed with two sons. 12 years into the marriage, petitioner filed a case for the annulment of the marriage under Article 36 of the Family Code. The parties submitted to the court a compromise agreement regarding the separation of their assets and expenses thereto. The parties shall have joint custody of their minor children. However, the 2 minor children shall stay with their mother, Susie Tan. The trial court issued a partial judgment approving the compromise agreement. The trial court rendered a decision declaring the marriage void on the ground of mutual psychological incapacity of the parties. Respondent filed an omnibus motion seeking custody of the children alleging petitioner brought the children out of the country without his knowledge and without prior authority of the trial court; and petitioner failed to pay the remaining balance for one of the properties which, if forfeited would prejudice the interest of the children. The trial court, in its 17 May 2004 resolution, awarded to respondent custody of the children. Petitioner filed a motion for reconsideration alleging denial of due process on account of accident, mistake, or excusable negligence. She alleged she was not able to present evidence because of the negligence of her counsel and her own fear for her life and the future of the children. But such was denied by the trial court.

RECALL: PANGANIBAN v. PILIPINAS SHELL RULE 7; RULE 16 (E) Litis Pendencia!

PENTACAPITAL INVESTMENT CORPORATION v. MAHINAY G.R. No. 171736, July 5, 2010 Rule 7 Sec 5 Forum Shopping in relation to Rule 16 Sec 1 (e) Res Judicata!

RALLOS v. CITY OF CEBU GR No. 202651August 28, 2013 Rule 7 Sec 5 Forum Shopping; Rule 16 (e) Litis Pendencia

RECALL: HERNANDO BORRA vs. CA Rule 7 Sec 5 Forum Shopping; Rule 16 (g) Res judicata!

RECALL: LILY LIM vs.KOU CO PING a.k.a. CHARLIE CO G.R. No. 175256 | August 23, 2012 RULE 7 Sec 5 FORUM SHOPPING, RULE 16 Sec 1 (E) a LITIS PENDENTIA, OR RULE 16 Sec 1 (G )RES JUDICATA! RECALL: ESPINO v LEGARDA UNDER BARANGGAY CONCILIATION FOR RULE 16 Sec 1 (j) Condition Precedent RECALL: ZAMORA V IZQUIERDO GR NO. 146195 NOVEMBER 18, 2004

Petitioner filed a motion to dismiss and a motion for reconsideration of the 12 October 2004 Resolution. She claimed she was no longer interested in the suit. Petitioner stated that withdrawing the petition was for the best interest of the children. Trial court denied both the motion to dismiss and the motion for reconsideration filed by petitioner for having been filed beyond the 15-day reglementary period without any timely appeal having been filed by either party.

FACTS: V.C. Ponce mortgaged a parcel of land to PCIBank. The land was subdivided into 239 lots. Because of nonpayment of the obligation, the property was foreclosed by PCIBank then sold to Plaintiff Landcenter. V.C. Ponce filed a complaint against PCI Bank with the RTC of Pasig City for reconveyance of 54 lots, and for refund of the amount representing overpayment and unused letters of credit. By way of amicable settlement Landcenter and PCI Bank entered into a compromise agreement concerning the 54 lots. Instead of the 54 lots, however, Landcenter was to sign and reconvey to respondents merely 24 lots representing full and final compromise settlement of the RTCs judgment of reconveyance. An allegedly fake deed of assignment signed by Manuel Ponce (Manuel), as president of Landcenter, showing that the latter signed, transferred and conveyed to respondent V.C. Ponce two more road lots caused the filing a of a complaint (Civil Case No. 97-0532) by Landcenter with the RTC of Paraaque City. RTC ruled in favor of Landcenter. After failing to get a favorable decision, respondentfiled with the CA a petition for certiorari under Rule 65. Subsequently, Landcenter filed a Motion to Dismiss the aforementioned petition for certiorari in view of the withdrawal of its complaint before the RTC of Paraaque City in Civil Case No. 97-0532. The motion to withdraw was approved by the RTC after the petition for certiorari was filed. The respondents' counterclaims were also dismissed, thus, terminating the case before the RTC. With such dismissal, Landcenter opined that respondents' petition for certiorari should be dismissed for being moot and academic since Civil Case No. 97-0532 which was the basis of said petition for certiorari had been terminated.

ISSUE: Whether the denial of petitioners motion to dismiss is proper? HELD: Yes. RULING: Section 7 of the Rule on the Declaration of Absolute Nullity of Void Marriages and Annulment of Voidable Marriages provides: No motion to dismiss the petition shall be allowed except on the ground of lack of jurisdiction over the subject matter or over the parties; provided, however, that any other ground that might warrant a dismissal of the case may be raised as an affirmative defense in an answer. The clear intent of the provision is to allow the respondent to ventilate all possible defenses in an answer, instead of a mere motion to dismiss, so that judgment may be made on the merits. Since petitioner is not the respondent in the petition for the annulment of the marriage, Section 7 of the Rule does not apply to the motion to dismiss filed by her. Section 1 of the Rule states that the Rules of Court applies suppletorily to a petition for the declaration of absolute nullity of void marriage or the annulment of voidable marriage. In this connection, Rule 17 of the Rules of Court allows dismissal of the action upon notice or upon motion of the plaintiff. However, when petitioner filed the motion to dismiss on 4 November 2004, the 30 March 2004 decision and the 17 May 2004 resolution of the trial court had long become final and executory upon the lapse of the 15day reglementary period without any timely appeal having been filed by either party. The 30 March 2004 decision and the 17 May 2004 resolution may no longer be disturbed on account of the belated motion to dismiss filed by petitioner. The trial court was correct in denying petitioner's motion to dismiss. Nothing is more settled in law than that when a judgment becomes final and executory, it becomes immutable and unalterable. 116. LANDCENTER CONSTRUCTION AND DEVELOPMENT CORPORATION v. V.C. PONCE, CO., INC. AND VICENTE C. PONCE Rule 17 Sec 2

ISSUE: W/N the orders of the court after the withdrawal of the complaint by the petitioner from the trial court damaged or prejudiced the rights of the respondents HELD: Yes. Petitioners rights have clearly been prejudiced by the issuance of the courts assailed orders. RATIO: A dismissal upon motion of plaintiff under Section 2 of Rule 17 is allowed provided that it shall be without prejudice to the defendant. It bears stressing that the withdrawal of the complaint in the RTC by Landcenter operates to annul orders, rulings or judgments previously made in the case. It also annuls all proceedings had in connection

therewith and renders all pleadings ineffective. Quite simply, the withdrawal of the complaint results in placing them to their original position, as if no complaint was filed at all. This should be so, otherwise, a plaintiff can peremptorily withdraw his complaint after securing an order favorable to him. The orders of the RTC are considered vacated.

117. BENEDICTA SAMSON vs. HON. JUDGE FIEL-MACARAIG Section 3, Rule 17

FACTS: In 1998, petitioners Benedicta M. Samson and Marcial M. Samson obtained a loan amounting to P10,000,000 from FEBTC. The loan was secured by a real estate mortgage over four parcels of land located in Marikina City and covered by Transfer Certificate of Title (TCT). When petitioners failed to comply with the terms of the loan agreement, FEBTC filed an application for extra-judicial foreclosure of the real estate mortgage with the Office of the Clerk of Court and Ex-Officio Sheriff of the Regional Trial Court (RTC) of Marikina City. FEBTCs application was given due course, and a Notice of Sheriffs Sale was issued, setting the public auction sale of the mortgaged properties. Prior to the sale, the Notice of Sheriffs Sale was duly published in Ri zal-Metro Gazette and was certified by Sheriff of the RTC of Marikina City to have been duly posted in three public places where the mortgaged real properties were located. On 8 June 2000, only one bidder, FEBTC, submitted its bid, thereby causing the sheriff to postpone the public auction sale to a later date in accordance with SC AM that in the event that there are less than two participants, it must be rescheduled. Sheriff made the required notices. On 29 June 2000, the mortgaged real properties were sold at public auction to FEBTC as the highest bidder, and a Certificate of Sale was issued in favor of the bank. Almost two years later, petitioners, together others, all surnamed Samson, filed a case for Annulment of Extra-judicial Foreclosure and/or Nullification of Sale and the Certificates of Title questioning the validity of the auction sale for alleged lack of posting and publication requirements. Later on, The Registrar of Deeds of Marikina City filed a Manifestation stating that the certificates of title subject of the case had already been cancelled and the titles to the mortgaged properties were consolidated in the name of BPI. The Registrar of Deeds also claimed that the complaint stated no cause of action against him for it mentioned no wrongful act on his part. A hearing on the application for a TRO was held. Private respondent BPI

filed its Answer with Counterclaim and Opposition. The RTC of Marikina City, Branch 192, issued an Order denying plaintiffs application for TRO and/or Writ of Preliminary Injunction. Six months later, or on 20 June 2003, the RTC issued an Order dismissing the complaint for failure to prosecute for an unreasonable length of time. Plaintiffs filed a Motion for Reconsideration, but this was denied by the RTC. Plaintiffs, except Benedicta and Marcial Samson, filed a Notice of Appeal. Benedicta and Marcial Samson filed with the Court of Appeals a Petition for Certiorari under Rule 65 of the 1997 Revised Rules of Civil Procedure. The Court of Appeals rendered judgment dismissing the petition. The appellate court ruled that a writ of certiorari lies only where there is no appeal or plain, speedy, and adequate remedy in the ordinary course of law. The availability of the right to appeal precludes recourse to the special civil action for certiorari. The RTC Order subject of the petition was a final judgment which disposed of the case on the merits; hence, it was a subject for an ordinary appeal, not a petition for certiorari. Petitioners filed the instant petition for review before this Court. ISSUE: Whether or not RTC was right in dismissing the case. HELD: YES. The appellate court correctly ruled that the petition for certiorari was not the proper remedy. A writ of certiorari lies only for an error of jurisdiction. It can be availed of only if the lower tribunal has acted without or in excess of jurisdiction, or with grave abuse of discretion amounting to lack or excess of jurisdiction, and if there is no appeal or any other plain, speedy, and adequate remedy in the ordinary course of law. Where the error is not one of jurisdiction but an error of law or fact which is a mistake of judgment, certiorari is not available. In such case, the remedy is appeal. Also, in said Order, the RTC dismissed the case with prejudice for failure to prosecute for an unreasonable length of time, pursuant to Section 3, Rule 17 of the Rules of Court which states, thus: Section 3. Dismissal due to fault of plaintiff. If, for no justifiable cause, the plaintiff fails to appear on the date of the presentation of his evidence in chief on the complaint, or to prosecute his action for an unreasonable length of time, or to comply with these Rules or any order of the court, the complaint may be dismissed upon motion of the defendant or upon the courts own motion, without prejudice to the right of the defendant to prosecute his counterclaim in the same

or in a separate action. This dismissal shall have the effect of an adjudication upon the merits, unless otherwise declared by the court. (Emphasis supplied) The RTC Order dated 20 June 2003 was a final judgment which disposed of the case on the merits. Even assuming that certiorari may lie, the Court still cannot grant the instant petition because the petitioners failed to show that public respondent, in issuing the assailed Orders, acted without or in excess of jurisdiction, or gravely abused her discretion amounting to lack or excess of jurisdiction. As mentioned earlier, the RTC issued the assailed Order in accordance with Section 3, Rule 17, in relation to Section 1, Rule 18 of the Revised Rules of Civil Procedure. There is no showing that the RTC judge issued the Order in a despotic or arbitrary manner, or that she was motivated by passion or personal hostility against petitioners. 118. SHIMIZU PHILIPPINES CONTRACTORS INC. V. MAGSALIN Section 3, Rule 17; Sec. 1, Rule 36; Rule 41! FACTS: The petitioner claims that one Leticia Magsalin, doing business as Karens Trading, had breached their subcontract agreement for the supply, delivery, installation, and finishing of parquet tiles for certain floors in the petitioners Makati City condomi nium project called The Regency at Salcedo. The breach triggered the agreements termination. When Magsalin also refused to return the petitioners unliquidated advance payment and to account for other monetary liabilities despite demand, the petitioner sent a notice to FGU Insurancedemanding damages pursuant to the surety and performance bonds the former had issued for the subcontract. Petitioner filed a complaint docketed as Civil Case No. 02-488 against both Magsalin and FGU Insurance. FGU Insurance was duly served with summons. With respect to Magsalin, however, the corresponding officers return declared that both she and Karens Trading could not be located at their given addresses, and that despite further efforts, their new addresses could not be determined. In an effort to assist the RTC in acquiring jurisdiction over Magsalin, the petitioner filed a motion for leave to serve summons on respondent Magsalin by way of publication. GU Insurance filed a motion for leave of court to file a third-party complaint. Attached to the motion was the subject complaint,with Reynaldo Baetiong, Godofredo Garcia and Concordia Garcia

named as third-party defendants. FGU Insurance claims that the three had executed counter-guaranties over the surety and performance bonds it executed for the subcontract with Magsalin and, hence, should be held jointly and severally liable in the event it is held liable in Civil Case No. 02-488. On December 16, 2003 the RTC issued a tersely worded orderdismissing Civil Case No. 02-488: ORDER: For failure of [petitioner] to prosecute, the case is hereby DISMISSED. SO ORDERED.

ISSUE/S: 1. Whether or not the dismissal order is void 2. Whether or not the appeal was properly filed under Rule 41 of the rules of court 3. Whether or not the dismissal of civil case no. 02-488 is supported by the facts of the case HELD: 1. The DismissalOrderis Void 2. The appeal was properly filed under Rule 41 of the Rules of Court 3. The Dismissal of Civil Case No. 02-488 is not Supported by the Facts of the Case RATIO: 1. The nullity of the dismissal order is patent on its face. It simply states its conclusion that the case should be dismissed for nonprosequitur, a legal conclusion, but does not state the facts on which this conclusion is based. The December 16, 2003 dismissalorderclearly violates Rule 36 Section 1 for its failure to disclose how and why the petitioner failed to prosecute its complaint. Thus, neither the petitioner nor the reviewing court is able to know the particular facts that had prompted the prejudicial dismissal. Thus the dismissal of Civil Case No. 02-488 constituted a denial of due process. Elementary due process demands that the parties to a litigation be given information on how the case was decided, as well as an explanation of the factual and legal reasons that led to the conclusions of the court.Where the reasons are absent, a decision has absolutely nothing to support it and is thus a nullity. 2. Since the dismissal of Civil Case No. 02-488 appears to have been rendered motuproprio (as the December 16, 2003 dismissalorder does not state if it was issued upon the respondents or the trial courts motion), the facts to be

determined by the CA should include the grounds specified under Section 3, Rule 17 of the Rules of Court. A court could only issue a motuproprio dismissal pursuant to the grounds mentioned in this rule and for lack of jurisdiction over the subject matter. These grounds are matters of facts. Thus, given that the dismissal order does not disclose its factual basis, we are thus persuaded that the petitioner had properly filed its appeal from the dismissal order under Rule 41 of the Rules of Court. 3. The developments in the present case do not satisfy the stringent standards set in law and jurisprudence for a non prosequitur.The fundamental test for non prosequitur is whether, under the circumstances, the plaintiff is chargeable with want of due diligence in failing to proceed with reasonable promptitude.There must be unwillingness on the part of the plaintiff to prosecute In this case, the parties own narrations of facts demonstrate the petitioners willingness to prosecute its complaint.

RULE 8, SEC. 10; RULE 35 SUMMARY JUDGMENT! RECALL: PBCOM V. SPOUSES GO GR No. 175514 February 14, 2011 RULE 8, SEC. 10; RULE 35 SUMMARY JUDGMENT! RECALL: SHIMIZU PHILIPPINES CONTRACTORS MAGSALIN Section 3, Rule 17; Sec. 1, Rule 36; Rule 41! RECALL: SALUDAGA v. COMELEC AND BALAG 189431 and 191120 April 7, 2010 RULE 7; RULE 39! INC. V.

G.R. Nos.

RECALL: MEDISERV INC. V. CA Rule 7; Rule 45/46! RECALL: SPOUSES VILLUGA CONSTRUCTION SUPPLY INC. Rule 10; Rule 26; Rule 35! vs. KELLY HARDWARE AND

RULE 18 PRE-TRIAL

RULE 19 INTERVENTION
RECALL: BPI v. LEE SEC 2 PARTIES-IN-INTEREST VIRTUAL PARTY; Rule 19 Intervenor; Rule 39

RECALL: ALFEO D. VIVAS v. THE MONETARY BOARD HIERARCHY OF COURTS AND CERTIORARI AND PETITION FOR REVIEW RECALL: SPOUSES CLEMENCIO C. SABITSANA, JR. and MA. ROSARIO M. SABITSANA v.JUANITO F. MUERTEGUI, represented by his Attorney-inFact DOMINGO A.MUERTEGUI, JR. JURISDICTION OF RTC OVER ACTION FOR QUIETING OF TITLE AND RULE 63!

RULE 20 CALENDAR OF CASES RULE 21 SUBPOENA

RECALL: CECILIO HERNANDEZ v. SAN JUAN-SANTOS WRIT OF HABEAS CORPUS, RULE 92, RULE 103! RECALL: AQUILINO Q. PIMENTEL, Jr., et. al., vs. SENATE COMMITTEE OF THE WHOLE REPRESENTED BY SENATE PRESIDENT JUAN PONCE ENRILE

You might also like